Share Penguins Medicine August-Septumber
Share Penguins Medicine August-Septumber
Open pop up
to see notes
1
1. Patient did surgery in sinuses I, later he developed numbness in lower lid and upper lip, what
is nerve injuries?
A. facial
B. Infraorbital
Answer is: B
3. Young Female with Migraine 2-3 attack per week. What is the treatment of acute attack?
A. Sumatriptan
B. Aspirin
Answer is: B
4. A patient presents with nausea and vomiting then develops bilateral cranial nerve palsies and
bilateral symmetrical descending weakness. He has no fever and has not urinated in the last
four hours. What is the most likely diagnosis? NEW20
A. generalized tetanus
B. Lead poisoning
C. Botulism
D. Myasthenia gravis
Answer is: C
5. A 23-year-old female is brought to the Accident & Emergency department with severe
weakness, shortness of breath and fatigability. She is previously diagnosed with myasthenia
gravis and is on treatment with pyridostigmine. What is the most appropriate next step?
NEW20
A. Plasmapheresis or Iv Immunoglobulins (IVIG)
B. Glucocorticoids
C. Urgent thymectomy
D. Azathioprine
Answer is: A
6. Pt with atrial fibrillation maintained on warfarin, develop intracranial haemorrhage, what you
will do?
A. Give Vitamin K
B. FFP
C. Give Heparin
Answer is: B
2
8. How to manage bronchiolitis?
A. Rehydration
Answer is: A
10. Patient presented with cloudy urine with bubbling for 2 months. He has A history of
recurrent left iliac fossa pain for the past 2 years. Colonoscopy: shows an arrow lumen
otherwise normal Cystoscopy: erythema on dome of the bladder. What is the diagnosis?
A. Bowel ischemia
B. Diverticular disease
C. Transitional cell carcinoma of the bladder
D. Squamous cell carcinoma of the bladder
Answer is: B
12. patient with dyspepsia for 3 weeks, medically free and can eat and drink solid and liquid.
What next management?
A. urgent endoscopy
B. refer to gastroenterology
Answer is: B
14. patient with fresh blood per rectal, they did NGT and it was greenish, colonoscopy report
show: normal has no colorectal ca, what is next step?
A. upper GI endoscopy
B. 99 tec
Answer is: B capsul is better
15. patient with GERD on maximum PPi dose with minimal response and Upper GI endoscopy is
normal, what to do next?
A. 24 PH monitoring
Answer is: A
3
16. Hepatitis B acute marker?
A. HBsAg
B. HBeAg
C. Anti-HBs
D. Anti HB core IgM
Answer is: D
18. A patient presented with bilateral lower limb edema and distended abdomen. On
examination, there are small dilated vessels on the abdomen. What could be the cause?
NEW20
A. Heart failure caput medusae due to PVH
B. End-stage renal failure
C. Liver cirrhosis
D. Bowel obstruction
Answer is: C
20. DM patient on Metformin 1g BID, random blood glucose and a1c still high, what to modify in
treatment?
A. increase Metformin dose.
B. give insulin.
C. add_ gliptin
D. add_ glenide
Answer is: C
21. Case with signs and symptoms of pancreatitis but amylase was high but not even reach
double normal value, what is the best initial investigation?
A. Urine amylase
B. CT scan
C. US
Answer is: C
4
22. CASE with head trauma and polyurea?
A. central DI
Answer is: A
24. Female patient C/O Generalized muscle and bone ache associated with abdominal cramp or
epigastric, this symptom resolved When She drink Cup of cold milk What is the diagnosis?
Lab Value: Ca+2 = Normal, Phosphate= Low, Vit D3= Low PTH= with upper normal level.
What do you suspect?
A. Milk Alkali Syndrome
B. Primary hyperparathyroidism
C. 2ry hyperparathyroidism
Answer is: C
25. patient present with signs & symptoms of DKA. His Glucose is 34 mml. What is the goal in
this management?
A. Decrease glucose by 3 mml per hour 10%/h = 34*10/100= 3.4
B. Decrease glucose by 6 mml per hour
C. the glucose level goal should be above 18 mml
Answer is: A
26. DKA treated with IV, what about insulin way of administration?
A. Insulin infusion (IV regular insulin)
Answer is: A
27. patient diagnosed with acromegaly what you do for him in future?
A. transoesophageal echo
B. colonoscopy
Answer is: B
5
30. Drug decrease risk of stone in hyperuricemia?
Answer is: thiazide
32. A man with flank pain, dysuria - UTI what you’ll give?
A. Nitro
B. 3rd generation cephalosporin
Answer is: A, flank pain could be hint for pyelonephritis > B.
33. A 63 years old women presents to the Primary Care Physician with haematuria. She reports
having had a throat infection 2 weeks ago, for what her doctor prescribed penicillin V for 10
days. On the physical exam she has 150/90 blood pressure, her eyelids and both her ankles
are swollen. What is the treatment she must receive now?
A. Prednisone
B. Furosemide
C. Hydrochlorothiazide
D. Ondansetron
Answer is: B
35. Patient presents with splenomegaly, jaundice and spherocyte red blood cells on peripheral
blood smear. Which of the following is the most likely diagnosis?
A. Immune thrombocytopenia (ITP)
B. Autoimmune haemolytic anaemia
C. Primary myelofibrosis
D. Aplastic anaemia
Answer is: B
36. Which of the following therapy is useful to increase the production of fetal hemoglobin levels
(HbF) in patients with sickle cell disease?
A. Deferoxamine
B. Folic acid
C. Hydroxyurea
D. Penicillamine
Answer is: C
6
37. Known case of sickle cell disease presented with unilateral lower limb pain since of short
period. Vital signs: Tachycardia. Fever 38.3. The range of movement is intact with no signs of
inflammation over the limb. Which of the following is the most likely diagnosis?
A. Osteomyelitis
B. Vaso-occlusive crisis
C. Deep venous thrombosis
D. Cervical nerve compression
Answer is: B
39. A patient with a long history of bleeding, epistaxis and ecchymosis presents with lab results
showing anaemia, thrombocytopenia and leukopenia. Which of the following is the most
likely diagnosis? NEW20
A. Aplastic anaemia
B. Iron deficiency anaemia
C. Haemolytic anaemia
D. Kidney failure
Answer is: A
40. A young female patient presents to the emergency department with abdominal pain,
vomiting, and fever. Physical exam reveals splenomegaly and mild jaundice. Ultrasound
reveals pigmented gallstones. Which of the following is the most likely diagnosis? NEW20
A. G6PD deficiency
B. Thalassemia
C. Hereditary spherocytosis
D. Paroxysmal nocturnal haemoglobinuria
Answer is: C
41. A patient with a history of sickle cell disease presents to your office with progressive hip pain.
Which of the following is the most likely diagnosis?
A. Avascular necrosis
B. Developmental dysplasia of the hip
C. Spontaneous fracture
D. Deep venous thrombosis of the hip
Answer is: A
7
42. patient Known case of SLE came with on diagnosed with UTI Which of the following drugs is
Contraindication?
A. Cephalexin
B. Amoxicillin
C. Nitrofurantoin
D. Sulfamethoxazole
Answer is: D
44. Valvular lesion, round, well defined patch, scales, and plagues. You suspect psoriasis. How to
confirm the diagnosis?
A. clinical diagnosis
B. Biopsy
Answer is: B, N.B: psoriasis is always a clinical diagnosis. Skin biopsy is used as a definitive
diagnosis
47. Open fracture came 4 days later to ER with signs of infection (blue, necrosis) what's the most
common organism?
A. Actinomyces israelii
B. Staph aureus
C. clostridium
Answer is: C
48. Pt with TB. CXR revealed pleural effusion. How to knowledge the effusion is due TB?
A. Pleural protein over serum is < 0.3
B. Pleural LDH > 100
C. Pleural WBC > 1000
D. Pleural LDH over serum > 1/3
Answer is: C
8
49. patient suspected Tuberculous pleural effusion, what is the sensitive test that support your
diagnosis?
A. AFB of pleural effusion
B. High lymphocytes in pleural effusion
C. Low glucose in pleural effusion
D. Elevated pleural fluid ADA levels
Answer is: D
50. Patient came from India with diarrheal, what is the causative organism?
Answer is: E. coli
51. patient in his 20s complains of anal pain for 2 years, on examination there was a cauliflower
polyp that are painful when touched, what's your diagnosis?
A. Fibroepithelial polyps
B. Condylomas acuminate
C. Anal cancer
D. Rectal cancer
Answer is: B
55. A married male patient has history of unprotected sex. Three days later, he developed
purulent discharge. It was examined, and yielded a gram-negative intracellular diplococcus.
What is the most likely causative microorganism?
A. Streptococcus pneumoniae
B. Treponema pallidum
C. Neisseria gonorrhoeae
D. Chlamydia trachomatis
Answer is: C
9
56. What is the optimal duration of antibiotic treatment for Streptococcal pharyngitis? NEW20
A. 7 days
B. 5 days
C. 10 days
D. 3 days
Answer is: C
57. A patient on first line anti-TB medication complains of numbness and paresthesia. What
should be administered to him?
A. iron
B. Thiamine
C. Pyridoxine
D. Vitamin A
Answer is: C
59. 45 years old male presented to you with the history of cough from 3 months and white
colored patches around lips and inside the mouth with infections. Upon taking history doctor
reveals that patient was suffering from TB in previous 3 years ago. Which one of the
following can be the correct answer according to this statement? NEW20
A. Chronic cutaneous Candidiasis
B. Pulmonary emboli
C. Psoriasis
Answer is: A
60. Which of the following is the most specific test for pulmonary tuberculosis?
A. C reactive protein
Answer is: incomplete Q. ADA
61. A 25-year-old teacher complains of abdominal pain, fatigue, itching, and dark urine. On
exam, the liver is palpable 3 cm below the right costal margin. Two students are also having
the same complaints. What is the most likely culprit?
A. HAV
B. HBV
C. HCV
D. HDV
Answer is: A
10
62. Patient was found to have cavity on x ray, what is the type of precaution?
A. Airborne
Answer is: A
63. patient with MERS treated with vancomycin with 1000 unit over 20 mint then he develops
face swallowing and flashes what to do?
A. stop infusion and mark the pt allergic
B. Continue with slow rate
Answer is: B
64. UTI patient allergic to penicillin and sulfa and shellfish what drug can be used?
A. nitrofurantoin
B. Amoxicillin
C. Trimethoprim sulfamethoxazole
D. Cephalexin
Answer is: A
65. A patient presents to the emergency department in hypovolemic shock after being involved
in an automobile accident. The patient is hypotensive and tachycardic. Hemoglobin level is 6
g/dL. Which of the following is the best next step in management if all items are readily
available? NEW20
A. Whole blood transfusion
B. Packed RBCs
C. Ringer’s lactated.
D. Normal saline
Answer is: B
66. A patient presents to the emergency department with his wife, who claims the patient took
too many of his digoxin medication. What is the next best step in management?
A. Fb immunoglobulin
B. Calcium Chloride
C. Protamine sulfate
D. N-acetylcysteine
Answer is: A
11
67. An 82-year-old man presents to the emergency department with a 24-hour history of
progressive headache, vomiting, and dizziness. The patient is accompanied by his wife, who
is experiencing similar symptoms. The wife was asymptomatic prior to finding the patient
confused and disoriented. On physical examination, both the man and his wife are awake
and confused. The patient attempts to get up, but is overwhelmed with generalized
weakness and has difficulty walking to the bathroom. Which of the following is the most
likely diagnosis? NEW20
A. Migraine headache
B. Acute coronary syndrome
C. Carbon monoxide poisoning
D. Depression
Answer is: C
68. Patient watching TV, he says the one who is talking in TV is the god, what does this patient
have?
A. mania
B. schizophrenia
Answer is: B
69. Female 30 y/o with arthralgia and fever, harsh systolic murmur radiates to neck, previous 2
weeks has throat infection on urine analysis show: Proteinuria and haemorrhagic urine what
is the Dx?
A. SLE
B. infective endocarditis
C. post GN
Answer is: B
72. Patient with heart failure and admitted for surgery for some disease and connected to IV
fluid, post-op 2 days later complained of SOB and bilateral basal crepitation, how could this
be prevented?
A. monitoring IV fluids daily
Answer is: A
12
73. Heart failure due to left ventricular hypertrophy?
A. Diastolic dysfunction
Answer is: A
76. year old patient, HTN on Hydrochlorothiazide (I think the dose 20), for 5 years, no
improvement, what to do?
A. Increase the dose of Hydrochlorothiazide
B. Add BBB
C. Add CCB
Answer is: C
77. ECG with tall T wave and potassium level 6.5 immediate action?
A. calcium gluconate
B. insulin
C. dialysis
D. sodium bicarbonate
Answer is: A
78. Case of 60 years old male medically free came for routine checkup, everything is normal, and
he doesn't complain about anything, when you calculate his risk for 10 years risk of MI it was
6.9 by score, what you will do for this patient?
A. CT Angio
B. MRI cardiac
C. Stress Echo
Answer is: reassurance if not in choice > C.
79. What's the minimum number of days should do muscle strengthening activities?
A. everyday
B. Every two days minimum is 3 times per week
C. every week
Answer is: 5 days if not in choice > C.
13
HBV 30
80. Risk for hep C after needlestick?
HIV 0,3
A. 3%
Answer is: A
82. lung cancer risk increases with smokers for how many folds?
A. 30 folds
Answer is: A
14
87. Malaria is a life-threatening disease that usually transmit through bite of infected anopheles'
mosquito. A man who is planning to travel such areas where he can be bitten by mosquitoes.
What the best prophylaxis you will recommend him? NEW20
A. Antibiotics
B. Bite prevention
C. Residual sprays and lotions
D. Vaccination
Answer is: A
88. The patient is diagnosed with rheumatoid arthritis and is on aspirin he developed symptoms
of heartburn which was relieved with antacids, what will you give him? NEW20
A. Continue antiacid
B. Sucralfate
C. Misoprostol
D. H2 blocker
Answer is: C
89. nurse got bitten by patient with hepatitis B + and she is vaccinated, what to give her?
A. vaccine + IG
B. IG
Answer is: B
90. A 65-year-old male known case of liver cirrhosis and ascites came to your clinic complaining
of significant weight loss, which of the following tests you should do next?
A. Abdominal US
B. CEA
C. CA125
D. Alfa-fetoprotein
Answer is: A
91. A 93 yo man has dementia and multiple sicknesses and was found to have cancer and
metastasis to liver and lung and lymph nodes how will you manage ?
A. chemotherapy
B. Palliative surgery to improve his breathing I think
C. Watchful care
Answer is: B
92. Pt with bronchogenic carcinoma, presented with progressive SOB, there’s elevated jvp, clear
lung and quiet heart sounds. What will confirm your dx?
A. CXR
B. ECHO
C. ECG
Answer is: B
15
93. Patient of ALL post chemo develops headache and dry mouth what is the electrolyte
derangement?
A. Hypocalcaemia
B. Hypercalcemia
C. Hyponatremia
D. Hypernatremia
Answer is: A
94. patient with a diagnosis of lung cancer and has facial flush and other sign of Superior vena
cava obstruction. Which type of lung Ca the patient has?
A. squamous cell horner syndrome
B. Adenocarcinoma
C. small cell lung cancer
Answer is: A
95. patient with early satiation and muscles wasting, and supraclavicular lymph nodes. What is
Diagnosis?
A. gastric Ca
Answer is: A
96. Myasthenia gravies case) pt well morning and deteriorating throughout the day with Bilateral
ptosis, what medication to give?
A. pyridostigmine
Answer is: A
97. Elderly K/c case of long time poorly controlled DM2 and hypertension she is on aspirin
atenolol metformin, insulin, multivitamin She complains of dizziness when she stands what is
the cause? In examination There is decreasing of BP when standing but the HR didn’t
change? NEW20
A. Side effects of the medication.
B. Autonomic neuropathies
Answer is: B
98. Case of dm pt with sx of 3rd nerve palsy (ptosis, double vision when looking laterally) Dx?
A. Dm 3rd nerve neuropathy
Answer is: A
99. pt have difficulty feeding cannot put fork to his mouth and have traffic accident and where is
lesion?
A. cerebellum
Answer is: A
16
100. Case of Cn3 palsy with intact and reactive pupils, etiology?
A. posterior communicating aneurysms
B. Diabetic cranial nerve 3 palsy
C. Cavernous sinus thrombosis
Answer is: B
101. A 33-year-old woman is seeing you with a chief complaint of “dizziness.” Upon further
characterization, she describes a “spinning” sensation and a sense of “falling forward.” Based
on this description, which of the following terms should be used to characterize her
complaint? NEW20
A. Vertigo
B. Orthostasis
C. Presyncope
D. Dysequilibrium
Answer is: A
102. A 37-year-old male complains of severe headaches that typically involve his right eye, and
often cause the eye to tear. The headaches occur at about the same time each day and recur
for several days in a row before remitting. He reports that he is currently experiencing a third
episode of these headaches. Which one of the following therapies will help prevent future
recurrences of this patient’s headache?
A. Oxygen
B. Sumatriptan
C. Lithium
D. Verapamil
Answer is: D
103. A patient is brought to the emergency department (ED) with seizures for the past 25
minutes. He has a history of epilepsy but missed the anti-epileptic drugs for the past 48
hours as he was on a vacation trip. After stabilization in ED, he was given 10 mg of
intravenous diazepam with no benefits. What will be the next step in the management?
A. phenytoin
B. Ethosuximide
C. Midazolam Infusion
D. Lamotrigine
Answer is: A
17
104. A 25-year-old male presents to neurology clinic with a complaint of severe headache for the
past 3 days. The headaches are unilateral, around right-eye, severe in intensity, lasting for
around 45 minutes, and associated with right eye lacrimation and rhinorrhea. These episodes
typically occur at night around the same time and causing the patient to wake up from sleep.
He had experienced similar episodes 3 years ago that lasted for one week. They were
triggered by alcohol. What is the best abortive treatment for this condition?
A. Carbamazepine
B. 100% oxygen
C. NSAIDs
D. Verapamil
Answer is: B
105. A female patient presented with headache on the temporal lobe. During clinical
examination, on palpation of temporal lobe tenderness is felt. What complication is she at
highest risk of developing? NEW20
A. Blindness Giant cell arteritis
B. Coronary artery disease
C. Ischemic stroke
D. Brain tumor
Answer is: A
106. 64 old male comes to your office accompanied by his wife because of tinnitus that has
affected both ears for the last 3 years. It has been most troublesome at bedtime. His wife
says that he is becoming irritable and depressed because he is bothered by the buzzing in his
ears many times during the day. His only medication is allopurinol for the prevention of gout.
The most likely identifiable cause of this patient’s tinnitus is? NEW20
A. medication
B. Meniere’s disease
C. temporomandibular joint dysfunction
D. sensorineural hearing loss
Answer is: A
107. patient with tremor increase with activity like writing associated with head hobbing What is
treatment?
A. propranolol
B. Levodopa if resting tremur
Answer is: A
18
109. Nerve sensation of the pinna of the external ear?
A. Great auricular nerve
B. lesser occipital nerve
C. Cervical nerve
Answer is: A
110. patient with PE scenario, on radio saddle shaped PE was seen and on vital signs patient was
hypotensive, what's the treatment?
A. Alteplase
B. Warfare
C. LMWH
D. Unfractionated heparin
Answer is: A
112. smoker came with dyspnea and hemoptysis, examination and x ray showed positive pleural
effusion, but the trachea and the apex beat where not shifted what is the cause of non-
shifting trachea and apex beat?
A. transudate pleural effusion
B. TB pleural effusion
C. cancer in the ipsilateral main bronchi
Answer is: C
114. A patient presents with an asthma exacerbation. Which drug will decrease the mucous
secretion more than bronchodilation?
A. Cromolyn sulphate
B. Ipratropium
C. Montelukast
D. Prednisone
Answer is: D
19
115. You are treating a 52-year-old woman with a 40-pack-year history of smoking. She reports a
productive cough that has been present for the last 3 to 4 months, beginning in the fall. She
remembers having the same symptoms last year in the fall, and attributed it to a “cold that
she just couldn’t kick.” She does not have fevers, reports mild dyspnoea when walking
upstairs, and denies haemoptysis. Which of the following is the most likely diagnosis? NEW20
A. Irritation of airways from cigarette smoke
B. Chronic bronchitis
C. Postnasal drainage due to seasonal allergies
D. Lung cancer
Answer is: B
116. COPD patient on Salbutamol & now not controlled what to add?
A. ICS
B. Tiotropium
C. Ventolin
D. LABA
Answer is: B
117. A female patient who has progressive Dyspnea and fatigue. On auscultation, she has fine
crackles. While examining her legs she has tender red induration on her shin. What is the
best next step?
A. Oral steroid
B. Chest X-Ray
C. LMWH
Answer is: B
118. Patient with CHD developed epigastric pain and increase bowel movement?
A. Pancreatitis
B. Acute mesenteric emboli
Answer is: B
120. Fever and Chronic diarrheal for 10 months meal Paraumbilical pain?
A. Crohnʼs disease.
B. Celiac
Answer is: A
20
121. A 33-year-old healthy nonsmoking man presents to you for evaluation of his cough. He says
the cough has been present for about 8 weeks. After a few days, he went to an urgent care
where he received antitussives and a bronchodilator. Those did not help, and he returned 2
weeks later and was given a course of azithromycin. His cough has persisted. He complains of
an associated sore throat, but no fever. He reports worsening nighttime symptoms, and
drinking coffee seems to precipitate the cough. Which of the following treatments is most
likely to help? NEW20
A. A proton pump inhibitor Reflux aspiration
B. An antihistamine
C. An angiotensin-converting enzyme (ACE)-inhibitor
D. A steroid inhaler
Answer is: A
123. A patient presented with vomiting and hematemesis having multiple erosions and ulcers in
oesophagus and stomach what will you screen him for
A. Ca
B. Gastrin
C. Vasoactive intestinal peptide Zollinger-Ellison syndrome
Answer is: B
124. Osteoporosis is one of the extraintestinal manifestations of Crohn’s disease, what of the
following will increase its risk if associated with CD?
A. age 20-40
B. premenopausal
C. long term steroids
D. extensive bowel involvement
Answer is: C
126. Patient develop black tarry stool after H. pylori treatment drugs, which drug cause it?
A. Bismuth
Answer is: A
21
127. Pt heavy smoker with symptoms of GERD, upper endoscopic examination show’s squamous
cell with high-grade dysplasia, next step in management?
A. Council smoking cessation
B. Endoscopic mucosal resection
C. refer for possible esophagectomy
Answer is: B
129. An old male patient was admitted as a case of large intestinal obstruction, underwent rigid
sigmoidoscopy which showed a mass in the sigmoid region. A biopsy was taken and resulted
as adenocarcinoma, what's your next step?
A. Colonoscopy
B. CT abdomen
C. MRI pelvis
D. Sigmoidectomy
Answer is: A
131. Young man history of 6 months of episodes of watery diarrhoea and constipation & he’s
totally free in between. Tx?
A. Metronidazole
B. No Tx
C. Mesalazine
D. Steroids
Answer is: B
22
132. Q about diagnosed MALT lymphoma & helicobacter was confirmed what to do?
A. Resection anastomosis
B. Chemotherapy
C. Eradicate helicobacter
D. Antacids
Answer is: C
134. Old patient with chronic hepatitis B jaundice, asymptomatic, liver enzymes up what are
treatment?
A. entecavir
Answer is: A
136. Elderly with abdominal pain, jaundice, gallbladder palpable but no tender?
A. Liver cancer
B. Pancreatic cancer
C. Primary Choledocholithiasis
D. Secondary Choledocholithiasis
Answer is: B
137. Patient in day 8 or 9 post-cholecystectomy, develops right mouth corner pain and fever
38.5. What is your management?
A. paracetamol
B. Antibiotics
C. CT
Answer is: B
139. Which of these are associated with severe eye symptoms in thyrotoxic pt?
A. Male gender
B. Resistance to thiouracil drugs
Answer is: Smoking
23
140. 55 years old, male, have a family history of diabetes, HgA1C is 6.3, fasting 7.3 what will you
do next?
A. Random glucose
B. Fasting after 3 months
C. HgA1C after weeks
D. 2 hours of glucose
Answer is: D
141. A 50-year-old woman presented with sudden onset shortness of breath, PND, and feelings
of drowning. On examination, she has pulmonary edema and s3 gallop. She is also a known
case of Diabetes Mellitus and was currently on metformin for glycaemic control. Which of
these medications is considered safest for glycaemic control of this patient?
A. SGLT2 inhibitor
B. Thiazolidinedione
C. Dipeptidyl-peptidase IV inhibitor
D. Biguanide
Answer is: A
143. Diabetic patient suffers from fasting hypoglycaemia and postprandial hypoglycaemia, he is
on Glargine and Aspart. What to do?
A. stop both drugs
B. decrease both drugs
C. continue same drugs
Answer is: B
145. A 70-year-old smoker woman with low vitamin D and osteoporosis. Which of the following
has the highest risk for osteoporosis? NEW20
A. Vitamin D deficiency
B. Obesity
C. Age
D. Smoking
Answer is: C
24
146. patient with signs of Cushing (thin skin, buffalo hump ...etc) Not on any medication and lab
show: low ACTH what is the next test?
A. CT adrenal
B. MRI pituitary
Answer is: A
148. asymptotic patient and no goiter His lab show: TSH (high) T4 (N) what are the appropriate
next step?
A. Reassurance and discharge
B. Repeat the test after 6w
C. give thyroxine
Answer is: according to TSH level, more than 10 > C, less than > B.
150. A patient presented with the classical triad of polydipsia, polyphagia, and polyuria. A
diagnosis of Diabetes Mellitus was made. Which of these changes is observed in Diabetes
Mellitus?
A. Decrease in lysosomal enzymes
B. Impaired phagocytosis
C. Decrease in protein breakdown
D. Increase in mitochondrial plasticity
Answer is: B
151. patient with cons disease cause of hypokalaemia despite having iv and oral?
A. Check aldosterone renin ratio
B. observation
Answer is: check Mg level, if wasn’t there > A.
25
152. old female presented with hypertension, proximal muscle weakness, weight gain especially
in the upper body and purple striae on the lower abdomen. Which of these tests gives a clue
about the etiology of the most probable diagnosis?
A. High-dose dexamethasone suppression test
B. 24-hour urinary free cortisol screening
C. Low-dose dexamethasone suppression test
D. Midnight salivary cortisol
Answer is: A
153. ICU patient immobile for 8 months, his labs show normal PTH and phosphate. What is the
cause of High calcium levels in this patient?
A. Malignancy Multiple myeloma
B. Primary hyperparathyroidism
C. Immobility
Answer is: C
154. Female complain of fatigue and she has scar turned hypo pigment (dark), Addison case he
asks what test to do?
A. synth Test Synacthen test
Answer is: A
156. Young patient T1DM on rapid and long-acting insulin with hypoglycaemia, what’s the next
appropriate management?
Answer is: decrease the level of rapid and long
157. A patient develops hyperthermia, generalized muscle rigidity, altered mental status,
tachycardia, hypertension. labs show elevated creatine kinase. He started metoclopramide
two weeks ago to treat diabetic gastroparesis. Which of the following drugs can also cause
these symptoms?
A. Nortriptyline
B. Fluphenazine
C. Methamphetamine
D. Tranylcypromine
Answer is: B, N.B: case of NMS.
26
159. Which type of renal stone is associated with infections? NEW20
A. Calcium oxalate
B. Uric Acid
C. Struvite
D. Cystine
Answer is: C
160. patient on many hypertension drugs with a symmetrical kidney what is the diagnosis?
A. Renal artery stenosis
Answer is: incomplete Q, could be PCKD if symmetrical large kidney.
161. patient with DM and HTN on many drugs and has grade 3 limb edema , lab shows low
creatinine clearance what to do?
A. stop ACEi
B. change foresomide to thiazide
Answer is: A
163. 60 years old patient after 10 minutes of blood transfusion developed pain in the site of the
cannula, difficulty breathing, and is febrile. What is the most likely blood transfusion
reaction?
A. Febrile non-haemolytic anaemia
B. Haemolytic anaemia
C. Allergic reaction
D. Acute haemolytic reaction
Answer is: D
164. old with DVT which of the following indicate thrombophilia test?
A. age.
B. Hx of OCP use.
C. Connective tissue disease
Answer is: C
165. long case and investigation high Ferritin and low lymphocyte) what is the diagnosis?
A. Adult still disease
Answer is: A
27
166. SLE on prednisone with neurological manifestation what add?
A. Atropine
B. Abx
C. Cyclophosphamide
D. Chloroquine
Answer is: C
167. RA patient on methotrexate and NSAID but still not control, what to add?
A. Adenolimab
Answer is: A
168. RA with only joint manifestation not respond to NSAIDS what is the treatment?
A. Metho
B. Cyclo
C. Azithromycin
D. Sulfa
Answer is: A
171. Patient was found to have cavity on x ray, what is the type of precaution?
A. Airborne
Answer is: A
172. MRI was given diagnosed with toxoplasmosis, what is the TTT?
Answer is: Pyrimethamine + sulfadiazine
173. Schistosomiasis patient presents with dyspnoea, what complication most likely happened?
A. Pulmonary HTN
B. Pericarditis
Answer is: A
28
174. Pt presented with urosepsis; urine culture revealed E. coli >100000. TTT? NEW20
A. Meropenem
B. Nitrofloratin
C. Ceftriaxone
Answer is: A
175. patient presents with seizure and Hx of multiple sexual partners in past 6y, what
investigation you will order?
A. HIV
B. Syphilis CSF
Answer is: A
176. A 55-year-old patient with HIV presents with shortness of breath and a productive cough.
Work up includes a bronchoalveolar lavage showing soap bubble appearance. What is the
most likely organism involved? NEW20
A. Cryptococcus neoformans
B. Aspergillus Fumigatus
C. Streptococcus pneumoniae
D. Pneumocystis jiroveci
Answer is: A
177. patient on TB drug and has elevated uric acid level which drug caused this?
A. Pyrazinamide
Answer is: A
178. Patient with painless ulcer + skin rash. what is your diagnosis?
A. Secondary syphilis
Answer is: A
180. Patient came from India with diarrhoea, which organism may it be?
A. E. coli
B. C. jejuni
C. difficile
Answer is: A
29
181. Hypertensive patient on lisinopril 20 mg and amlodipine 5 mg. Recently developed
thromboembolism so warfarin was prescribed in 2 mg. Shortly he develops a productive
cough and fever then sputum for AFB requested and result appear + ve, so anti-tuberculous
was started. What is the next step?
A. Decrease lisinopril dose.
B. Increase amlodipine dose. clopidogrel should be in ques
C. Increase warfarin dose.
D. stop rifampicin
Answer is: C
182. A 30-year-old man with a history of Crohn disease develops an enterocutaneous fistula and
is placed on total parenteral nutrition through a right subclavian central venous catheter.
After 5 days, the patient develops a fever and leucocytosis; CT scan of the abdomen reveals
no intra- abdominal abscess. The subclavian catheter insertion site is inspected and noted to
be erythematous and painful. Blood cultures are positive. Which of the following organisms is
the most likely cause of his fever? NEW20
A. Coagulase-positive staphylococci
B. Coagulase-negative staphylococci
C. Group A Streptococcus Enterococcus
Answer is: B
183. A patient with end-stage liver disease, on central venous line, septic, blood showed OOD.
C/S shows budding yeast. What antifungal is appropriate at this stage? NEW20
A. Caspofungin
B. Flucytosine
C. Fluconazole
D. Itraconazole
Answer is: A
184. A healthy 50-year-old male consults you about preparations for a business trip to India. He
will be traveling in rural areas at times and has read that he should have antibiotics with him
in case of traveller's diarrheal. Which one of the following would be best for you to
prescribe? NEW20
A. Trimethoprim/sulfamethoxazole (Bactrim)
B. Azithromycin (Zithromax)
C. Ciprofloxacin (Cipro)
D. Rifampin (Rifadin)
E. Doxycycline
Answer is: C
30
185. Which one of the following comorbid conditions increases the risk that latent tuberculosis
infection will progress to active disease? NEW20
A. Hypertension
B. Lung cancer
C. Obesity
D. Coronary artery disease
Answer is: B
186. A man with multiple sexual partners (Hx of unprotected sexual encounters) has decreased
LOC and other CNS symptoms. Toxoplasmosis is confirmed (they gave the results) and asking
what is the best treatment?
A. Pyrimethamine + sulfadiazine
B. trimethoprim/sulfamethoxazole
Answer is: A
187. Genital ulcer then develops the palm and sole rash (diagnosis)?
A. Chlamydia
B. Chancroid
C. Herpes
D. Syphilis
Answer is: D
189. Patient with cough and haemoptysis. On auscultation: consolidation on the right upper lung
zone. What will help in confirmation of diagnosis? Most specific?
A. PPD test
B. Sputum culture
C. Chest X-ray
Answer is: B
190. Young Patient with fever and dark urine has history of constipation, urinalysis: high
leukocyte, high erythrocytes, high protein Case of pyelonephritis + Nitrate in UA, what the
best next step? (fever, chills, pyuria, vomiting + flank pain + hx of UTI) + look for pyuria?
A. Oral Abx
B. IV Abx
Answer is: B
31
191. A 22-year-old gravida 1 para 0 with a history of homelessness and recent intravenous drug
use presents for prenatal care. Mantoux tuberculin testing is performed and produces 20
mm of induration. She had a negative tuberculin test 1 year ago. A chest radiograph is
normal. You decide to treat her with isoniazid for latent tuberculosis infection. Which one of
the following should also be given to prevent the development of peripheral neuropathy?
A. Folate
B. Vitamin B6
C. Vitamin D
Answer is: B
192. Low-grade fever, o2 sat 88, hypotension 90/50 what is the initial step of management?
A. Iv fluid
B. O2
C. Antipyretic
Answer is: B
193. Patient has sepsis, what are the things you will not need?
A. IV fluid
B. Inotropes
C. IV Abx
D. Steroid
Answer is: D
194. A patient presents to the emergency department after spontaneously collapsing while
waiting for the bus. The patient does not have any history of cardiac abnormalities. Which of
the following is the most likely diagnosis? NEW20
A. Cardiac syncope
B. Vasovagal
C. Seizure
D. Orthostatic hypotension
Answer is: B
195. A patient presents to the emergency department after ingesting organophosphates and is
treated with atropine. Which of the following is the most common side effect of atropine?
NEW20
A. Bradycardia
B. Dry mouth
C. Miosis
D. hyperhidrosis
Answer is: B
32
196. A patient presents to the emergency department with organophosphate poisoning. Which
of the following is the correct antidote?
A. Atropine
B. Propranolol
C. Methacholine
D. Pralidoxime
Answer is: A
197. Patient with MVA and hit his head. XRAY AND CT are unremarkable but he is complaining of
a headache. what is your diagnosis?
A. Concussion
B. hematoma epidural
C. hematoma subdural
Answer is: A
198. A patient post TB bronchiectasis presents with cough, hemoptysis. CXR shows right upper
lobe cavitary lesions. What is the best initial next step?
A. Place the patient in the right lateral decubitus position
B. Chest physiotherapy
Answer is: A
199. A patient is playing soccer outside when he is stung by a bee. After a few minutes the
patient is in respiratory distress and collapses. Which of the following is the best immediate
treatment for this patient?
A. Antihistamine
B. Epinephrine
C. Norepinephrine
D. Dopamine
Answer is: B
200. patient on TPN developed weakness in the lower limbs and tremors. what is the problem?
A. Hyperkalaemia
B. Hypomagnesemia
C. Hypoglycaemia
D. Hyperphosphatemia
Answer is: B
33
202. 67-year-old male presents to the emergency department with abdominal pain that radiates
to his back. He has a medical history of poorly controlled hypertension. He also has a history
of atherosclerosis and had a myocardial infarction two years ago. He smokes one pack of
cigarettes daily. His vital signs reveal: Temperature 37.3°C, Blood pressure 98/60 mmHg,
Heart rate 116/min, Respiratory rate 22/min. The patient is diaphoretic. Further physical
examination is significant for a pulsatile abdominal mass just above his umbilicus that is
tender to palpation. The next step in this patient’s workup is? NEW20
A. Computed tomography
B. Angiography
C. Chest radiography
D. Magnetic resonance imaging
Answer is: A
203. Elderly pt with CHF on ACE, spironolactone, furosemide, admitted for some reason I forgot,
upon admission she was on fluids? Labs show she’s hypotensive with increased urea and
creatinine and FENa 0.6 What to do? NEW20
A. Increase furosemide.
B. Increase spironolactone
C. Increase fluids carefully
Answer is: C
204. A 24-year-old male who was in a high-speed motorcycle collision undergoes emergent
splenectomy. His estimated blood loss was 1500 mL. His family prohibited to transfuse blood.
Which of the following strategy should be employed for his resuscitation? NEW20
A. Synthetic colloids should be administered as the primary resuscitation fluid in a 3:1 ratio
to replace the volume of blood loss.
B. Vasopressors should be primarily utilized for the maintenance of his blood pressure.
C. 0.45% normal saline should be administered in a 3:1 ratio to replace the volume of blood
loss.
D. Lactated Ringer solution should be administered in a ratio of 3:1 to replace the blood loss
Answer is: D
205. Case of patient complaints of abdominal pain, exam tenderness and. The pulsatile mass
above the Umbilicus What is the most appropriate investigation?
A. US
B. CTA
C. angiography
Answer is: A
34
206. young healthy man after long plane travel developed chest pain and dyspnoea. He’s tall and
thin, chest X-ray showed pneumothorax (without any details or measures). What to do?
A. Discharge
B. NSAIDs
C. Thoracostomy
Answer is: C
207. pt with MI (give information and vital signs with increase JVP), what is the tx of shock?
A. Dopamine
Answer is: A, according to full scenario.
208. DM, htn, claudication after walk 200 m, no distal pulse what to do?
A. heparin and follow up
B. surgical catheter
Answer is: A, if there was risk factor stratification is more accurate.
210. pt with dm, smoker, with peripheral artery disease, what would you do to improve his
condition?
A. dual aspirin.
B. smoking cessation program
C. Exercise program
Answer is: B
211. PT pos-inferior MI, few hours developed hypotension, raised JVP, clears lungs on
auscultation?
A. Right ventricular infarction
Answer is: A
212. Case of inferior MI present with clear lung sound, JVP, hypotension not mentioned muffled
sound and no murmur ?
A. Papillary muscle rupture harsh systolic murmer
B. Right ventricular infarction
C. Cardiac tamponade
Answer is: B
35
213. The systolic murmur that decreases by Valsalva manoeuvre and increases by hand grip ?
A. AS
B. MR
MR
C. MS AR
D. AR Diastolic
Answer is: B
215. pt in 20s c/o sharp pain centrally after activity for 3 weeks .. what next?
A. Reassure and follow up
B. Nitrate muscular pain
C. Ibuprofen
Answer is: C
217. You have diagnosed a 66-year-old woman with heart failure. She has a history of
hypertension, but has never had heart failure before. Which of the following tests is routinely
indicated in the initial evaluation of a person with a new diagnosis of heart failure?
A. Echocardiogram
B. Holter monitor
C. Left heart catheterization
D. Treadmill stress test
Answer is: A
218. Elderly female with inferior MI, hypotension & JVP raised, what is the cause?
A. Rt Ventricular infarction
B. left V infarction
C. tamponade
D. complete heart
Answer is: A
219. Adult known Asthmatic and Heart disease, presented to you with increasing SOB 3 weeks
ago, + Ejection Systolic Murmur, grade 4/6. that confirms the diagnosis?
A. Transoesophageal Echocardiogram (TEE)
Answer is: TTE
36
220. pt medically free and asymptomatic have screen and discover have diastolic murmur 2/6
what to do?
A. transthoracic echo (TTE)
B. Transoesophageal echo
Answer is: A
221. Patient on ACE develop Dry cough, tongue and facial swelling(angioedema), with
hyperkalaemia 6.5 what to switch it with?
A. Switch to losartan (ARB)
Answer is: A
222. Patient k/c of DM HTN came with unstable angina treated with a statin, aspirin, b blocker,
heparin, nitrate What to add?
A. Candesartan
Dual antiplatlets
B. Clopidogrel
C. Warfarin
Answer is: B
224. Patient with a clinical feature of heart failure. What is the associated enzyme?
A. CRP
B. Troponin
C. BNP
Answer is: C
225. 59-year-old man is planning to undergo a coronary artery bypass. He consumes aspirin over
the past year. Which of the following is the most appropriate treatment prior to surgery to
minimize his risk of bleeding? NEW20
A. Give fresh frozen plasma few hours before surgery.
B. Stop the aspirin 1 week prior to surgery.
C. Stop the aspirin 1 to 2 days prior to surgery.
D. Begin vitamin K 1 week prior to surgery.
Answer is: B
37
226. 61-year-old white male with type 2 diabetes mellitus sees you for a follow-up visit. His
blood pressure is 156/94 mm Hg. At a visit 1 week ago his blood pressure was 150/92 mm
Hg. Laboratory studies obtained prior to this visit show a BUN of 16 mg/dL (N 6–20), a serum
creatinine level of 0.9 mg/dL (N 0.7–1.3), and microalbuminuria on a urinalysis. His diabetes
is well controlled with metformin (Glucophage) and he is taking aspirin. Which one of the
following would you recommend?
A. Observation only
B. An ACE inhibitor
C. A -blocker
D. calcium channel blocker
Answer is: B
227. Elderly women in her 70s with DM & HTN, normal renal profile, which antihypertensive
should be started?
A. thiazide
B. ACEI
Answer is: B
229. Patient happen to her chemical burn during resuscitations they remove her close and found
chemicals powder what next step?
A. swipe the powder
Answer is: irrigation with saline, if wasn't there > A.
230. Which of the following ways of transmissions is the most difficult to prevent? NEW20
A. Person to person transmission
B. Sexual transmission
C. Airborne transmission
D. Vector transmission
Answer is: C
38
232. You are caring for a 26-year-old man with dyslipidaemia and a family history of early
coronary arterial disease. Laboratory analysis reveals a low HDL. Which of the following
interventions, if adopted by the patient, would raise his HDL levels to the greatest extent?
NEW20
A. Eat oat bran
B. Lose weight
C. Start exercising
D. Quit smoking
Answer is: C
233. 36 yr man, and identified that he meets criteria for therapeutic lifestyle changes to improve
his lipid profile. After 4 months of adhering to your recommendations, his LDL is still higher
than goal. Which of the following drug classes should be initiated?
A. statin
B. bile acid sequestrant
C. Nicotinic acid
D. A fibrate
Answer is: A
234. You are evaluating a 36-year-old obese woman who complains of fatigue. She denies
polydipsia, polyuria, polyphagia, or weight loss. Which of the following laboratory
reports\confirms the diagnosis of diabetes?
A. random glucose reading of 221 mg/dL
B. random glucose reading of 221 mg/dL, and another, on a later date, of 208 mg/dL
C. A fasting glucose measurement of 128 mg/dL
Answer is: B
235. A healthy 33-year-old male sees you for a pre travel consultation. He plans to attend a 4-
week South American, you discuss immunizations, malaria prophylaxis, injury prevention,
and traveller's diarrhoea. Which one of the following interventions is most likely to prevent
traveller's diarrhoea? NEW20
A. Avoiding food street
B. Avoiding the use of ice in beverages
C. Taking a prophylactic antibiotic
D. Washing hands frequently
Answer is: A
39
237. A 43-year-old female smoker has type 2 diabetes mellitus, morbid obesity, and a recent
diagnosis of symptomatic peripheral arterial disease. You have started her on atorvastatin
(Lipitor), offered a supervised exercise program, and discussed smoking cessation and
interventions. Which one of the following should be recommended to prevent cardiovascular
events in this patient? NEW20
A. Aspirin
B. Cilostazol
C. Enoxaparin
Answer is: A, if D is stop smoking is more accurate.
238. An old patient presented with retroperitoneal haemorrhage. History of Hodgkin lymphoma.
What's the diagnosis?
A. Lymphosarcoma
B. Liposarcoma
C. Sarcoma
Answer is: C
239. 18 Y has ALL. 17 days after chemo he develops fever around 38.7 C. No focus of infection
found. Labs shows: low WBC 0.6 (normal was 4-10x..), Neutrophil level 60% (normal 40-
60%), i’m not sure if he has low platelets as well, what you’ll do?
A. Blood urine, ... culture and paracetamol
B. Blood, urine, ... culture and PO Abx
C. Blood, urine, ... culture and IV Abx
Answer is: C
240. Patient known case of polymyalgia rheumatica presents with malignancy symptoms (weight
loss with fatigue), CBC shows high lymphocyte count what is the diagnosis?
A. CLL
Answer is: A
241. A middle-age company worker with mild depression and he has interpersonal conflicts with
colleagues?
A. Psychotherapy CBT
B. Antidepressants
C. SSR
Answer is: A
40
242. A patient presents to the emergency department with euphoria, hypertension, and visual
hallucinations. The patient complains he can feel his heart beating. Which of the following is
the most likely diagnosis?
A. Acetaminophen poisoning
B. Amphetamine poisoning
C. Opioid overdose
D. Benzodiazepine overdose
Answer is: B
243. lady who lost her close friend in grief and complaining from trouble sleeping Which is the
best treatment?
A. Imipramine.
B. Chlorpromazine
C. Lorazepam
D. Fluoxetine
Answer is: C
244. A patient presents to the emergency department with his family after a drug overdose. The
family reports that the patient has a history of depression and high blood pressure. Now the
patient presents with palpitations, dry mouth and skin. Electrocardiography reveals
arrhythmia. Which of the following is the most likely causal agent?
A. SSRI
B. TCA
C. Acetylcholinesterase inhibitor
D. Beta Blocker
Answer is: B
245. A 55-year-old woman has alcohol use disorder and presents with glossitis, diarrhoea,
progressive memory loss, and hyperpigmented rash in sun-exposed areas. Which of the
following deficiency is the likely cause? NEW20
A. Vitamin B5 (panthothenic acid)
B. Vitamin C (ascorbic acid)
C. Vitamin B7 (biotin)
D. Vitamin B3 (niacin)
Answer is: A, if there was Vit B 12 is more accurate.
246. A patient presents with acute agitation with disorganized speech and behavior following a
6-week history of delusions, social withdrawal, and depressed mood, it is affecting his social
and occupational functioning. What is the likely diagnosis?
A. Delusional disorder
B. Schizophreniform disorder
C. Schizophrenia
D. Brief psychotic disorder
Answer is: B
41
247. heroin addiction?
Answer is: methadone
248. Case of that girl who ingested 2 tablets of paracetamol after one day she goes to the
hospital with N/V and RUQ pain what to give?
Answer is: acetylcysteine
251. septic arthritis started on cloxacillin: report came after 3 days with staph aureus resistant to
cefotaxime (cefoxitin)?
A. start Vanco
B. add gents
C. continue cloxacillin
Answer is: A
252. An 8 years old girl ingested 2 pack of paracetamol after a fight with her mother come after
a day of ingestion with RUQ pain in which stage of toxicity she is?
A. Stage 1
B. Stage 2
C. Stage 3
D. Stage 4
Answer is: B
253. patient known case of asthma on inhaled corticosteroids and short acting beta agonist as
needed what to add?
Answer is: Add LABA
254. Asthma pt with 6-8 times night symptoms. She is on SABA. What to add?
A. Inhaled corticosteroids/formeterol.
B. Inhaled corticosteroids and anticholinergic.
C. Inhaled corticosteroids and albuterol as needed
Answer is: A
42
256. K/O asthma came for check-up, on SABA, LABA, ICS, his last need for inhaler 1 year ago Pt
controlled What is your mX ?
A. Give monolocast
B. Duble dose of ICS
C. Stop Both ICS and LABA
D. Decrease dos of ICS
Answer is: D
258. which of the following indicate severe asthma? Baseline PEFR 400ml?
A. PEFR less than 300
B. RR>25
C. HR more than 100
Answer is: B
259. asthma patient not in any medication with no exacerbation, he gets some day symptoms 2-
3 time per week with no night symptoms what to give?
A. Saba
B. laba
C. ICS
D. SABA & ICS
Answer is: D
260. asthmatic girl, on SABA and they prescribed ICS twice daily. She’s very well controlled didn’t
use SABA at all for the past 2 years. What action should you do now?
A. stop both SABA and ICS
B. Decrease ICS dose by half and reassess after 6 months
Answer is: B
43
263. Patient 23 old came with murmur “forgot the description sadly”, she had history of TOF
repair at age 3 years?
A. VSD
B. MS
C. AR
D. PR
Answer is: D
264. known case of polymyalgia rheumatica presents with malignancy symptoms (weight loss
and other non-specific constitutional sx), what is the diagnosis? Labs showed anemia, low
platelets and increased WBC (lymphocytes)?
A. CLL
Answer is: A
265. 70 y/o female bed bounded due to basal ganglia (bleeding or damage can’t remember
clearly) , weak mastication muscles, absent gag reflex, losing wt, (no mention of coma) how
are going to feed her?
A. jejunostomy tube
B. NGT if acute
Answer is: A
266. 45 years old man had MVA presented with isolated head injury and coma for 5 days in ICU,
which of the following is best initial way of feeding?
A. Nasogastric tube feedings
B. Gastrostomy feeding
C. Central line feeding
D. Peripheral line feeding
Answer is: A
267. Which of the following should the asymptomatic people do screen for?
A. Colorectal cancer
B. Ovarian cancer
Answer is: A
268. Patient “forgot age” known history of diabetes mellitus, complaining of ptosis, inability to
adduct, elevate and depress right eye. Upon examination pupils are reactive bilaterally what
is the most common cause?
A. posterior communicating artery aneurysm
B. cavernous sinus thrombosis
C. 3rd nerve neuropathy
Answer is: C
44
269. pt with HTN, DM with well demarcated non tender swelling? NEW20
A. erysipelas
B. Necrobiosis Lipoidica Diabeticorum
Answer is: B
270. retrosternal & epigastric pain & regurgitation & heart burn, did barium X ray show
dilatation of oesophagus and lower oesophagus stenosis and there’s multiple contraction in
it (no oesophageal movement) what’s the diagnosis?
A. GERD
B. achalasia cardia
C. diffuse spasmatic oesophagus
Answer is: B
271. heart burn & regurgitation NOT respond to PPI, endoscope see esophagitis, what you will
do?
A. PH 24 hrs if normal endoscopy
B. Manometry
Answer is: B
272. 65 YO female, with change with bowl habit only (no weight loss or any cancer signs)?
A. Anal cancer
B. anal warts cauliflower ?
Answer is: A, according to full scenario could be rectal cancer.
273. which prevent or decrease incidence of getting post herpetic neuralgia? NEW20
A. Amitriptylin
B. Acyclovir
C. Varicella vaccination
Answer is: B
274. Obese diabetic hypertensive female patient tried lifestyle but it was not effective. BMI is 28.
What is the best drug with tolerable side effects?
A. Orlistat
B. Lorcaserin
C. Phentermine-topiramate
Answer is: A
275. Pt with bronchogenic carcinoma, presented with progressive SOB, there’s elevated JVP,
clear lung and quiet heart sounds. What will confirm your dx?
A. CXR
B. ECHO
C. ECG
Answer is: B
45
276. patient with raised JVP, Hypotension, unclear heart sound, clear lung sounds. How to
confirm the Dx?
A. Echo
B. CXR
C. ECG
Answer is: A
277. Healthy asymptomatic woman with grade 2 holo systolic murmur heard at apex and radiate
to axilla, how to confirm diagnosis?
A. Echo
B. ASO titer MR
Answer is: A
46
283. Pt complaining of fever and headache with picture of blood smear (schistocytes) high aptt
and pt , low fibrinogen What is the diagnosis?
A. DIC
B. ITP
C. TTP
Answer is: A
284. Patient has allergy after taking contrast CT what initial to do?
A. adrenaline
B. Intubation
C. Steroid
Answer is: A
286. Left eye unable to abduct, when looking forward there is squint towards the nose which
nerve is affected?
A. Left 6th Cranial nerve
Answer is: A
287. Patient with hyperlipidaemia started on anti-lipid then developed facial flushing, the
doctors prescribed to him aspirin to relive this adverse effect?
A. niacin
B. Atorvastatin
C. Cholestyramine
Answer is: A
47
291. Patient with memory loss and change in behavior?
A. front temporal dementia
Answer is: A
292. Nurse work in pulmonary clinic C/O with fever, productive cough. She is not responding to
AB (Xray show opacity left upper lobe)?
A. Streptococcus pneumoniae
B. Hemophilus pneumonia
C. Mycobacterium
Answer is: C
293. Highly suspicion of malaria in person who came from Sudan, his blood film was negative,
what to do?
A. Repeat thin film
B. Repeat thick film
C. Repeat every 8h for 48h
Answer is: C
295. patient going to a country with malaria endemic, which method of prophylaxis is the best?
A. Chemoprophylaxis
B. Vaccination
C. Immunization (i think it was like this)
D. don't go to endemic country
Answer is: A
296. Mother and her child visited the paediatrician for 6 months vaccination, however the
mother stated that her child was hospitalized after receiving the 4 months vaccination he
develops anaphylaxis, what is the right thing to do? NEW20
A. Test the child for skin test
B. Give him all vaccines except DTP
C. Vaccinate him and discharge home
D. Vaccinate him and observation for 1 hour
Answer is: A
297. asking about which of the following Malaria drugs Highly resistant? NEW20
Answer is: chloroquine
48
298. patient came with hx of vomiting multiple time what's the finding?
A. hypokalaemia
B. metabolic acidosis
C. respiratory alkalosis
D. high urine K
Answer is: A
300. 57-year-old man has just gone PCI without stent placement for evaluation of unstable
angina. He has been placed on nitroglycerin, BB, and aspirin, but still feels chest pain with
activity. Lipid profile was provided (and all were on the upper limit of normal, but not
elevated). What would you add to this man’s medications?
A. Atorvastatin
Answer is: A clopdogrel is better answer
301. Elderly with meningitis, what prophylactic TTT to give contact people?
A. ceftriaxone
B. Cipro
C. Rifampin
Answer is: C
302. A 25-year-old man was referred to the Endocrine Clinic with a recent history of fragility
fracture. He had no significant past medical history and was on no medication. On
examination, he had sparse facial and axillary hair growth. Which of the following
investigation useful in establishing the aetiology of his fragility fractures?
A. Calcitonin level
B. DEXA
C. Insulin like growth factor (IGF1)
D. testosterone and gonadotrophin levels
Answer is: D
303. 59 Y.O. female Patient diagnosed with MI and was treated at the hospital, upon her
discharge she asked you what is the best way to prevent reinfarction?
A. Use of antiplatelet for short duration.
B. Use of Calcium channel blocker indefinitely.
C. Use of ACEI if she develops Heart failure to prevent cardiac changes,
D. Use of spironolactone for long duration
Answer is: C
49
304. 34 years old nulliparity with previous HX of DVT, drugs will give this patient?
A. Enoxaparin
B. Heparin
C. Warfarin
D. No need treatment
Answer is: A
305. 30 years old postpartum developed DVT and was started on enoxaparin 80 mg bid. On the
same admission she developed sudden onset of shortness of breath and right pleuritic chest
pain (PE). BB 125/70 HR 100 RR 22 temperature 36.6 oxygen saturation 95% CT shows
thrombus in right lower pulmonary artery. What is most appropriate?
A. Change enoxaparin to sodium heparin
B. Thrombolytic therapy
C. Same management
D. Thrombectomy
Answer is: C
306. 56 years old, with of HTN, medication Losartan, Amlodipine, failed to control his HTN, all
labs normal, which best drugs can be giving and (or not contraindication)? NEW20
A. Lisinopril
B. Atenolol
C. Indapamide
D. Can’t remember maybe dijxon
Answer is: C
307. patient with painful bilateral inguinal lesion, vesicles, most common cause?
A. HSV
B. Syphilis painless
Answer is: A
309. Young female presented with dyspnoea, low grade fever and arthralgia. On examination
there was tender erythematous nodules on her shin. And on auscultation basal crackles.
What is best next step in management?
A. Chest X Ray
B. Steroid sarcoidosis
C. Blood culture
D. skin biopsy
Answer is: A
50
310. Pheochromocytoma (High blood pressure high catecholamine, how to treat?)
A. Alpha blocker
Answer is: A
311. man has HTN and his family father and 2 sisters have HTN, medically free before, O/E and
investigation all with normal limits, his BMI 27 and his blood pressure 160/100, what is the
appropriate management?
A. ACE
B. alpha blocker
<55 year or diabetic ACE
C. BB >55 CCB
D. CCB
Answer is: A
312. 29 years old woman complaining of dysuria and urgency, allergic to penicillin, shel fish and
TMX, What to give?
A. Cephalexin
B. Nitrofurantoin
C. TMX
D. Amoxicillin
Answer is: B
315. Pt DM and hypertension. And k/c of chronic HF. Past history of MI. Has decreased systolic
ejection fraction with no symptoms. His K is elevated (~6). Blood pressure controlled
110/70s. He’s on ACEI, frusemide, Atorvastatin. What should you do regarding his med?
NEW20
A. Add BB
B. D/c ACEI
C. D/C statin
D. D/c furosemide
Answer is: B
51
316. hepatitis C needle stick injury, chance of transmission?
A. .03 %
B. .3 % HIV
C. 3 %
D. 30 % HBV
Answer is: C
317. Female pt C/O generalized muscle and bone ache associated with abdominal cramp or
epigastric this symptom resolves when she drink a cup of cold milk what is the diagnosis? Lab
result: ca+2 = normal, phosphate =low, vit D3 = low, PTH = with upper normal level?
A. milk alkali syndrome.
B. Primary hyperparathyroidism
C. 2ndry hyperparathyroidism
Answer is: C
318. Pt had stones and abdominal pain relived with a cup of milk, labs (high Ca, high PTH)?
A. Primary hyperparathyroidism
B. Secondary hyperparathyroidism
C. Milk Alkali Syndrome
Answer is: A
322. COPD patient present acutely with signs of CHF with SaO2: 86% and ABG: Normal PH, PaO2
> 60 mmhg, PaCo2 < 45 mmgh, asking what to give?
A. Diuretic
B. O2
Answer is: B
323. Young female smoker patient recently diagnosed with SLE, life style modification?
A. Smoking cessation
Answer is: A
52
324. SLE patient what antibiotics are contraindications as they cause SLE flare?
A. Sulfa
Answer is: A
325. Do you think you are mentally retardant? The doctor here wants to assess?
A. insight
B. judgment
Answer is: A
326. Patient had a long flight then admitted complaining of SOB, on examination patient is tall,
investigation showed pneumothorax, what is the appropriate management?
A. Chest tube if>3cm
B. Conservative management if <3cm
Answer is: A
327. Old patient has hypercalcemia, what is the important initial step?
A. Normal saline
B. IV pamidronate
Answer is: A
328. HYPERTENSION And HF AND LV hypertrophy no valvular lesion. What the cause of HF?
A. Dysfunction in CO
B. DIASTOLIC DYSFUNCTION
C. STOLIC DYSFUNCTION
Answer is: B
329. case of gun shot, Raised JVP and no heart sounds, best appropriate management
A. Pericardiocentesis
B. Thoracotomy
Answer is: A
330. Stab wound in the chest. Came with weak thread pulses, raised JVP, EBAE. What's the
diagnosis?
A. Pneumothorax.
B. Cardiac tamponade.
C. Pulmonary contusion
Answer is: B
331. Child with stab wound in the lower right chest, he is hypotensive and tachycardiac, what
the most appropriate thing to do?
A. Thoracotomy
B. tube thoracostomy
C. exploration wound
Answer is: B
53
332. Bilateral pyelonephritis in old age. What is the Cause?
A. BPH
B. prostate CANCER
Answer is: A, Short recall > read the full scenario in exam answer may be difference.
333. 60 y/o male known case of hypertension; he came for toteine follow up. Examination
revelled Ejection systolic murus at right sternal border radiating to carotid. Loud S2. What is
the diagnosis?
A. Aortic stenosis
B. Pulmonic stenosis
C. HOCM
Answer is: A
335. Pt with CKD has hyperkalaemia what should the pt eat with same amount everyday?
A. Green leaves
B. Grapes
C. Tomatoes
Answer is: C, food rich in potassium
336. Which one of the following diseases consider Pulmonary HTN class 3?
Answer is: Lung disease
338. women just delivered 2 days ago from unbooked marriage, still not breastfeeding, which
method is saved?
A. HIV women direct breastfeeding
B. HIV women expressed milk
C. TB women direct breastfeeding
D. TB women expressed milk
Answer is: D
54
339. Female coming back from east Asia, typical presentation of TB with chest X ray showing
cavitation, what will you do next?
A. Sputum for acid fast bacilli
B. Start anti tb
Answer is: A, if there was isolation is more accurate.
340. Pt with ascending limb weakness with history of gastroenteritis 3 weeks back GBS what’s
the prognosis of these diseases?
A. deterioration
B. resolve with weakness
C. full recovery
Answer is: C
341. A tall patient complaining of dyspnea and chest pain. CXR: Opacity on the upper border of
the right lung that is less than 3 cm. What's the management?
A. Tube thoracostomy.
B. US guided needle biopsy
C. Observation
Answer is: A, N.B: No needle thoracostomy in choices
342. Pt with CHF and Afib. What will you use to control his cardiac rhythm?
A. Bisoprolol
B. Amiodarone
Answer is: B, N.B: for Rate control: in compensated > BB, CCB, uncompensated > digoxin.
343. patient with DM, smoker, with peripheral artery disease, what would you do to improve his
condition?
A. dual aspirin
B. smoking cessation
C. exercise program
D. strict glucose control
Answer is: B
344. ICU pt post operation complaining of high-grade fever with no other symptoms. What's the
management?
A. IV antibiotic
B. IV paracetamol
Answer is: A
55
345. Female had gastroenteritis and she took metoclopramide that leads to involuntary
movement facial grimace and tongue protruding what to give how to manage?
A. Diphenhydramine
B. Epinephrine
C. Cyproheptadine
D. Tizanidine
Answer is: A
346. Old patient develops productive cough, Fever, Dyspnoea on examination crepitation CXR
showed bilateral right middle lobe consolidation. They mention on lab Reticulocyte Cold
agglutination test What is the Dx?
A. BA
B. Atypical pneumonia
C. Streptococcus pneumonia
D. Exception of HF
Answer is: B
347. Adenoma was found incidentally in a pt complaining of recurrent headache. What's the
management?
A. Neurosurgery referral.
B. Order anterior pituitary hormones
Answer is: A
349. A 36-year-old woman presents to the Emergency Room with a 1-week history of fever,
dysuria, and flank pain. She is admitted to the hospital after fluid resuscitation (a total of 6
litres of isotonic saline) and started on IV antibiotics. She also requires a low dose of
norepinephrine to maintain her blood pressure. The day after admission, she complains of
shortness of breath and requires 5 liters of supplemental oxygen; oxygen saturation is 91%
after this management. Chest X-ray confirms new, bilateral alveolar infiltrates. Bedside
echocardiogram does not show evidence of left ventricular dysfunction, Blood pressure
78/34 mmHg, Heart rate 125 /min Respiratory rate 25/min Temperature 38.6, Oxygen
saturation 95%, Which of the following is the most likely diagnosis? NEW20
A. Diffuse alveolar haemorrhage
B. Cardiogenic pulmonary oedema
C. Healthcare-associated pneumonia
D. Acute respiratory distress syndrome
Answer is: D
56
350. DM with cardiac disease your goal in management?
A. triglyceride less than 150
B. BP less than 140/90
C. LDL less than 70
D. HDL more than 70
Answer is: C
351. Child with periorbital edema and lower limb edema , sacral edema and with high AST and
Amylase, what is the cause?
A. chronic liver failure
B. CHF
C. nephrotic syndrome
Answer is: C
352. 70 years old admitted to day care in the past 2 weeks he had decreased interest, decreased
weight and appetite he is away from his family and he has short term memory impairment
diagnosis?
A. Alzheimer
B. depression
C. vascular dementia
D. Delirium
Answer is: B
354. patient dm and has symptoms of UTI in lap high creatinine what’s contraindicated?
A. Nitrofurantoin
B. cipro
Answer is: A
57
357. simple case with low mcv and Hb?
Answer is: IDA
360. case of Sickle cell disease ask about initial step in mx?
Answer is: IVF and analgesia
361. Psychiatry Patient came with four months of hallucinations and delusions and behavioural
issues but no mood issues, then he became fine. What is your diagnosis?
A. schizophreniform
B. schizophrenia
C. something maybe like brief psychotic episode
Answer is: A
362. Which statement is correct regarding schizophrenia medications: (I don't remember the
statements exactly but im just trying)??
A. 90% of patients get significant improvement on them
B. 30% get improve enough to lead a functional life
C. 5% get Cured
Answer is: B
363. Patient i remember in his early 20s or 18, came with paroxysmal cough and many others
(most likely they are hinting for pertussis), which of the following is a fact on the vaccine for
this disease?
A. immunity lasts for 10 years
B. immunity lasts for 5 years
C. Not sure about other choices
Answer is: B
364. Patient came with new onset of psychotic symptoms (hallucinations bizarre behaviours and
others) how will you treat him?
A. CBT
B. CBT and antipsychotics
C. fluoxetine
D. olanzapine
Answer is: B
58
365. drug used for osteoporosis and malignancy AND bone metastasis?
Answer is: Denosumab
366. case of multiple gastroduodenal ulcers with UGIB, asking about what lab test to do?
A. fasting gastrin
B. VIP ZE syndrome
Answer is: A
369. COPD elderly admitted exacerbation at night became aggressive disoriented what the nurse
should do first?
A. Call the duty dr for lorazepam iv
B. Restrain her
C. Call family sit beside her
D. elevate head of bed put nasal oxygen try to oriented her about place and time
Answer is: D
370. Elderly in her 80s came with sever pneumonia and became confusion and there’s sever
disturb in sleep/wake cycle what you’ll do?
A. isolation for patient and keep her in quite dark room
B. keep the family with her
Answer is: incomplete Q, if there was haloperidol is accurate.
372. Patient 60 something k/c of dyslipidaemia on statin report high blood pressure reading of
150/90, In the clinic Labs all normal Bp: 145/90 ,What’s your management??
A. Start amlodipine
B. life style modification and weight reduction
Answer is: A, check the rest of answer.
59
373. 21 boys go to dentist clinic for tooth extraction he has bleeding after extraction and bruises,
teat of factor VIII activity (87%), lab show slight increase in aPTT?
A. Von wellbrand
B. haemophilia A
Answer is: A
375. 55 ys with chest pain for 2 months after exercise, ECG is normal?
A. reassurance
B. Stress Echo
Answer is: B, his age > risk for MI, but read full scenario could be muscle pain.
377. young male patient 30 y.o who developed over 1 hr sudden unilateral facial nerve palsy,
asking about Mx? NEW20
A. give thrombolytics
B. antiviral therapy
C. hyperbaric oxygen
Answer is: steroid, if not there > B.
378. Old pt k.c of asthma on salbutamol other med ranitidine+ candesartan + spironolactone his
k= 2.6 which of the following drug cause hypokalaemia in this pt?
A. Salbutamol
Answer is: A
379. 56 years old male Patient hypertension on diuretics Came with high bp reading, Labs: All
normal except Fasting 7.3 (high), What to add to his medication?
A. CCB
B. ACE
C. BB
Answer is: B
60
380. patient had loss of sensation of lower ear lobe ear pinna and skin covering angle of
mandible?
A. great auricular
B. lesser auricular
C. great occipital
D. lesser occipital i think
Answer is: A
384. patient with exudative pleural effusion had positive gamma interferon?
A. Empyema
B. Mesothelioma
C. Parapneumonic effusion
D. Tb pleural effusion
Answer is: D
386. Pt did orthopaedic surgery on heparin and start to have bruises, what is the diagnosis?
A. ITP
B. TTP
C. HIT
D. DIC
Answer is: C
61
387. A case of pt with psoriasis on topical steroids presented with Cushing sx, low ACTH, low
morning 8 am cortisol?
A. iatrogenic Cushing syndrome
B. Cushing syndrome
Answer is: A
389. case of typical Cushing features and increased weight 15 kg with recently diagnosed
diabetes...etc Labs done free urine cortisol high, Low dose dexamethasone high cortisol (All
given ranges), ACTH < 1 (low), What is the appropriate next step?
A. Late night salivary cortisol
B. Brain MRI
C. Adrenal imaging
Answer is: C
390. 18 Y has ALL. 17 days after chemo he develops fever around 38.7 C. No focus of infection
found. Labs shows: low WBC 0.6 (normal was 4-10x..), Neutrophil level 60% (normal 40-60%),
i’m not sure if he has low platelets as well, what you’ll do?
A. Blood urine... culture and paracetamol
B. Blood, urine, ... culture and PO Abx
C. Blood, urine, ... culture and IV Abx
Answer is: C
391. pt with hepatitis C recently diagnosed with +ve HCV Ab, -ve HCV RNA, what to do next?
A. repeated hepatitis C test
B. Perform ultrasound
C. don’t do anything
Answer is: C
393. female came her child has Sickle cell disease and she remarried again, came for pre-marital
screen, what test should order?
A. maternal genetic test
B. paternal genetic test
Answer is: paternal Hb electrophoresis
62
394. Nephrotic syndrome child with abdominal pain and distention what would be the
complication?
A. Peritonitis
B. appendicitis SBP
C. gastritis
D. hepatitis
Answer is: A
395. low pitch Mid diastolic murmur, rumbling in character heard best at apex?
Answer is: Mitral stenosis
396. Crohn’s with vomiting for 10 days, Patient develop refractory hypokalaemia despite
adequate replacement with KCL?
A. IV Mg sulphate
B. Spironolactone
Answer is: A
397. Pt post thyroidectomy had hypercalcemia didn’t improve even after iv and oral calcium.
What is ur management?
A. diuretic
B. Recombinant PTH
C. Check magnesium level
D. Iv potassium gluconate
Answer is: C
398. Pt has status epilepticus for 5 min, with iv access, what is the first line?
A. IV lorazepam
B. Rectal diazepam
C. buccal midazolam
D. Oral lorazepam
Answer is: A
399. 25-year-old female came with generalized seizure for 35 minutes and started on 20 mg iv
lorazepam and did not respond, what are you going to give her next?
A. Iv phenytoin
B. Iv phenobarbital
Answer is: A
63
400. urine incontinence male has problems with initiate urine, and after voiding still have
palpable bladder? what type?
A. Overflow
B. Urge
C. Reflex
D. Stress
Answer is: A
401. HBsAg +ve, Anti-HBc (IgG), +ve, HBeAg +ve, Anti-HBs -ve?
A. Immune because of vaccines of HBV
B. Immune because natural infection
C. Chronic infection
D. Acute infection
Answer is: C
402. HBV vaccine, 3 times, no previous infection, what serological markers will be positive?
NEW20
A. Hbs antibody
B. Hbc antibody
C. Hbe antibody
D. Hbe and hbs anti body
Answer is: A
403. 4-year-old male came with fatigue “prolonged diarrhoea” he drinks goat milk and picky in
his food, Labs: Anaemia with high MCV What’s the diagnosis? NEW20
A. IDA
B. giardiasis
C. celiac
Answer is: B
404. Paediatric patient diagnosed with celiac disease what food to avoid?
A. Barely
Answer is: A
405. Adult patient with bronchiectasis he’s on antibiotic and physiotherapy Presented with
sputum what to give?
A. Steroid
B. Supine sputum suction
C. The rest I exclude by me
Answer is: A
406. young patient with HTN everyone in the family has hypertension?
A. give alpha blocker
Answer is: ACE
64
407. Non-ACTH secreting adenoma, size on CT scan, what do you want to do next?
A. low dexamethasone suppression test+ urine metanephrine
Answer is: A
408. Patient 77 y/o on 3 antihypertensive medication, still not controlled, US was done and show
Asymmetrical kidney size, What is your dx?
A. adult PCKD
B. Renal artery stenosis
C. Other choices unrelated at all
Answer is: B
410. 35 Y.O. Pt was diagnosed with GERD and was started on omeprazole 40mg. he improved
and stopped it for months and symptoms came back. what is your next step?
A. Add ranitidine at bedtime.
B. Refer him for upper Gl endoscopy.
C. Increase omeprazole to 80mg.
D. same management
Answer is: D
411. Nurse work in pulmonary clinic C/O with fever, productive cough. She is not responding to
AB, (Xray show opacity left upper lobe)?
A. Streptococcus pneumoniae
B. Hemophilus pneumonia
C. Mycobacterium
D. Pulmonary TB
Answer is: D
65
413. patient known case of RA now present with acute knee pain swelling redness and effusion
showed WBC of 3500 culture pending febrile and tender on exam what to do?
A. IV AB
B. high dose IV steroids
C. intra articular steroids
Answer is: C
414. which one of the dipeptidyl peptidase-4 inhibitors used without limitation?
A. Sitagliptin
B. Saxagliptin
C. Linagliptin
Answer is: C
415. A Young male Presents with Altered sensorium, Fever, Nuchal rigidity And CSF analysis
shows high protein, high cells, normal glucose?
A. ceftriaxone + vancomycin
B. Ampicillin
C. Acyclovir
Answer is: C
416. scenario of elderly patient who had chest inf 4 weeks ago, initially improved on Abx, now
worsened again, CXR: right pleural effusion. Dx?
A. Lung cancer
B. TB
parapneumonic effusion
Answer is: C
417. 72 years old male, medically free and vitally stable came with afib. What of the following
should you give him?
A. aspirin
B. warfarin with INR 2-3
C. warfarin with INR 3-4
D. nothing
Answer is: B
418. Pt with CKD & A-fib what drug could give to prevent any future cardiac problems?
A. Warfarin
Answer is: A
66
419. long scenario of a patient with A-fib, and high CHADS score now developed MI, and PCI
done for him what is the Tx?
A. Didcon. Warfarin
B. Start on 2 antiplatelet
C. Start on 3 antiplatelet
D. cont. Warfarin and add 2 antiplatelet
Answer is: D
420. Long scenario at the end the Patient had schistosomiasis, what do predict to be associated
with?
A. Pulmonary HTN
Answer is: A
421. Patient you suspect chronic thromboembolic what’s the highest diagnostic test?
A. V/Q scan
B. Spiral CT with contrast
Answer is: A
423. S&S of Acute HF with and did O2 mention saturation, What’s next?
A. Furosemide
Answer is:A
425. CXR with scenario of sickle cell anemia (new one) and symptoms of respiratory destress,
what medication will decrease the incidence?
A. Hydroxyurea
Answer is: A, N.B: case of Acute chest syndrome
426. Know HCV pt came to the ER with massive oesophageal varices haemoptysis, on p/e he is
found to have tender ascites, Labs: Hb is 8, bp/ 100/90 What will reduce life mortality?
A. octreotide
B. blood transfusion
C. IV ceftriaxone
Answer is: C
67
427. SAAG 1.2, PMN 350 and in the history, he was confused?
A. Lactulose & IV ceftriaxone
Answer is: A, N.B: SAAG > Serum-ascites albumin gradient.
429. 50 yo pt. with 3wk H of fatigue and tiredness and tongue and lower limb she also has
unilateral tongue fasciculation. diagnosis is?
A. myasthenia gravis
B. Myasthenic syndrome.
C. Mononeuropathy.
D. Motor neuron disease
Answer is: D, the full scenario in 2019 Q bank there was both UMNL and LMNL.
431. Female with infertility and anxiety and depression, what is the feature that you will see in
clinic that make you suspect she is on depression and anxiety?
A. Eye contact
B. Fingers ....
C. Scratching
Answer is: A
432. Hypernatremia dehydration after IV fluids became comatose how to replace his fluid
deficit?
Answer is: Over 48 hours
68
434. smoking decreases survival rate?
A. 5
B. 10
C. 15
D. 20
Answer is: B
437. Pregnant Patient was having dyspnoea and cough of white sputum she tells you that she is
allergic to dust, fumes her cough started 2 days ago which was associated with clear fluid,
phlegm, what will you do next?
A. spirometry
B. Chest x ray
C. ABG
Answer is: A
69
441. Bipolar case asking about the most appropriate long-term treatment?
A. Lithium
B. Olanzapine
C. Mirtazapine
Answer is: A
442. case came with signs of aortic stenosis, what is related to the diagnosis? NEW20
A. the murmur increases when patient is standing
B. ECG will show RBBB
C. the main treatment for symptomatic patient is diuretics
D. associated symptoms of congestive heart failure indicate poor prognosis
Answer is: D
443. Female patient complaining of headache and myalgia for 5 day, 2 days ago she had fever
and rash on her face, 10 day ago she was in Jeddah, how would you treat her?
A. steroid
B. IV antiviral
C. IV antibiotic
D. supportive management
Answer is: D
70
447. Pt with DVT given thrombolytics and after that he developed bleeding, what reverse the
action of streptokinase?
A. vit k
B. activator of VIII
C. Aminocaproic acid
Answer is: C
449. electrolytes disturbance after TPN? Other recall: Patient on TPN, developed weakness and
convulsion what is the cause of these symptoms?
A. Hypokalaemia
B. Hypomagnesemia
Answer is: hypophosphatemia
450. Euthyroid sick syndrome > asking about thyroid hormones level?
Answer is: most prominent is low T3 and high reverse T3 (rT3)
451. Child with Abdominal pain, +ve urea breath test. Given triple therapy. How to know
improvement? NEW20
A. pH 24 hours monitoring
B. Clinical
C. Urea breath test
Answer is: B
453. 60ys DM, heavy smoking, Claudication while walking to pray in the mosque, 100 yards;
Intact femoral, weak peripheral Most important is?
A. Dual antiplatelets
B. Cessation smoking
C. restrict glycaemic control
D. involved in sporting program to increase his capacity
Answer is: B
71
454. patients on antihypertensive medications lisinopril, she develops dry cough, what
alternative drug that she can use?
A. propranolol
B. Enalapril
C. Losartan
Answer is: C
456. Which of the following increase risk for ischemic heart dis?
A. BMI 31
B. Waist cricmferness 103
C. Bp 137/80
D. 2spearate Fasting blood suger 7.3 7.6 or7
Answer is: D
457. year old woman known case of DM did cholecystectomy 2 years ago, she has been using
NSAID to relieve RA symptoms She develop HTN, Na normal, K Low, Cr higher than normal
150s what do think the cause of HTN?
A. Essential hypertension
B. pheochromocytoma
C. NSAID induced HTN
D. Primary hyperaldosteronism
Answer is: bad recall, by exclusion > C, NSAID will have high K.
459. man c/o unilateral knee pain, medical free before, lab show high PMNs, and positive for clot
( i think wifn clot something like this), what is the Dx?
A. Gout
B. Pseudogout
C. RA
D. Septic arthritis
Answer is: D
72
460. Patient in his 30s with right knee swelling Joint aspiration showed, Yellow, turbid, positive
mucin clot test, 15000 WBC, 90% PMN (no mention of crystals) What’s the most likely
diagnosis?
A. septic arthritis
B. Rheumatoid arthritis
C. Gout
D. Pseudogout
Answer is: A
461. man has severe pain in metatarsophalangeal joint, for couple of days, it's was red and
tender, what is Most likely Dx?
A. Ca crystal
B. Uric crystal
C. S AURES
D. Gonorrhoea
Answer is: B
462. (lead pipe sign) they wrote it like this without pic ask about diagnosis?
A. UC
Answer is: A
464. Patient with hypothyroidism, you started thyroxine, she came after 2 weeks still
complaining of symptoms, TSH was still high, What you should do? NEW20
A. Increase the dose and check after 3 weeks
B. Increase the dose and check after 6 weeks
C. Decrease the dose and check after 3 weeks
D. Decrease the dose and check after 6 weeks
Answer is: B
73
466. Football player received a trauma to lateral side of his left knee, the patient now is
complaining of severe pain and swelling of the medial side of his left knee, positive valgus
and (-) anterior drawer and lachman, most likely diagnosis?
A. Medial meniscus tear
B. Lateral meniscus tear
C. Medial collateral ligament sprain
D. Lateral collateral ligament sprain
Answer is: B
467. patient came after a car accident and the tibia was moving forward to femur what’s the
likely injury or rupture?
A. Lateral collateral ligament
B. Medial collateral ligament
C. Anterior Cruciate ligament
D. posterior Cruciate ligament
Answer is: C
468. Patient came with vomiting and nausea and upper quadrant pain Lab show elevated alt and
ast. What the indicate poor prognosis?
A. Alt
B. Prothrombin time
Answer is: B
469. Narrow complex tachycardia and HR 250-300 > abnormal rhythm What’s the diagnosis?
A. SVT
B. Afib
C. Atrial flutter
D. V tach
Answer is: A
74
472. adult women with abdominal pain and anxiety for 9 months, pain is associated with loose
stools, didn’t loose weight, all labs are normal. fecal calprotectin negative what is the most
appropriate treatment? NEW20
A. metronidazole
B. increase fiber intake
C. Tricyclic antidepressant
D. Can’t remember
Answer is: C
473. A 19 years old girl with IBS on she asked u more medication because her current one is not
effective, what to do?
A. TCA
B. Relaxation therapy
C. Other choices
Answer is: A
474. IBS case taking loperamide and another medication her symptoms not improved what is
most appropriate management?
A. cognitive behavioural therapy
B. Psychotherapy
C. TCA
Answer is: C
476. Patient with duodenal ulcer had an endoscopic clipping of the ulcer he is doing well, what
to give him?
A. Iv PPI for 24 hours then oral PPI
B. Iv PPI for 72 hours then oral PPI
C. High dose oral PPI
Answer is: B
477. Patient has penicillin allergy k/c of diabetes, came back from Egypt recently Complaining of
Red and render shin lesion for 3 days, was prescribed Abx and (1 DAY later) he had severe
diarrhoea, Diagnosis?
A. Salmonella
B. Allergy to Abx
C. Diabetic gastroparesis
D. Difficele
Answer is: D
75
478. 69 yo man known to have DM, present to ER with 4 days of production cough and
tachycardia, on examination, oriented to TPP, chest examination revealed right lower lobe
crepitation. RR 23/min Tmp 38.5, WBC 12, Urea 5, Chest x ray: right lower lobe infiltrate,
which of the following most appropriate management?
A. admit to general word and start iv abx
B. admit to icu amf iv abx
C. start oral abx and observe in ER for 24h
D. start oral abx and seen in outpatient in7 days
Answer is: D
479. Young Patient presented with migratory polyarthritis and subcutaneous nodules. What will
confirm diagnosis?
A. High ESR
B. High CRP
C. blood culture
D. ASO titre
Answer is: D
480. 28 years old male presented with attacks of bloody stool for 2 months. Colonoscopy shows
diffuse polyps in rectum and colon. Multiple biopsies were taken, what is the dx?
A. UC
B. Familial adenomatosis polyposis
C. HSV adenomas
Answer is: B
481. Healthy man come with lung nodule 7mm in the CT What next?
A. Ct after 3 th 6 month
B. PET scan
C. PFT
D. Review his last chest x ray
Answer is: D
482. Male with swelling & sinuses in axillary area what origin of these?
A. Apocrine gland
B. Eccrine gland
C. Follicular gland
D. Sebaceous gland
Answer is: A
483. female in her 20s with bilateral breast discharge I think milky where will you do the
imaging?
A. Pelvis
B. Sella turcica
Answer is: B
76
484. which one is a minor Jones criterion?
A. Erythema marginatum
B. Arthritis
C. Subcutaneous nodules
D. Elevated ESR
Answer is: D
485. Pt with Unstable angina ،ECG show depression ST, giving aspirin, Bb, atorvastatin, nitrate
Next appropriate mx?
A. Clopidogrel
B. candesartan
C. Streptokinase
Answer is: A
486. Old lady with 3ys knee pine improvement with rest + analgisa with pain in medial side +
varus leg on examination Dx?
A. osteoarthritis
B. Medial collateral ligament tear
C. Inflammatory arthritis
Answer is: A
487. Motion sickness what do you want to give to prevent the vomiting?
A. ondansetron
B. diphenhydramine
Answer is: B
77
490. When to consider MS sever?
A. <3
B. <2
C. <1
Answer is: C
491. Diabetic patient in metformin, after surgery he’s hospitalized now what to give him?
A. continue metformin
B. Basal bolus insulin
Answer is: B
493. Male 26 hx of spinal stenosis takes NSAID relieves pain, pain improves throughout the day
worse 1st 30 min of awaking?
A. Physiotherapy
B. spinal Steroid
C. Laminectomy
D. If remembered
Answer is: A
494. Women with severe headache and vomiting right eye vision loss history of two years
Amenorrhea and mild headache upon examination everything was normal, sensory, motor,
reflexes, CT: acute sella hemorrhage, MRI: mass compressing optic chiasm What to do in ER?
NEW20
A. IV hydrocortisone
B. pituitary lab
C. urgent trans-sphenoidal drain
Answer is: A
495. 57 pt with diabetes on insulin and statin what prevent cardiovascular disease?
A. Aspirin
B. Warfarin
C. Beta-blocker
Answer is: A
78
496. Pt having progressive arm and leg weakness, absent tendon reflex I don’t remember the
rest but this is the most important information asking about dx?
A. GBS
B. Gravies
C. Transevear myositis
D. Polymyositis
Answer is: A
497. MG , Painless weakness that is more proximal, with/out bulbar symptoms e.g., dysphagia,
nasal speech. Etc?
Answer is: incomplete Q.
498. GBS, Painless weakness that is more distal than proximal, pt may have hx of infection or
vaccine received 2weeks back)? GBS
Answer is: incomplete Q.
499. Patient complain of protrusions the jaw he seemed to bother by this issue and his friends
don’t see the the problem, the doctor examined he patient and everything was normal and
the patient mentioned surgical hx of rhinoplasty and blepharoplasty, what is the dx?
A. dysthymia
B. somatization
C. body dysmorphic disorder
D. chondriasis
Answer is: C
500. Male presents with peptic ulcer resistant to medications, with positive secretin stimulation
test, what is the diagnosis?
A. VIPoma
B. Glucagonoma
C. Gastrinoma
Answer is: C
501. Child his brother died suddenly when he was walking to work, wt imp disease to roll out?
A. HOCM
Answer is: A
502. Pts had seizure 3 days ago, His labs all normal expect = Sodium is low 121, Potassium
normal, Serum osmolarity low Urine osmolarity normal, Dx?
A. SIADH
B. Cushing disease
C. Conns disease
D. Addison disease
Answer is: A
79
503. Patient with family hx of type 2 DM came with polyurea labs shows FBS of 7.0, HbA1c of
7.1. Dx? NEW20
A. prediabetes
B. DM type 2
C. Impaired glucose intolerance
D. If remembered
Answer is: B
504. Pregnant lady came with facial rash, oral ulcer and painful hand joints, on examination
there was a tender car hand joints, which of the following is the most likely diagnosis?
A. rheumatoid arthritis
B. osteoarthritis
C. Carpal tunnel syndrome
D. SLE
Answer is: D
505. Employment screening, found to have dilated cardiomyopathy. No other symptoms and all
investigations are normal. EF is 40%. What to do?
A. ACEI
B. Do echo again after 6 months
Answer is: A
506. Weird new question about old lady can't walk has all PE normal all pulses and nerves were
intact; she can walk only 50 m and sometime linger if she was walking on hill or leaning fwd
and supporting herself with the walker what's is her dx?
A. peripheral artery disease
B. lumbar stenosis
Answer is: B
507. Pt with MI admitted to ICU due to urosepsis and septic shock not respond to fluid challenge
start him with low pulmonary artery pressure on inotrope hypoxic despite oxygen on x-ray
increase lung marking cardiomegaly?
A. Fluid overload
B. Cardiogenic edema
C. Acute respiratory distress syndrome
Answer is: C
80
508. Female with symptoms then diagnosed with Addison's disease and started on
hydrocortisone and fludrocortisone, at 9:00 12:00 15:00, Then she informed the doctor that
she has night shift What is appropriate?
A. No change in the treatment plan
B. Emit using cortisone during her night shift
C. Take the first dose at 9:00 then the rest at her night shift
D. Take her first dose at the beginning of night shift then 3 hours later then 6 hours later
Answer is: D
509. Solider complain of flat foot pain in the medial aspect of the sole, what is affected?
A. flexor retinaculum
B. Extensor retinaculum
C. Achilles tendon
D. Spring ligament
Answer is: D
510. patient developed tender well demarcated tender erythematous streaks on the right leg.
What is the most likely organism?
A. Strep pyogens
B. Klebsella pneumonae
C. Enterococcus
Answer is: A
511. Male patient said that the alien takes to him and put their thought into his mind. What is
the most appropriate management?
A. Anti-depressant
B. Anti-psychotic
Answer is: B
512. A 28 years old man with delusion and visual hallucination. When he asked that if he has any
problem, he said no., Which of the following is the most likely diagnosis?
A. Neurosis
B. Psychosis
Answer is: B
513. young female presented to ER with tachypnoea and tachycardia her father mentioned that
she recently failed in her math exam Dx?
A. can't remember
B. hyperventilating syndrome
C. munchausen syndrome
D. anxiety disorder
Answer is: D
81
514. How to confirm from imaging that the mass is benign or malignant?
A. hypoechoic
B. Bilateral
C. Size
Answer is: A, N.B: case of ovarian cyst.
515. Old patient presented with symptoms of 2 years memory loss and behaviour changes. Once
a caring parent, not he is agitated and aggressive. He is independent with his daily needs.
(Early Alzheimer) Which of the following is the most appropriate next step?
A. Cost effective laboratory test
B. Refer to geriatric
C. Admit into nursing facility
Answer is: A
517. Pt with bronchogenic carcinoma, presented with progressive SOB, there’s elevated jvp,
clear lung and quiet heart sounds. What will confirm your dx?
A. CXR
B. ECHO
C. ECG
Answer is: B
518. Pt has status epilepticus for 5 min, with iv access, what is the first line?
A. IV lorazepam
B. Rectal diazepam
C. buccal midazolam
D. Oral lorazepam
Answer is: A
519. Old pt after major rectal surgery he is not doing well in the recovery he start to have leg
pain he developed DVT from the popliteal to the femoral?
A. Enoxaparin
B. Heparin
C. Warfarin
D. IVC
Answer is: A
82
520. Old age with sarcoma in right thigh how to staging?
A. CT for bone
B. CT for chest
C. MRI
Answer is: B
521. Patient with symptoms of hypothyroidism Labs (TSH high T4 high)- (Thyroid peroxidase
antibody high), She also complains of galactorrhoea What investigation you will order to
reach diagnosis?
A. thyroid US
B. MRI pituitary Hashimoto thyroditis
C. FNA
Answer is: B, bad recall: symptom of hypo and lab is hyper, most likely case of
secondary hyperthyroidism.
523. Y.O female diagnosed with hyperprolactinemia for one year on cabergoline. Which of the
following is an indication for brain MRI in this patient?
A. Blurry vision.
B. Bilateral milky breast discharge.
C. Doubling of serum prolactin level.
D. Amenorrhea.
Answer is: A
524. child 15 ys have a disease he now thinking he will die what is this disorder?
A. panic attack
Answer is: A
525. Pt had massive haemoptysis caused by TB bronchiectasis and was treated, X-ray provided:
there is collection in upper right lung, your immediate management?
A. Put him on Right lateral position
B. Chest physiotherapy
Answer is: A
83
526. Y.O. Pt was diagnosed with GERD and was started on omeprazole 20mg. not responding
despite mild improvement he still complains of nausea. no dysphasia His P/E is insignificant.
what is your next step?
A. Add ranitidine at bedtime.
B. Refer him for upper Gl endoscopy.
C. Increase omeprazole to 40mg.
D. Follow up after 2 weeks.
Answer is: C
527. 65 Y.O. male patient k/c of liver cirrhosis, presented with upper GI bleeding. after initial ER
management with IV fluid, which of the following should be given before endoscopy?
A. Propranolol.
B. Octreotide.
C. VIT K.
D. Vasopressin.
Answer is: B
528. 59 Y.O. female Patient diagnosed with MI and was treated at the hospital, upon her
discharge she asked you what is the best way prevent reinfarction?
A. Use of aspirin for short duration.
B. Use of Calcium channel blocker indefinitely.
C. Use of ACEI if she develops heart failure to prevent cardiac changes.
D. Use of spironolactone for long duration.
Answer is: A
529. 26 Y.O. male patient presented to the hospital with syncope during a football game. His
father had a sudden cardiac death at 37. O/E he has mid-systolic murmur heard best at the
left lower sternal border. ECG is normal. What is the most likely diagnosis?
A. HOCM
Answer is: A
530. An 8-year-old boy’s parents complain that he has episodes where he blinks multiple times
and becomes okay after that he is conscious and responsive during those episodes. The most
likely diagnosis?
A. Tics
B. Tourette syndrome.
C. Blinking disorder
Answer is: A
84
531. 18 Y.O patient presented with vomiting 6 hours after eating steak from a local restaurant.
What is the most likely cause?
A. Staph. aureus. 6h
B. salmonella. 12-48h
C. coli 1h
D. shigella 48-72
Answer is: A
532. 55 Y.O. Pt on chemotherapy complained from fever (39C). Neutrophils are low. what is the
best next step?
A. Bone marrow aspiration.
B. Blood culture and IV antibiotics.
C. Culture from sputum, urine, blood and IV antibiotics.
D. Culture from sputum, urine, blood and oral antibiotics
Answer is: C
533. 66 Y.O patient diagnosed with MRSA and started on vancomycin. Immediately he developed
rash and itching in his trunk. What is the best next step?
A. continue but slow infusion rate.
B. Stop the infusion
C. Change vancomycin to cephazolin.
D. continue but Decrease the dose.
Answer is: A
534. SLE smoker with flared disease, what thing you ask the patient to do to decrease flaring?
A. Stop smoking
B. Wight loss
Answer is: A, if there avoid sun exposure is more accurate.
535. 66-year-old male Patient has a history of polyarthralgia with pain in distal and proximal
interphalangeal joint, no History of morning stiffness, medically free and no active arthritis,
what is the management?
A. Methotrexate
B. Finger splint
C. cyclophosphamide
D. weight loss
Answer is: B
85
536. Man, with history of reactive arthritis has another attack with joint aspiration showed
40000 WBC, neutrophils predominance. What is the initial management?
A. Ibuprofen
B. Methotrexate
C. Infliximab
D. Amoxicillin
Answer is: A, if acute attack > A, chronic > B.
538. Patient with decreasing GFR and may need dialysis next year, what is the most common
cause of death?
A. Renal failure
B. Cardiovascular disease
C. Respiratory infection
Answer is: B
539. Old patient develops productive cough, Fever, Dyspnoea on examination crepitation CXR
shows bilateral right middle lobe consolidation They mention on lab Reticulocyte Cold
agglutination test What is the Dx?
A. BA
B. Atypical pneumonia
C. Streptococcus pneumonia
D. Exacerbation of HF
Answer is: B
540. Patient on parenteral feeding developed weakness and convulsions what cause these
symptoms?
A. hypokalaemia
B. hypomagnesemia
C. hyperkalaemia
D. hypermagnesemia
Answer is: B
541. Patient have anxiety for 4 years and have asthma she noticed when she uses salbutamol
symptoms get worse, what to give?
A. Beta blocker
B. Bupropion
C. Sertraline
Answer is: C, if there was CBT is more accurate.
86
542. COPD acute exacerbation with a mouth full of secretions, what to do?
A. Invasive Mechanical ventilation
B. Bipap
C. Cipap
Answer is: A
543. Patient is heavy smoker, Facial plethora and JVP distended, which of the following is
associated with this condition?
A. Small cell lung cancer
B. Adenocarcinoma
C. Squamous
Answer is: A
544. A 23-year-old male known case of asthma presents with history of cough and night sweats
for 3 months. Vital signs are normal. CBC and metabolic panel are normal. Chest x-ray shows
right upper lobe cavitation which is not presented in last chest x-ray 6 months ago. What is
the next step in management?
A. Bronchodilator
B. Isolate patient in negative pressure room
C. Isolate the patient in positive pressure room
D. Antibiotic
Answer is: B
546. Patient with septic arthritis on empiric treatment colistin. His culture result showed
staphylococcal Aureus resistant to cloxacillin what antibiotic to give?
A. continue same
B. vancomycin
C. clindamycin
Answer is B
547. patient started on chemotherapy with high uric acid (Tumour lysis syndrome) what will you
expect to see?
A. hypocalcaemia
B. hypercalcemia
C. hypokalaemia
D. hyperkalaemia
Answer is: A
87
548. DM with UTI symptom and inv show (high) creatinine, which drug contraindicated?
A. Nitrofurantoin
B. Ciprofloxacin
Answer is: A
550. patient on TB drug and has elevated uric acid level (they say it like that and what is the
cause?
A. Pyrazinamide
B. Isoniazid
C. Rifampin
Answer is: A
551. Case of TB, what investigation should order before start treatment?
A. LFT
Answer is: a
552. Known case of GERD on PPI, symptoms only mild improvement, endoscopy done: normal.
what's the most important next step?
A. repeat Endoscopy in 6m.
B. Nissen fundoplication.
C. esophageal manometry.
D. ambulatory PH monitoring
Answer is: D
553. Patients complain of epigastric pain increase with vomiting and lab finding low b12, normal
Hb and folate, what investigation to do? NEW20
A. gastroscopy and antroscopy
B. colonoscopy
C. anti-intrinsic factor
Answer is: C
554. ECG with tall T wave and potassium level 6.5 immediate action?
A. calcium gluconate
B. insulin C-dialysis
C. sodium bicarbonate
Answer is: A
88
555. Obese women RUQ pain jaundice puritus know SLE on indirect high, AST: 700 AST: 700 AlP:
high Bright echogenicity Liver what is management?
A. Prednisolone
B. Methotrexate
C. Vit E and reduce weight
D. urodexolic
Answer is: A
556. Boy with glomerulonephritis (haematuria), after week he developed haemoptysis what is
the dx?
A. Henoch Schoenlein Purpura.
B. Good pasture syndrome
Answer is: B
557. old with DVT which of the following indicate thrombophilia test?
A. age.
B. Hx of OCP use.
C. Connective tissue disease
Answer is: C
558. Patient know case of idiopathic bronchiectasis, presented c/o progressive SOB over the past
2 month and dry cough on physical examination decreased airway entry bilaterally with
coarse crepitation" give?
A. Prednisone
B. antibiotics
C. nebulized hypertonic saline
Answer is: B, N.B: no physiotherapy in choices.
89
562. Case of a cancerous pt did multiple pleural tap due to pleural effusion with no
improvement, next?
A. Chemical pleurodesis
Answer is: A
563. Patient known case of polymyalgia rheumatica and other disease, presents with Fatigue,
CBC shows high lymphocyte count, most likely diagnosis?
A. CLL
B. lymphoma
Answer is: A
567. Known drug abuser found in deep coma and cyanosed, vitally unstable. Initial
management?
A. Hx from the family
B. Mechanical ventilation
C. CT brain
Answer is: B
568. 55 Y.O male patient presented with epigastric abdominal pain, constipation and behavioural
irritability. His wrist x ray showed bone erosions. His labs show: elevated PTH, elevated Ca.
what is the most appropriate management?
A. Hydration, diuretics, pamidronate
B. Hydration, diuretics, statin
Answer is: A
90
569. Patient with jaundice and Fleisher keiser ring, they gave laps and ceruloplasmin is low. what
is the treatment?
A. Penicillamine
Answer is: A
wilson disease
570. A Young male Presents with Altered sensorium, Fever, Nuchal rigidity And CSF analysis
shows: high protein, high cells, normal glucose?
A. Tuberculosis meningitis
B. Viral meningoencephalitis
Answer is: B
571. When you order the test for screening cancer you will chose?
A. Highly specific
B. Highly sensitive
C. Quick to perform
D. Cheap
Answer is: B
572. Case of drug abuser with painless rash in the palm and has retinal and splinter haemorrhage
and splenomegaly?
A. Bacterial endocarditis
B. Syphilis
C. HIV
Answer is A
573. Patient presented with diarrheal for two weeks. Stool showed trophozoites What is the
treatment?
A. Metronidazole
Answer is A
574. Patient RTA with head trauma, increased urine output, decrease in Urine osmolarity
increased blood osmolarity?
A. Central diabetes insipidus
B. Cushing
C. Conn’s
D. Nephrogenic diabetes insipidus
Answer is: A
91
575. A 36-year-old woman presents to the Emergency Room with a 1- week history of fever,
dysuria, and flank pain. She is admitted to the hospital after fluid resuscitation (a total of 6
litres of isotonic saline) and started on IV antibiotics. She also requires a low dose of
norepinephrine to maintain her blood pressure. The day after admission, she complains of
shortness of breath and requires 5 litres of supplemental oxygen; oxygen saturation is 91%
after this management. Chest X-ray confirms new, bilateral alveolar infiltrates. Bedside
echocardiogram does not show evidence of left ventricular dysfunction (see lab results).
Blood pressure 78/34 mmHg, Heart rate 125 /min Respiratory rate 25/min Temperature 38.6
oC, Oxygen saturation 95%, Which of the following is the most likely diagnosis? NEW20
A. Diffuse alveolar haemorrhage
B. Cardiogenic pulmonary edema
C. Healthcare-associated pneumonia
D. Acute respiratory distress syndrome
Answer is: D
578. 40 ys man came with his brother complaining of changing his habits and behaviour and
personality and forget most of events, even he may forget what he ate for breakfast and
become irritable, even he may leave you while you answering his question, the condition is
badly progressive; What's the diagnosis?
A. Vascular dementia
B. Frontotemporal dementia
C. Lewy bodies
D. Huntington's disease
Answer is: B
579. 77 males have HTN on 4 med on US renal showed asymmetrical kidney size?
A. renal arteries stenosis
B. Polycystic kidney
Answer is: A
580. Precaution of TB
A. Airborne
B. Droplet
C. contact
Answer is: A
92
581. Old patient, parkinsonism + visual hallucinations. Diagnosis?
A. Lewy Body Dementia
Answer is: A
582. pt with HTN and progressive memory lost and decrease cognitive function, MRI shows
hyperintense lesion?
A. Alzheimer disease
B. vascular dementia
C. Lewy body dementia
Answer is: B
583. Nurse work in pulmonary clinic C/O with fever, productive cough. She is not responding to
AB (Xray show opacity left upper lobe)?
A. Streptococcus pneumoniae
B. Hemophilus pneumonia
C. Mycobacterium
Answer is: C
584. male with epigastric fulness. On Examination, there is pulsatile abdominal mass. What
investigation you would order?
A. bilirubin
B. amylase
Answer is: B
585. Young male Pt with DM has UTI + MRSA. Which medication you will give?
A. Cefutrium
B. Vancomycin
C. Gentamicin
D. Floxacillin
Answer is: B
586. screening of colon Ca in average risk group at which age? In other recall Screening of low-
risk male colon cancer at which age?
A. 40
B. 45
C. 50
D. 55
Answer is: C
93
588. Most nephrotoxic anti TB?
A. Rifampicin
Answer is: A
590. 66 years old patient come with progressive difficulty breath. In history he is being treated
for bronchogenic carcinoma. In P/E: JVP elevated, lung clear and heart sound very quiet.
What’s the confirmatory investigation? Other recall Pt with bronchogenic carcinoma,
presented with progressive SOB, there’s elevated JVP, clear lung and quiet heart sounds.
What will confirm your dx?
A. CXR
B. ECO
C. ECG
D. ABG
Answer is: B
592. Female ingested 20 tablets of acetaminophen then came with N/V and right UQP what to
do?
A. N-acetylcysteine
B. no need for antidote
Answer is: A
593. Old pt with hx of recent travel, came with difficulty getting aroused/awaken, he reports
multiple falls, examination shows no head wounds and skull intact, most likely diagnosis?
A. post-concussion syndrome
B. chronic subdural hematoma
Answer is: B
94
595. women came to infertility clinic, her period every 45 days and now period in past 6 months
lab show high TSH and high prolactin… what the cause?
A. Hypothyroidism
B. Hyperthyroidism
Answer is: A, according to level T4.
598. most common site of the chronic inflammatory in Chron’s disease process is the?
A. ileocecal region
Answer is: A Terminal ileum
603. 57-year-old man has just gone PCI without stent placement for evaluation of unstable
angina. He has been placed on nitroglycerin, BB, and aspirin, but still feels chest pain with
activity. Lipid profile was provided (and all were on the upper limit of normal, but not
elevated). What would you add to this man’s medications?
A. Atorvastatin
Answer is: A
95
604. case of STEMI and center doesn’t have PCI what will give this pt?
A. ASA streptokinase heparin BB
B. ASA streptokinase nitroglycerin BB
C. other choices I exclude BV was no streptokinase
Answer is: A
605. pt presented with sweating dyspnea, palpation and headache. She is a known case of HTN
and despite taking medication it’s not controlled. imaging reveals supra renal mass. Which of
the following meds u will give to control her HTN?
A. CCB
B. Alpha blocker
C. ACEI
D. BB
Answer is: B
606. PTT was very low (in 20s) and macrocytic anemia. All other labs were normal. Your dx?
A. DIC
B. TTP
C. ITP
D. HIT
Answer is: incomplete Q, you can't Dx B without neurological manifestation.
607. Hereditary hemochromatosis asymptomatic all liver enzymes normal and ferritin around
850 asked about Tx?
A. penicillamine
B. initiate therapeutic phlebotomy
C. follow up after 6 wk or months forget.
Answer is: B
609. pt complete 2 weeks of H Pylori treatment today and come to ask you for test eradication
when to do it?
A. same day of the visit
B. after 2 other weeks
C. after 3 weeks
D. after 4 weeks
Answer is: D
96
610. What kind of medication used safe with kidney disease?
A. Linagliptin
Answer is: A
611. female patient complains of medical knee pain that increase with activity, tenderness on
medial knee? NEW20
A. Tendonitis
B. OA pain
Answer is: B
612. Mean Arterial Pressure is 55, intracranial pressure 15. What's the cerebral perfusion
pressure?
A. 40
Answer is: A
613. Case of chronic diarrhea blood culture show c. diff what is the antibiotics?
A. iv metro
B. oral vancomycin
C. oral penicillin
Answer is: B
615. 65 y/o female with DVT which of the following factor indicate thrombophilia assessment?
A. Age
B. Hx of OCP
C. Connective tissue disease
D. -ve FHx
Answer is: C
616. Patient with diarrhea, abdominal bloating and weight loss, symptoms increases with gluten
diet best nest step?
A. Gastroenterology referral
B. Stop gluten diet
C. Resume gluten diet, and order celiac blood test
Answer is: C
97
617. scenario of elderly patient who had chest inf 4 weeks ago, initially improved on antBx, now
worsened again, CXR: right pleural effusion. Dx?
A. Lung cancer
B. TB
C. Parapneumonic effusion
a. Answer is: C
618. A scenario of STEMI with ECG shown, lead II, III, and AVF ST elevation, Next step?
A. Echocardiogram
B. Chest X-ray
C. CT of the abdomen
D. ECG lead on the right side
Answer is: D
620. Case of inferior MI present with clear lung sound, JVP, hypotension not mentioned muffled
sound and no murmur?
A. Papillary muscle rupture
B. Right ventricular infarction
C. Cardiac tamponade
Answer is: B
621. ICU patient developed abnormal thyroid function test (low TSH, low T4, low T3). What’s the
diagnosis? And what lab highly indicates your diagnosis?
→ Dx > Sick Euthyroid Syndrome (SES)
→ Lab> Reverse T3 will be high
622. long scenario of a patient with Afib, and high CHADS score now developed MI, what should
you do?
A. Didcon. Warfarin
B. cont. Warfarin
C. cont. Warfarin and add clopidogrel
D. cont. Warfarin and add 2 antiplatelet
Answer is: D
98
623. when to consider a nephrotic syndrome is a cortisone resistant?
A. 4 months of daily cortisone
B. 12 days of daily cortisone
C. 4 weeks of daily cortisone
Answer is: C
624. patient lost sensation over medial side of leg but motor function is intact. Which nerve is
injured?
A. obturator
B. femoral
C. saphenous
D. Sciatic
Answer: C
626. case about meningitis with pneumonia (lymphocytes high, neutrophils high) what type?
A. bacterial
B. viral meningitis
C. TB meningitis
D. after something phago meningitis
Answer is: A, according to glucose level.
627. DM, HTN, claudication after walk 200m, cold leg and pain there is pulse in femoral but no
distal pulse. ECG show Afib What is the most appropriate (next step) management?
A. Heparin
B. CT angio
C. Duplex US
Answer: A
628. patient 64 y.o had pervious stroke, DM, HTN, basal lung crebetion ,S3 and lower limb
edema calculate the chadsvascs score? NEW20
A. 1
B. 2
C. 4
D. 5
Answer is: D
99
629. Patient 56 y.o came for routine follow up known case of CHF asymptotic not on medication
his EF 25% what you want to give him? NEW20
A. Beta Blocker
B. ACEI
C. Spironolactone
D. Diuretics
Answer is: B
630. When there will be significant decrease in the risk of MI after smoking cessation? In years
A. 1
B. 2
C. 3
D. 4
Answer: A
631. Case of patients vague complain, labs all normal except potassium 6, what you will do?
A. Hemodialysis
B. IV normal saline
Answer is: incomplete Q.
632. Pt with MRSA sepsis given vanco 1000 mg q72h, over 20 min flushing and itching?
A. Stop considering allargic
B. Decrease the infusion rate
C. Decrease the dose
D. Change to cefazolin 1000 q24.
Answer is: B
633. 25 yrs old male referred from orthopedic clinic with fragility fracture, he has sparse axillary
and face hair. What inv. You will do?
A. Calcitonin
B. testosterone and gonadotropin
C. bone densitometry
D. TSH B
Answer is: B
634. Where 12 Years old must sit in car to avoid injury in accidents?
A. in the front unrestrained
B. in the back restrained
C. in child chair in the back
Answer is: B
100
635. Case that seemed like obstructive sleep apnea. Obese patient with uncomfortable sleep +
snoring. Has cyanosis and dyspnea. what is the best investigation?
A. spiral CT chest
B. methacholine challenge test
C. spirometry
D. polysomnography
Answer is: D
636. patient had MI 2 wk ago and PCI were done and discharged, came back with pale, cold limb
swelling Dx?
A. DVT
B. acute ischemic emboli
C. chronic ischemic thromboembolism
Answer: B
637. sharp, shooting pain of neck radiated to shoulder and jaw, what is the diagnosis?
A. cervical disk prolapses
B. multiple sclerosis
C. rheumatic myalgia
Answer is: A
638. Patient 72 years old have DM, for 3-month can't stand from the chair What is happening to
him?
A. Fall
B. RTA
C. gunshot
D. Fire
Answer is: A
639. patient with DM, smoker, with peripheral artery disease, what would you do to improve his
condition?
A. dual aspirin
B. smoking cessation
C. exercise program
D. strict glucose control
Answer is: B
640. fever and cough and vitally stable, HR 110 No hypotension increase in RR, But the WBC
might have been 11,000 what is the diagnosis?
A. Bacteremia
B. SIRS
C. Sepsis
Answer is: C, if the case post-surgery > B.
101
641. Long case with ECG pic but in hx they mentioned yellow vision so it's?
A. digoxin toxicity
Answer is: A
642. Patient suspected to have TB pleural effusion whats is the most sensitive test that supports
ur Dx?
A. AFB of pleural effusion
B. High Lymphocytes in pleural effusion
C. Low glucose pleural effusion
D. High pleural fluid ADA levels
Answer is: D
643. A 30s-year-old came with high PTH, and High Ca+ and low phosphate?
A. PTH adenoma
B. PTH carcinoma
C. 2ry hyperPTH
D. Tertiary HyperPTH
Answer is: A
644. 25 males come with Epigastric pain from 2 Hours relieved by passing a bowel movement
and Mucus in the stool. before 2 weeks come with same Hx, most likely diagnosis?
A. GERD
B. H. pylori
C. celiac
D. Anxiety
Answer is: D
645. A case of adult who was showering and he incidentally found a hyperpigmentation in his
back, Labs? HypoN+ , Hyperkalemia What is most likely Dx?
A. Addison disease
Answer is: A
646. Pt has status epilepticus for 5 min, with iv access, what is the first line?
A. IV lorazepam
B. Rectal diazepam
C. buccal midazolam
D. Oral lorazepam
Answer is: A
102
647. 35 Y.O. Pt was diagnosed with GERD and was started on omeprazole 20mg. not responding
despite mild improvement he still complains of nausea. no dysphasia His P/E is insignificant.
what is your next step?
A. Add ranitidine at bedtime.
B. Refer him for upper Gl endoscopy
C. Increase omeprazole to 40mg.
D. Follow up after 2 weeks.
Answer is: C
648. male with impotence and difficulty to maintain an erection due to stress in his work and
life? NEW20
A. Sildanfil
B. Endocrine consultation
C. urology consultation
D. something about decreasing the stress
Answer is: D
649. 16-year-old girl with acute migraine headache and nausea and aura. What do you give for
acute management?
A. aspirin
B. triptans
C. Oxygen
Answer is: A
650. Adult with positive urea test was given PPI and metronidazole, which of the following drugs
u add to his treatment?
A. Clarithromycin
B. Tetracycline
Answer is: A
651. urine incontinence male has problems with initiate urine, and after voiding still have
palpable bladder? what type?
A. Overflow
B. Urge
C. Reflex
D. Stress
Answer is: A
103
652. 55 years old, male, have a family history of diabetes, HgA1C is 6.3 fasting 7.3 what will you
do next?
A. Random glucose
B. Fasting after 3 months
C. HgA1C after 6weeks
D. 2 hours glucose
Answer is: D
653. 65 yo male medically free came for general checkup. What to recommend regarding
pneumococcal vaccines?
A. Give (PCV13) then (PPSV23)
B. Give (PPSV23) then (PCV13)
C. Give one of them (forgot which one)
D. He doesn’t need the vaccine
Answer is: A
655. Pt with history of recurrent vomiting for 2 weeks what is the expected metabolic
abnormality?
A. Metabolic Acidosis
B. Metabolic Alkalosis
C. Compensated Metabolic acidosis
D. Compensated Metabolic Alkalosis
Answer is: according pH > B or D.
657. K/c of SLE on hydroxychloroquine came with seizure. What’s your management?
A. Cyclophosphamide
B. Methotrexate
C. Steroid
Answer is: A
104
658. child with mild respiratory infection increase symptoms & fever, develop respiratory
distress. Xray shows lobar infiltration, CBC shows low HB and high Retic ?
A. SCA
B. Mycoplasma pneumoni
C. pneumococcal pneumonia
Answer is: A
661. Patient is having very high blood pressure 220/180, not decreasing with BB or diuretic, what
is the Mx?
A. Increase BB
B. Add CCB
C. Alpha blocker
Answer is: C, if there ACE is more accurate.
663. pt with history of peptic ulcer and +ve urea breath test. You started and finished the
treatment course. When you can re-examine the urea breath test?
A. at least 4 weeks
B. at least 2 weeks
C. at least 1 week
D. examine now
Answer is: A
664. patient had knee pain, no trauma no fever and also have epigastric pain?
A. ibuprofen
B. paracetamol
C. Aspirin
Answer is: B
105
665. Adult patient came to ER with Substernal pain for 1 hour not radiating no other issue, with
previous attacks 2 weeks ago last for seconds only?
A. GERD
B. MI
C. Pericarditis
Answer is: A, if there anxiety is more accurate.
667. What symptom makes you suspect eosinophilic esophagitis the most?
A. prolonged chewing of food
Answer is: A
668. Burning chest pain for 6 months increased at night and unpleasant taste when lifting heavy
objects?
A. Esophagitis
B. Acute gastritis
C. Boerhaave syndrome
D. Perforated peptic ulcer
Answer is: A
670. Patient HTN has EF 55% and LVH. Cause of HF in this case?
A. Systolic dysfunction.
B. Diastolic dysfunctions.
C. HCM
Answer is: B
672. Old man fall down and has hx so they put for him FC due to immobility Given paracetamol
and Codeine for pain Then he develops fever and nurse noticed cloudy urine, Mx?
A. Review the need for Folly catheter
Answer is: A
106
673. Pt in ICU developed cloudy urine?
A. check daily need of a catheter
Answer is: A
674. patient with CHF and DM and HTN not tolerating ACEI what to switch it with?
A. ARBs (Lorestan)
Answer is: A
675. DM, heart failure and admitted for hysterectomy and received Normal Saline cause poor
oral intake, 4 d nurse noticed decreases in Sat and SOB, o/e: crackles, how to prevent this
complication?
A. cardiopulmonary consult
B. daily fluid assessment
Answer is: B
676. case with flat T wave what you will find in the pt the?
A. hypokalemia
Answer is: A
677. Female with 3 wks hs of productive cough with decrease both FEV and ratio, What other
finding?
A. Increase lung compliance
B. Decrease lung compliance
C. Increase air way resistant
Answer is: A
678. Patient k/c of COPD alert, has moderate respiratory distress.SO2 93%, ABG showed hypoxia,
hypercapnia and acidosis. What is next step?
A. noninvasive ventilation
B. Decrease oxygen
C. Increase oxygen
D. Mechanical ventilation
Answer is: A, according full scenario, if pH less than 7.25 > D.
679. scenario about a man with epilepsy poor controlled, have productive cough and fever,
Asked about what the source of infection?
A. Aspiration
B. TB
C. Infectious mononucleosis
D. Pneumonia
Answer is: A
107
680. COPD elderly admitted exerberation at night became aggressive disoriented what the,
nurse should do first?
A. Call the duty dr for lorazepam iv.
B. Restrain her
C. Call family sit beside her
D. elevate head of bed put nasal oxygen try to oriented her about place and time
Answer is: D
682. pt medically free go to tooth extraction and pt develop petechial rash and Hb high+,
erythropoietin low + Platelet high?
A. Myelofibrosis
B. essential thrombocytopenia
C. polycythemia vera
Answer is: C
684. soldier going to southern of Saudi Arabia came for malaria prophylaxis?
A. Proguanil / Atovaquone
Answer is: A
685. 18 Y has ALL. 17 days after chemo he develops fever around 38.7 C. No focus of infection
found. Labs shows low WBC 0.6 (normal was 4-10x..), Neutrophil level 60% (normal 40-60%),
I’m not sure if he has low platelets as well, what you’ll do?
A. Blood urine, ... culture and paracetamol
B. Blood, urine, ... culture and PO Abx
C. Blood, urine, ... culture and IV Abx
Answer is: C
108
687. Male pt asymptomatic came for checkup, drinks alcohol occasionally every weekend. LFT
showed mildly elevated enzymes, ferritin 490 high, TIBC high, dx?
A. alcoholic hepatitis
B. hemochromatosis
Answer is: B
688. facial plethora and JVP distended, which of the following is associated with this condition?
A. SLCC
Answer is: A
689. Pt with bilateral hydronephrosis with dilated bladder and proximal urethra?
A. Posterior urethral valve
B. Ureteropelvic occlusion
Answer is: A
690. Clear case with headache that is thromping and unilateral aggravating with light and
movement ?
A. migraines
Answer is: A
693. A traumatic pt lost 25% of his blood. Which of the following is the most suspected to be
effected first?
A. Urine output
B. Pulse pressure
C. Respiratory Rate.
D. Glasco-coma scale
Answer is: B
109
694. Women diabetic controlled developed dysuria and frequency urine analysis nitrate,
creatinine high. Which drug is contraindicated?
A. Amoxicillin
B. Trimethoprim / Sulfamethoxazole
C. Ciprofloxacin
D. Pipracilin tazopactam
Answer is: C
695. Male pt with hx of right knee pain and swelling, left ankle pain and swelling, for 6 months.
Previous hx of UTI 2 weeks prior to symptoms, what to give?
A. methotrexate
B. cyclosporin
C. Sulfasalazine
D. Azithromycin
Answer is: C
696. Which test is appropriate to study 5 different types of exercise given to obese ladies over a
period of time and follow their BMI?
A. cohort
B. cross sectional.
C. case control.
D. clinical trial
Answer is: A
697. Patient with a BMI of 40, she doesn’t c/o visual disturbance, no hypothyroid symptoms, she
came with lab works of: TSH high, prolactin normal, FSH and LH are abnormal. What is the
possible cause?
A. non-functioning prolactinoma
B. Hypothyroidism
C. Morbid obesity
Answer is: incomplete Q, in 2019 Q bank there was pituitary adenoma in CT > A.
699. Patient is Caucasian and symptoms started when he was introduced to regular food
Patient has no history of drug use or alcohol drinking, presented with vomiting, diarrhea
and jaundice. Which of the following indicates her infection?
A. HAV IgM
B. HAV IgG
Answer is: A
110
700. What is the optimal time for anticoagulant for initial PE episode?
A. 3 months provoked
B. 6 months unprovoked
C. 12 months
D. 6 weeks
Answer is: A
701. Patient had thenar muscle atrophy however his sensation was intact over that area,
what nerve injury is most likely?
A. Radial
B. Median
C. Axillary
D. Ulnar
Answer is: B
702. Hemochromatosis asymptotic, liver enzymes within normal only increased in ferritin level
750 What’s appropriate next step?
A. regular phlebotomy every month
B. follow up after 6 months
Answer is: A
703. Patient saying that he has a disease, and all the doctors he visited told him he didn’t have
anything and the tests are normal, but still, he is saying that he has, diagnosis?
A. Somatization
B. Something pain disorder
C. Hypochondriasis
Answer is: C
704. Type of shock in post op patient day 4, vitals were stable (NO FEVER) and no signs of allergic
reactions nor provided anything in the history and on examine his peripherals were warm,
what's the type of shock?
A. hypovolemic
B. cardiogenic
C. Septic
D. Anaphylactic
Answer is: C
111
706. Highest risk factor for diabetes 2?
A. Pregnancy
B. Abdominal obesity
C. High body index weight
Answer is: B
708. scenario about female that complain of non-pitting edema, what is the initial test you will
order?
A. Venous duplex
B. CTA
C. MRA
Answer is: A
709. Female that complain of non-pitting edema, what is the best investigation?
A. Lymphoscintigraphy
Answer is: A
710. Old pt diabetic and heavy smoker with typical chronic limb ischemia claudication after
100m. The left popliteal artery is not palpable, what is the appropriate management?
A. smoke cessation and strict glycemic control
B. Stent of the stenotic artery
Answer is: A, if there was risk factor stratification is more accurate.
711. Pt liver cirrhosis with tender ascites, which of the following drug is best to prevent
recurrence?
A. Aldosterone antagonist
B. Loop diuretic
C. Osmotic diuretic
Answer is: A
712. pt with multiple sclerosis, presented now with acute attack (diplopia.), mx?
A. iv steroid
B. oral steroid
C. interferon
D. IVIG
Answer is: A
112
713. Burn case young with both limp 2nd degree burn (wight 70 kg - age 17) and calculate
parkland formula then what to give for resuscitation?
Answer is: Parkland formula: 4 ml x BSA x weight (kg)
716. Vet with fatigue, hepatosplenomegaly, and thigh pain (other brucellosis symptoms) only
vitals are shown, what is the best next step?
A. Liver function tests
B. Xray of sacroiliac
C. Tuberculin skin test
D. serum agglutination test
Answer is: D, N.B: no blood cultures in the options
717. Pt with HTN DM his taking medication (labetalol, nefipidn and others) came with
bronchospasm what is the cause for his presentation?
A. Labetalol (BB)
Answer is: A
718. Typical case of hyperthyroidism and palpation What will you do for her palpation?
A. PTU
B. Methimazole
C. Propranolol
Answer is: C
719. Case of patient with past history of TB presented with chest pain on examination elevated
JVP, cardiac exam no murmur?
A. cor pulmonale
B. constrictive pericarditis
Answer is: B
113
720. Pt 35 yo newly diagnose with HTN his BP is 150/90 Lab shows high creatinine what is the
first line in management?
A. ACEI
B. diuretics
C. CCB
Answer is: A
722. COPD patient presenting with bilateral lower limb edema and pulmonary hypertension. PO2
8.6, PCO2 7.5, pH within normal range Which of the following is the appropriate
management?
A. start furosemide
B. Start oxygen therapy
C. Prednisolone therapy
Answer is: A
723. Male 39 years old presented with decreased lipido and dysfunctional erectile dysfunction.
Labs: hyperprolactinemia and normal LH/FSH what is the best investigation to do?
A. testosterone levels
B. MRI pituitary
C. CT pelvis and abdomen
Answer is: B
114
726. NF1 mode of inheritance?
A. AR
B. AD
C. X linked R
D. X liked D
Answer is: B
727. pt with SOB only no other symptoms they asked what’s the best diagnostic test lab results
showed respiratory alkalosis (PE ecg)?
A. echo
B. D dimer
C. spiral CT
D. ecg
Answer is: C
728. 40-year-old diabetic female presented with history of pruritis. On examination the vulva is
erythematous with white- gray odorless discharge. The vagina was cover with white-gray
discharge. discharge was examined under microscope and revealed spores: Which of the
following is the likely diagnosis?
A. Trichomonas vaginalis
B. Candia albicans
C. Pediculosis pubis
Answer is: B
729. 40-year-old lady presented with dribbling urine with dysuria and dyspareunia. No surgical
history. Which of the following is the most likely cause?
A. Stress incontinence
B. Urge incontinence
C. Urethral diverticulum
D. Over flow incontinence
Answer is: C
731. Case about crohn’s. Which of the following is the most expected finding?
A. Nail pitting
B. Perianal diseases
Answer is: B
115
732. Female complain of lethery skin pigmentation in her neck What is the name of the
condition?
A. acanthosis nigricans
Answer is: A
735. Young 20-30 Patient has sx of hyperthyroidism diarrhea and irregular and milky discharge
she is not pregnant lab both high T4 19? And TSH 9?
A. MRT of pituitary
B. Biopsy/us of thyroid
C. Us of breast
Answer is: A
736. 32 women smokers with polyarthralgia and malar rash fatigue has positive ANA she is on
hydroxychloroquine and mycophenolate. Lab 24urine high? Which of the following
nonpharmacological would help?
A. wt loss
B. Smoking cessation
C. Low protein diet
D. Reduce physical activity
Answer is: B
737. A 45-year-old patient present to the clinic with a three-month history of stumbling weak
grip dysphagia and. Generalized weakness and also had to Episode aspiration pneumonia has
brisk reflex and fasciculation?
A. Motor neurons disease
B. Myasthenia gravis
C. Myasthenia syndrome
Answer is: A
116
738. A fib patient with MI will undergo PCI what to do?
A. Cont warfarin and add two antiplatelet
Answer is: A
739. ECG picture of not clear but from the history chest pain just had recent infection two weeks
ago?
A. Pericarditis
Answer is: A
740. Patient with very high blood pressure above 220 with high catecholamines what to five?
A. Alpha blocker
Answer is: A
741. Patient with a fib and dilated cardiomyopathy what to give to control the rate now?
A. Digoxin
B. BB
C. diltiazem
Answer is: B
742. Male pt with fatigue and sexual dysfunction BMI 40 presented with low FSH, LH, TSH,
testosterone, prolactin wasn’t high, MRI showed 2.5 cm pituitary adenoma?
A. nonfunctioning adenoma
B. morbid obesity
C. hyperprolactinemia
D. primary hypothyroidism
Answer is: A
744. Pt on PPI not controlled endoscopy done and showed reflux esophagitis, next step?
A. PH monitoring,
B. Nissan fundoplication,
C. Manometry
Answer is: C
117
746. Pt with sx of hemochromatosis how to dx?
A. Liver bx showing iron load
Answer is: A
748. 26-year-old with sickle-cell and your present with paler, lethargy and headed, Hb was 44
and reticulocyte 2.8?
A. sequestration crisis
B. hemolytic crisis
C. aplastic crisis
D. thrombotic crisis
Answer is: B
749. Pt first with depression symptom later buying expensive car and very talkative?
A. Lithium
B. Mirtazapine
C. Olanzapine
Answer is: A
750. year old woman known case of DM did cholecystectomy 2 years ago, she has been using
NSAID to relieve RA symptoms She develop HTN, Na normal K below borderline Cr higher
than normal 150s what do think the cause of HTN?
A. Essential hypertension
B. pheochromocytoma
C. NSAID induced HTN
D. Primary hyperaldosteronism
Answer is: incomplete Q.
751. Most common organism in infective endocarditis (clinical presentation only with splinter
hemorrhage)?
A. Staph aureus
B. Streptococcus
Answer is: A, according to full scenario.
118
753. A case of pt with psoriasis used steroids presented with Cushing sx, low ACTH, low morning
8 am cortisol:
A. iatrogenic Cushing
B. Cushing syndrome
Answer is: A
754. 50 yrs man diabetic, well controlled had colon cancer surgery (coloectomy) they kept him
on insulin and dextrose, after surgery by 2 days he became irritable, in shock, his electrolyte
(Na 129) (K 3.2) urine and serum osmolality Normal, what's the Dx?
A. Fluid overload
Answer is: A
756. Hematochezia: NGT greenish, colonoscopy done was normal, Best next step to diagnosis?
A. Technetium 99
B. Gastroduodenal endoscope
Answer is: A, according to full scenario and stability and volume of bleeding.
757. An 8 years old presented with nausea vomiting diarrhea for 2 days. On examination: HR
110, RR23, capillary refilling 3 sec, and cold extremity. How to replace her fluid deficit?
A. over 48 h
B. Over 24 h
C. Over 12 h
D. Over 6h
Answer is: B
758. 60 yo male known case of hypertension, he came for toteine follow up. Examination
revealed Ejection systolic murus at right sternal border radiating to carotid. Loud S2. What is
the diagnosis?
A. Aortic stenosis
B. Pulmonic stenosis
C. HOCM
Answer is: A
119
759. Case of Obese patient with uncomfortable sleep + snoring. Has cyanosis and dyspnea. What
is best investigation for diagnosis?
A. Polysomnography
B. PFT
C. HRCT
Answer is: A
760. Case of a patient with cardiac disease on medications, has slightly elevated BNP, which of
the following may cause elevated BNP? NEW20
A. ACE
B. Furosemide
C. Obesity
D. COPD
Answer is: D
761. Which of the following stool electrolytes testing would represent vibrio cholera? NEW20
A. Stool osmosis of 300 mOsm/kg
B. Stool osmosis of 410 mosm/kg
C. Fecal osmotic gap of 30 mosm/kg
D. Fecal osmotic gap of 110 mosm/kg
Answer is: C
762. What is the incubation period of lung cancer in smokers (in years)?
A. 10
B. 20
C. 30
Answer is: B
763. Lady screened for HBV negative, her husband positive on treatment?
A. protected sex with condom
Answer is: A
120
765. 6 yrs male patient with crampy abdominal pain not related to specific food with diarrhea
alternating with constipation?
A. IBS
B. Crohns
C. lactose intolerance
Answer is: A
766. Two cases patient with pulmonary embolism and hemodynamic instability (BP less than
90/60)?
A. treatment with thrombolytics
Answer is: A
767. Rash, - ANA, fever, arthritis and arthralgia, hyperferritinemia What is Diagnosis?
A. Adult still disease
Answer is: A
768. Young female presented with dyspnea, low grade fever and arthralgia. On examination
there was tender erythematous nodules on her shin. And on auscultation basal crackles.
What to do next?
A. Chest X Ray
B. Steroid
C. Blood culture
D. skin biopsy
Answer is: A
770. Elderly comes with BPH he has urinary symptoms. Labs including renal and PSA are normal.
What is your treatment?
A. Alpha blocker
Answer is: A
121
772. pt with GERD on maximum PPi dose with minimal response and Upper GI endoscopy is
normal, what next?
A. 24 ph monitoring
B. Manometry
Answer is: A
773. Long case of patient with resp symptoms and after evaluation the doctor gave the patient
oseltamivir What is the precaution?
A. Contact
B. Standard Inflenza
C. Droplet
D. Airborne
Answer is: C
775. 15-year-old, his friend died in a car accident and he told his mom he's thinking of suicide
but not going to act on it, what to do?
A. Asking him directly about his suicidality is going to increase the risk of him acting on it
B. Ask him in details about his suicidal thoughts and feelings
Answer is: B
776. signs of heart failure like dyspnea and distress and cough and bilateral crackles and other
symptoms, asking about treatment?
A. IV furosemide
Answer is: A
777. CAH case and they asked about the definitive treatment. + low glucose?
A. Normal saline and hydrocortisone
B. Normal saline and hydrocortisone and glucose
Answer is: B
122
779. 57 Years old (no gender) presented with LLQ abdominal pain for a few days, associated with
nausea and constipation. What is the diagnosis?
A. Ovarian torsion
B. Diverticulitis
Answer is: B
780. Case of heroin or opioids withdrawal so what are you going to give?
A. Naloxone for overdose
B. Methadone
Answer is: B
781. pt with band like headache with throbbing sensation behind eye and he had a hx of
stressed school... what is most likely diagnosis?
A. Sinus headache
B. Tension headache
C. Vascular related headache
Answer is: B
783. A pt. had lower parathyroidectomy, came to the clinic c/o bone pain, numbness around the
lips. Lab results showed high Ca + high PTH. What’s the most likely dx?
A. Missed adenoma
B. New adenoma
C. Parathyroid hyperplasia
Answer is: A
785. Patient was present with hyperthyroidism symptoms (Weight loss, ect) and ther were small
gouiter in the neck, she missed her period lately, and develop milky discharge from
nipples.What will you do ?
A. Us thyroid
B. MRI pituitary
Answer is: B, secondary hyperthyrodism.
123
786. pt Iv drug abuser, what will you screen for?
A. HIV
B. MRI pituitary
Answer is: A
787. COPD pt on high flow o2 3 L with O2 saturation 93% , develop acidosis and hypercapnia,
what will you do ?
A. decrease O2 flow
B. mechanical ventilation
C. increase O2 flow
D. CPAP
Answer is: A
789. 29 years old patient came to clinic after having involuntary movements and change in
power and sensation He have history of multiple unprotected sex since 6 years There was
laps said he have cryptococcus infection What is most important investigation?
A. HIV serology
B. CSF for syphilis
C. Toxoplasmosis antigen
D. Blood culture
Answer is: A
790. patient with raised JVP, Hypotension, unclear heart sound, clear lung sounds. How to
confirm the Diagnosis ?
A. ECHO
B. ECG
C. Chest X-ray
D. PFT
Answer is: A
791. Pregnant woman in 31 gestational have new onset SOB and dyspnea and lower limb edema
on examination there is bilateral lung crackeles what is the appropriate investigation?
A. Chest x-ray
B. ECG
C. ECHO
D. chest CT
Answer is : C
124
792. 43-year-old male with hypertension on -sartan & amlodipine, his blood pressure is not
controlled. What to add? NEW20
A. Lisinopril
B. Atenolol
C. Indapamide
D. Zosin (alpha blocker)
Answer is: C
794. Patient presented to the ER with COPD exacerbation, severe resp distress cyanosis, lots of
secretion. ABG low O2 and acidosis. What's the appropriate management?
A. Intubate
B. Oxygen therapy by face mask
Answer is: A
795. Patient coming with COPD exacerbation, managed first with high flow oxygen. Then patient
became drowsy. ABG shows acidosis and high CO2. What's your mx?
A. Intubate and hyperventilate
B. Decrease flow of oxygen
Answer is: B, if Ph less than 7,25 > A.
796. Asthma pt in severe respiratory distress. ABG show low O2 and high PaCO2. Management?
A. IV mg sulfate
B. Intubate
Answer is: B
797. Patient came from egypt, she had sntibiotics there and developed redness in the leg. Now
shes on clindamycin and another abx. Then she had diarrhea and abd pain. Ask about dx?
A. Drug sllergy
B. C difficle infection
Answer is: B
125
799. 18-month-old boy has signs and symptoms of meningitis with +ve diplococci. What to give?
A. Vancomycin
B. Certiaxone
C. vancomycin and ceftriaxone
Answer is: C
800. Recurrent chest infection and weight loss, what is the dx?
A. cystic fibrosis
B. Alpha 1 antetrypsin
Answer is: A
801. case of highly productive cough ( i think Bronchiectasis), what is the diagnostic?
A. High resolution CT
Answer is: A
805. Pt came with all of signs of endocarditis sins and he also has mitral valve prolapsed and they
want to do for this patient surgery what proflatic antibiotics you give to this patient before
the surgery? NEW20
A. Vancomycin alone for endocarditis
B. Cefuroxime Alone for wound
C. Combine vancomycin + cefuroxime
D. don’t remember but it was also combine amoxicillin +....?
Answer is: Bad recall, in 2019 Q bank no sign of endocarditis, answer is > no need.
126
806. Pt had RTA, unconscious, GCS was 8, o/e JVP elevated, hyperresonant right chest,
decreased air entry. What is the most appropriate immediate action?
A. needle decompression
B. Intubation
Answer is: A
808. Pt with chronic limb ischemia, presented with sudden leg pain, diminished popliteal and
distal pulses in right leg, and diminished distal pulse with intact popliteal in the left, what's
the appropriate next action?
A. Heparin
B. CT angio
C. US
D. conventional angio
Answer is: A
811. Fracture tibia two fragments with open skin (hole) after RTA... Picture? Next...?
A. Debridement
B. Abx
C. Closed reduction
D. Open with im nail
Answer is: A
127
813. Fracture femuer...?what to check before reduction?
A. Pulse
Answer is: A
815. CASE of elderly done open hernia repair then he developed again hernia...? Ttt?
A. Open.
B. Lap
Answer is: B
816. Hx of travel to mecca for omrah and present with abdominal pain and watery diarrhea,
what is the most likely organism?
A. giardia
B. forget the others
Answer is: A
818. Old pt have pain in her legs and tingling (I guess) she relives when she bends and start
climbing the stairs but going down the stairs feels more pain what is the diagnosis?
A. Lumbar stenosis
B. Lumbar spondylitis
Answer is: A
819. Pt is diagnosed with Addison they give her medication (forgot the name of drug but focus
on hour of taking it) medication is taking at 9 am, 12 pm, 15 pm. She said before leaving that
I have night shift. When does she take the medication?
A. Same (9,12,15)
B. Keep 9 in morning Change only 12,15 And make it at night
C. Change medication to night shift
Answer is: C
128
820. long Scenario of a patient with afib and a high chad score now he developed MI what
should you do?
A. Cont warfarin
B. Cont warfarin and add 2 antiplatelet
C. Discont warfarin
D. Cont warfarin and add clopidogrel
Answer is: B
822. Pt complain from neurological symptoms support with picture of hyper segmented
neutrophile and megakaryocyte?
A. IDA
B. folic acid anima
C. vit B 12 anima
D. thalsima
Answer is: C
823. Female with SLE compline from UTI symptoms what is the contraindications drug?
A. Sulfa
Answer is: A
824. pt complain from confusion, fever and mild neck stiffness CT show low attention in Rt
temporal lope what is the Dx?
A. brain abcess
B. pneumococcal meningitis
C. herps smplelix encephalitis
D. listeria meningoencephalitis
Answer is: C
129
827. pt with weakness in lower limb after hx of gastroenteritis before 2 weeks what is the
prognosis? GBS?
A. Full recovery
Answer is: A
830. pregnancy on SABA complain of cough 2time /week and admitted two times?
A. LABA with ICS
Answer is: A
832. wife with her husband + HBV what is the sensitive test to diagnose in wife?
A. HBs Ag
B. HBe Ag
C. I don’t remember the rest of options
Answer is: A
130
835. asthma pt with anxity what to give?
A. bupropion
B. B blocker
C. Aldezemp
D. sertraline
Answer is: D
836. pt with abdominal pain worst with food, No blood or mucus in stool no diarrhea no
constipation Dx?
A. IBS
B. psychotic dyspepsia
C. Inflammatory bowel ds
Answer is: A
841. pt will go for renal daylisis what the most common cause of death?
A. Cardiac cause
Answer is: A
131
844. Case of cystic fibrosis but without mentioning the dx, what u will find else in examination?
A. Nasal polyp
B. Café au lait macules
C. Dermatitis herpetiform
Answer is: A
846. Male came to infertility clinic. On semen analysis he has immotile sperm. He had history of
chronic sinusitis, and multiple URTI. What is the diagnosis?
A. Kartagener’s syndrome
Answer is: A
847. male came with multiple episodes of forceful vomiting, He has abdominal pain, dyspnea,
and Subcutaneous emphysema in the chest, What is the diagnosis?
A. Esophagitis
B. Perforated deudonal ulcer
C. Boerhaave syndrome
Answer is: C
848. Abdominal pain relieved by defecation, no weight loss no night time symptoms non bloody?
A. IBD
B. Chrons
C. Ulcerative colitis
Answer is: A
850. Femal pt with hand joints swelling and deformity morning stiffness Confirm DX?
A. ANA
B. Anti CCb
Answer is: B
132
852. Patient with gastro symptoms + high AST and ALT in above 1000 high direct and indirect
bilirubin?
A. Immune hepatitis
B. Infective hepatitis
C. Two other hepatitis
Answer is: Incomplete Q, if there was SLE in Q > A, otherwise could be > B.
853. Chronic Diarrhea loss of weight did x-ray, lead pipe deformity?
A. Ulcerative colitis
Answer is: A
854. Patient with enlarged lymph nodes on biopsy it shows follicular cells Management?
A. Reassure
B. Refer to surgery
Answer is: B
856. Renal failure? Require dialysis in one year what is the cause of mortality?
A. Pulmary edema
B. Cardiovacular
C. Renal failure
Answer is: B
857. Candida esophagitis, multiple unprotected sex; what other tests u want to do? NEW20
A. HIV
Answer is: A
858. Pt travelled Indonesia, meningeal sings. Cryptococcal meningitis what other tests u want t.
Do?
A. HIV
Answer is: A
133
860. Hydatid cyst 6*5 cm what to do?
A. Surgical
B. percutenouce drainage
C. Metronidazole
D. I don’t remember
Answer is: Incomplete Q, if dauter cyst > A, otherwise more than 5 cm > PAIR and medical
ttt.
863. Hbs antigen and Hbc antibody +ve , anti IgM -ve what type of hepatitis B?
A. Chronic
B. Previous
C. acute
D. immunized
Answer is: A
134
866. Parents have confirmed TB and they have close contact with their child, Tuberculin test
done for the child and th results was 10mm induration, this means? NEW20
A. Positive
B. Negative
C. repeat The test
D. The test is not used nowadays!
Answer is : A
867. DM and hypertension sickle cell trait what kind of stone she has:
A. mixed.
B. pure cholesterole
C. pigmented
Answer is : A
869. Pit with pain admitting then improve, you see multiple gallstones with dilition intrahepatic
duct What you will do for him?
A. Cholesestctomy before discharge
B. Ercp
Answer is : A
871. K/c of multiple sclerosis come to complaining of loss of vision and increased the weakness
in the limbs MRI done show increased the sclerotic area in brain more than the previous
What’s treatment for active phase? NEW20
A. interferons
B. iv corticosteroids
C. Po corticosteroids
Answer is : B
135
873. K/C of TB pt , OE there is dullness on chest, we find out there was plural effusion
How can we confirm this effusion because of TB?
A. AFB in plural fluid
B. adenosine deaminse more than 45 IU
Answer is: B
874. Patient on TPN for a while and his coagulation profile and Vital signs are normal except
INR=3, what is the management?
A. vitamin K
Answer is: A
876. Man, with lose of interest and depression, his wife gave history that he bought fancy car
and bought many things that he doesn’t need few months ago. What do you give for long
term management?
A. Lithium
Answer is: A
877. SCA child with hepatomegaly and splenomegaly 6cm below costal margin with pallor and
fatigue for 10 days. Hb was 50 (normal was more than 100 i think) direct and and indirect
bilirubin were high, Normal platelets?
A. regular blood transfusion
B. splenectomy
C. hydroxyurea
D. steroids
Answer is: B
878. 16-year-old girl with acute migraine headache and nausea and aura. What do you give for
acute management?
A. aspirin
B. triptans
C. oxygen
Answer is: B
879. Adult with positive urea test was given PPI and metronidazole, which of the following drugs
u add to his treatment?
A. clarithromycin
B. tetracycline
C. Other drugs weren’t antibiotics and no bismuth in choices
Answer is: A
136
880. Pt hypothyroidism on levothyroxine dose125, 3 month later increase dose to 200 no
symptom everything normal TSH high - T4 normal What is the cause of lab?
A. small dose
B. non complince
C. 2ry hypothyroidism
Answer is : B
882. infant with s&s of meningitis csf showed positive cocci catalase positive what to give ?
A. Ampicillin
B. Gentamicin
Answer is : A
884. Pt came with s&s of osteoporosis what is the first line treatment?
A. calcitonin
B. zoledrenate
Answer is: B
886. How many years does smoke take off your life ?
A. 10
B. 15
C. I don’t know
Answer is : A
137
887. 24 years old patient present with bilateral ptosis and proximal lower and upper limbs
weakness, Simpson test was positive and after doing edrophonium test there was rapid
improvement of the ptosis. What is the most appropriate management?
A. pyridostigmine
B. Rivastigmine
C. Physostigmine
D. Bethanechol
Answer is: A
890. Diabetic patient with no HTN, he has Benign prostatic hyperplasia, what to give?
A. dipine
B. ACI
Answer is: Alfa blocker
892. Male patient with recurrent hematemesis, no other complains, Vitaly stable, labs show
anemia, LFT normal. (not mention any other symptoms) What's your diagnosis?
A. esophageal varices
B. peptic ulcer disease
incomplete or bad recall
C. erosive gastritis
D. Mallory Weiss tear
Answer is: A
138
894. Old pt with small lung nodule (<1cm), he is asymptomatic, not smoker, no family hx of
cancer, no constitutional, your next step?
A. f/u with CT scan
B. biopsy
C. chemo i think
Answer is: A
895. Pt had garlic odor with other nonspecific symptoms what is the diagnosis?
A. organophosphate poisoning
B. Cyanide poisoning
Answer is: A
896. A patient came with headache, dizziness, tachycardia, bronchospasm, vomiting and garlic
smell breath, what is the cause?
A. Organophosphate
B. cyanide poison
Answer is: A
897. Pt after RTA his GCS was 15/15 after a while he lost his consciousness, CT: temporal bone
fracture , what is the cause ?
A. subdural hematoma
B. Epidural hematoma
Answer is: B
898. Women with DIP and PIP arthritis no morning stiffness RF negative, diagnosis?
A. seronegative rheumatoid arthritis
B. Osteoarthritis
C. Reactive arthritis
D. Polyarticular gout
Answer is: A
899. Athlete complaining of medial side plantar pain what is the diagnosis?
A. Valgus
B. Varus
C. Plantar fasciitis
Answer is: C
900. Pt with meningitis CSF culture showed gram negative bacilli, what chemo-prophylaxis would
you treat her contacts?
A. ceftriaxone
B. Ciprofloxacin
C. Rifampicin
Answer is: C
139
901. Case of Patient with fever, CBC had increased WBCs and patient was hypotensive. Choices
were?
A. sepsis
B. septic shock
Answer is: B, N.B: there was no sever sepsis in answer.
902. 45 males come to the ER with upper GI bleeding, no other medical history on PE normal
abdomen no tenderness or guarding? lab decreases Hb, increase BUN?
A. Perforated PU
B. Esophageal varices
C. Mallory wess disease ?
D. Erosive esophagitis
Answer is: B
903. Case about 17 years old complain of fever, sore throat, take amoxicillin from private
hospital then after day develop macular rash and general lymphadenopathy?
A. Streptococcus
B. Infectious mononucleosis
C. Other bacterial
Answer is:B
904. 45 years prediabetic with family history of DM BMI 31 most important advice to maintain
healthy weight?
A. 30 min exercise 5 day a week
B. 30 min exercise most time of the week with low carbohydrate diet
C. low fat diet
Answer is: B
907. Pt k/c of asthma and mitral regursh presented with increase SOB and grade 3/6 murmur,
Dx?
A. spirometry
B. Echo
Answer is: B
140
908. A test applied among 800 diseased persons and 800 normal persons, detect 200 of the
diseased persons and 100 of the normal persons, what is sensitivity of the test? NEW20
A. 25 %
B. 33 %
C. 77 %
D. 90 %
Answer is: A
910. Pt have inflamed hand, do wash her hand to relieve anxiety and wash again if she touches
anything?
A. Obsessive compulsive
Answer is: A
911. Woman have chest pain (or abdominal) otherwise normal, mentioned to have similar
episodes in the past and go within seconds?
A. Anxiety
Answer is: A
913. Patient with lobar pneumonia and moderate pleural effusion (they use "moderate" without
any number). What is the appropriate next step?
A. Abx (azithromycin and ceftriaxone (I don't remember clearly)
B. thoracentesis
Answer is: Both Abx and thoracocentesis
914. Patient was diagnosed with rapidly progressive glomerulonephritis. What is expected in this
condition?
Answer is : Crescent shaped formation
141
916. Child e diarrhea, wt 18kg, maintenance fluid will be (ml/hr)?
A. 56
58.3
Answer is : A
917. child with eczema and recurrent respiratory infection and thrombocytopenia. his uncle and
his cousin also have the same complain what is the dx?
Answer is: Wiskott aldrich syndrome
919. Burn case black soot over nostrils and mouth 40% carboxyhemoglobin. (Carbon monoxide
toxicity)?
A. 100% oxygen followed by hyperbaric oxygen
B. Intubation and ventilation.
Answer is: B
920. 9 years girl presented with abdominal pain after ingested two boxes (each 20 tablets) of
paracetamol one day ago after fight with her mother? your management?
A. N-acetylcysteine
B. Gastric lavage
C. Observation and discharge patient if she had no symptoms
Answer is :A
921. young male known addict in coma RR: 8 sat :82% low BP What is your first action?
A. take detailed hx from family
B. intubation and mechanical ventilation
Answer is : B
142
924. Pt with CKD have risk of DVT what to give ?
A. unfractionated heparin
B. warfarin
C. aspirin
D. LMWH
Answer is: A
925. Old patient presented with symptoms of 2 years memory loss and behavior changes. Once
a caring parent, now he is agitated and aggressive. He is independent with his daily needs.
(Early Alzheimer) Which of the following is the most appropriate next step?
A. Cost effective laboratory test
B. Refer to geriatric
C. Admit into nursing facility
Answer is: A
926. 32 women smokers with polyarthralgia and malar rash fatigue has positive ANA she is on
hydroxychloroquine and mycophenolate Lab 24urine high Which of the following non
pharmacological would help?
A. wt loss
B. Smoking cessation
C. Low protein diet
D. Reduce physical activity
Answer is : B
927. Patient with DM and HTN presented with ingrown toe nail, what is the thing you must do
before the operation?
A. Check peripheral pulses
B. Check the other toe nails
C. Take swab culture
D. Antibiotics
Answer is :A
928. 30-year-old man, all laps were provided and a picture of dip stick shows +2 proteinuria
Also, on microscope? Oval fat bodies and hyaline casts?
A. nephrotic syndrome
B. nephritic syndrome
C. interstitial nephritis
Answer is: A
143
929. Pregnant lady, immigrant, night chills Productive cough, cervical lymph nodes, inter feron
gamma release assay was positive Which test would confirm the diagnosis?
A. chest x ray
B. sputum culture
C. WBC count
D. biopsy?
Answer is: B
930. Case about GERD patient her symptoms did not subside on omeprazole 20 mg, burning
sensation at bed time, her BMI is 28, did endoscopy and found lower esophagitis How do u
mange?
A. give ranitidine at bed time
manometry
B. antiacids
C. Nissan fundoplication
D. i don't remember but was not increase dose of PPI or sth and wasnt related.
Answer is: incomplete Q.
931. Herat failure patient complains of 3 months of nonproductive cough, he is on ACE, BB and
another drug Not smoker, ejection fraction is 40%, How u mange? NEW20
A. stop ace
B. do pulmonary function test
C. start SABA
D. ido t remember but not related
Answer is: A
932. ECG picture with PR depression and global ST elevation, What is the treatment?
A. NSAID
B. Steroid
Answer is : A
933. Patient with clear picture of SLE (Acute flare of multiple joints pain with malar rash provided
by the labs) what to give?
A. Steroid and hydroxy
B. Steroid and methotrexate
C. Steroid& biological drug
D. Steroid & other biological drug
Answer is: A
934. Case about someone take a lot nutritional supplements ... and show symptoms and
vomiting black What does he have?
A. Iron...
B. Hypervitaminosis
C. Hyper protein
Answer is : A
144
935. Picture of sickle cell smear asking what to do next?
A. Hgb electrophoresis
Answer is: A
936. Female with infertility had high: FSH LH prolactin TSH (i thinks it was like this)?
A. Hyperprolactinemia
B. Hypothyroidism
C. PCOS
D. Hypopituitarism
Answer is: according to T4 level, if low > B.
937. Elderly with hypertension on 4 meds with asymmetrical kidneys what does he have?
A. Renal artery stenosis
Answer is : A
938. Pleural effusion due to Malignancy did multiple pleural tap what will you do now?
A. Chemical pleurodesis
Answer is : A
145
944. Adult case of intussusception tx?
A. Surgical resection
B. Surgical reduction
C. Enema reduction
Answer is: A
946. A psychiatrist is evaluating a patient, he asked him (do you think you have an illness?). What
is the doctor testing the patient for?
A. Judgment
B. Insight
C. Believes
Answer is : B
948. Pt with report from a private clinic with Hep B serology: HepB surface Ag negative, HepB
surface AB positive, HepB AB IgG negative, what is the pt status?
A. acute infection
B. Chronic infection
C. Immunized
Answer is: C
146
950. 26-year-old with sickle-cell and your present with paler, lethargy and headed, Hb was 44
and reticulocyte 2.8?
A. sequestration crisis
B. hemolytic crisis
C. aplastic crisis
D. thrombotic crisis
Answer is: B
952. Long case patient with hx of Dm HTN TIA, calculate his chads score?
A. 2
B. 3
C. 4
D. 5
Answer is: according to age C or D.
953. PAD 100-meter claudication's, DM heavy smoker long scenario not important the question
was how to improve his walking distance?
A. supervised exercise program
B. strict glycemic control
C. smoking cessation
Answer is: A
954. Asymptomatic pt with clinical results of (subclinical hypothyroidism), TSH was > 10, how to
manage?
A. F/U in 6 months
B. thyroxine
C. FNA
Answer is: B
955. 2 questions about SLE with active arthritis, what to give? (One was already on
hydroxychloroquine, the other was written with all choices)?
A. methotrexate
B. mofetil
C. sulfasalazine
D. cyclophosphamide
Answer is: A, N.B: no steroid in answer.
147
956. Pt in his 60’s, presented with vague Sx of mild abdominal discomfort, loss of wt and
appetite, what to do next?
A. plain abdominal CT
B. esophagogastroduodenoscopy
Answer is: if it was epigastric pain > B, otherwise > contrast CT.
957. Elderly pt moved to care center before 4 months, now he c/o loss of wt and appetite, low
mood and loss of interest for 4 w, also complain of loss of short memory with attacks of low
mood, what is the Dx?
A. alzehaimer
B. depression
Answer is : B
958. Male with non-specific abdominal pain, all labs are normal, adrenal hormones are normal,
on CT: adrenal mass of 5cm, hypodense and fat content, Mx?
A. observe
B. adrenalectomy
Answer is: B
960. Pt S&S of pancreatitis and ask about which factor indicate worse prognosis?
A. HMC >44%
B. high amylase
C. high lipase
Answer is: A
962. a patient came with progressive weakness and ptosis that worsen in the end of the day and
his symptoms become better in the morning, what is the cause of his symptoms?
A. autoantibody to ach
Answer is : A
148
963. patient CKD with hyperkalemia, what will you do?
A. Insulin with dextrose
B. Ca gluconate
C. Hemodialysis
Answer is : if he symptomatic or K more than 7 > B, otherwise > A.
968. Pic of CXR of rt lobe consolidation with long scenario of SCD pt presented with (chest pain)
and dyspnea and back pain, what is the Dx?
A. Acute chest syndrome
Answer is : A
969. patient with cannon A waves and raised JVP asking about the mx?
A. ICD
B. Permanent pacemaker
Answer is : B
149
971. A 76 years old COPD patient he is taking salmeterol and albuterol 2-3 per day. He had 2
exacerbation and admission in past 6 months which of the following is consider a
maintenance drug for him?
A. prednisone
B. tiotropium
C. formoterol
Answer is: B
972. 30s female complaining of SOB arthralgia dyspnea fever she had dental procedure 2 weeks
ago and another procedure 2 months ago on PE there is heart murmur radiation to axilla.UA
showed hematuria and proteinuria what is the diagnosis?
A. infective endocarditis
B. post streptococcus Glomerulonephritis
C. SL
Answer is : A
973. Someone who is vaccinated but never got infected with hepatitis B virus, which of the
following will he have?
A. HBc Ab
B. HBs Ab
C. HBe Ab
D. HBe Ab and HBs Ab
Answer is: B
974. A female has a son with sickle cell, now she has "remarried" and wants genetic testing for
Saudi premarital screening what to offer (something like that)?
A. paternal electrophoresis
B. maternal electrophoresis
C. paternal chromosome
Answer is: A
975. (Long case) patient has “barking cough”, what is the most likely finding is auscultation?
A. wheezing
B. stridor
C. crackles
Answer is: B
976. (Long case) patient has midsystolic murmur grade 2/6 best heard at right lower sternal
border, and has S4 NO S3 best heard at the apex. What is the most likely diagnosis?
A. aortic stenosis
B. mitral regurgitation
C. aortic regurgitation
D. mitral stenosis
Answer is: A
150
977. Patient with symptoms of generalized anxiety disorder, how to manage?
A. SSRI
B. TCA
C. Benzodiazepine
Answer is: A
978. Female smoker + using OCP for 6 years she went for a surgery, during the surgery she
developed tachycardia, hypotension. what is your dx?
A. MI
B. PE
C. hypovolemic shock
D. septic shock
Answer is: B
981. female with LT leg swelling, intact distal pulse. Best investigation?
A. doppler venous
B. Doppler arterial
C. CT angio
D. CT venum
Answer is: A
151
984. Electrolytes disturbance on TPN?
Answer is: hypophosphatemia
986. Patient K/C of SLE, which of the flowing type of anemia associated with SLE? Hemoglobin
electrophoresis: normal, Ferritin: High, Reticulocyte: High, Lactate dehydrogenase: high,
Haptoglobin: low? NEW20
A. anemia of chronic disease
B. Hemolytic
C. Iron deficiency
D. Thalassemia
Answer is: B
987. patient complain of headache associated with lacrimation and red eye, what to give to
prevent this type of headache?
A. Verapamil
B. (one of the b blocker)
C. O2
Answer is: A
989. Female with HCV ab +, asymptomatic and all labs and LFT normal. what is the best next
step?
A. no need
Answer is: A
152
992. hyponatremia, low urine osmolarity?
A. SIADH
Answer is: A
995. man has painful neck swelling which is moving with swallowing + abnormal TFT, dx?
A. hashimoto thyroiditis
B. Subacute granulomatous thyroiditis
Answer is: B
996. 28 years man. Working in wood factory, he is having wheeze and SOB, Occupational asthma
was suspected, what is the most appropriate investigation?
A. Serial peak Flow measurements in work and in house
B. IgE measurement
C. Skin prick
D. patch test
Answer is: A
997. 25 males with known to have chronic asthma for 5 years, he is on LABA, he is complaining
of palpitation, when he speaks in public, he feels more palpitations and SOB, what to add to
relive his anxiety?
A. BB
B. Bupropion
C. sertraline
Answer is: C
998. When to stop metformin if GFR is? In other recall at which GFR metformin is harmful for
diabetic pt ?
A. 30-40
B. <30
C. <15
Answer is: B
153
999. Pt with tonsillitis when u examined u find white exudate, what’s most common
complication?
A. pharyngitis
B. scarlet fever
C. Glomerulonephritis
Answer is: B
1000. amebiasis and stool mixed (blood +mucin) and they ask about organism?
Answer is: ENTAMOEBA HISTOLYITCAL
1004. Inflamed knee with crystal formation what you will give?
A. Indomethacin
Answer is: A
154
1008. Beriberi disease caused by which vitamin deficiency?
A. B1 thiamin
Answer is: A
1011. you did CT or X-ray for something (I forget) but you found mass in adrenal gland what is
the most common mass you found accidently?
Answer is: non funaction adenoma
1012. Patient has depression and on paroxetine and she is thinking of getting pregnant
What will she do?
A. stop paroxetine
B. Continue
Answer is: A, N.B: in 2019 Q bank severe depression.
1013. Pt with IBS take medication (loperamide + antispasmodic) that not resolve the symptoms.
And there’s no sign of new symptoms appear what to do?
A. Behavior therapy
B. Antidepressants
C. Psychiatry TCA
Answer is : B
1014. Pt fever and other on inspection nothing but in rectum exam you found boggy swelling +
minimal discharges What’s the diagnosis?
A. Perianal fistula
B. intersphenicter abscess
Answer is : B
1015. obese BMI 40 Lab Low LH. FSH ..normal prolactin ..TSH (I don’t remember) but I think low
2*3 adenoma What’s dx?
A. Morbid obesity
B. Nonfunctioning adenoma
C. Hypothyroidism
Answer is : B
155
1016. When to do prolactin again or when to do prolactin ?
A. Double
B. Pregnancy
C. Visually impaired
Answer is: C, the correct Q is when to do MRI.
1018. Pt known to treated from lungs cancer now have dyspnea, quite heart sound, lung clear
How to confirm dx?
A. Chest x Ray
B. ABG
C. Echo
D. ECH
Answer is : C
1019. Woods injured in abdominal with signs of infection (fever mucopurulent discharge, tender
..) best nest step?
A. Culture and sensitivity of discharge
B. Ct abdominal
Answer is: C
1023. pt with bleeding and a lot lab investigation (fibrinogen low) dx?
A. DIC
Answer is: A
156
1024. Pt with bronchogenic carcinoma, presented with progressive SOB, on
P/E there’s elevated jvp, clear lung and quiet heart sounds. What will confirm your Dx?
A. CXR
B. ECHO
C. ECG
Answer is: B
1025. 67y old man felt dawn complain of pain on hip and knee assessment releved no fractures
foly catheter inserted on first day as he was unable to move with analgesic after 5 days, he
discharged to nursing home with unchanged foly catheter now complain of fever urine
analysis show full wbc in urine with bacteria, this could be prevented by?
A. Prophylactic antibiotics
B. Increase iv fluid intake
C. Daily review for the need of foley catheter
D. Change foley catheter before discharge
Answer is: C
1029. Doctor wants to do a total thyroidectomy; patient has mitral valve prolapse and mild
regurg what will you give for prophylaxis?
A. ampicillin for IEC
B. cefazolin for wound infection
C. no need
D. both
Answer is: C
157
1030. Old pt with hx of infective endocarditis more than 6 months ago he will do general surgery
what you will do?
A. IV gentamicin
B. Iv vancomycin
C. No need
Answer is: C
1031. Female came complains of headache + sudden painful loss of vision, Examination: loss of
vision in right eye. CT brain: pituitary haemorrhage MRI brain: pituitary mass, hge +
compressing on optic chiasma & cavernous sinus. Best next step?
A. Close observation
B. Medical therapy
C. Inferior petrosal sinus sampling
D. Urgent neurosurgery referral
Answer is: D
1032. 35 years old nullipara treatment on hyperprolactinemia past year. When pituitary imaging
indication?
A. prolactin level twice normal
B. persistent galactorrhoea
C. blurred vision
D. pregnancy
Answer is: C
1033. pt present with massive pleural effusion causing progressive SOB, caused by malignancy?
A. chest tube with under water drain
B. thoracentesis
Answer is: chemical pleurodesis
1034. Patient with metastatic bronchogenic tumour with recurrent pleural effusion with multiple
pleural taps in the past: What is the beat for management?
A. Thoracocentesis
B. Chest tube
C. Chemical pleurodesis
Answer is: C
158
1036. Indication for thoracocentesis?
A. Ph > 7
B. Glucose 60
C. Loculated pleura
Answer is: C
1037. Patient coughing blood, with pleural effusion that has high protein, glucose and LDH and
high lymphocytes What is results?
A. Exudative due to malignancy
B. Exudative due to heart failure
C. Transudative due to malignanc
Answer is: A
1038. Pt with bronchogenic cancer has recurrent pleural effusion and did
multiple Thoracentesis what should be added to the management?
A. Ultra-filtration
B. Diuretics
C. Thoracotomy
D. Low salt albumin
Answer is: C Pleurodesis,Indwelling pleural catheter,Tube thoracostomy
1040. Elderly smoker with progressive SOB, on CXR massive Rt pleural effusion (that's it no more
info) what's your next step?
A. Thoracocentesis
B. Intercostal tube under water seal
Answer: A
1041. Pt develop spontaneous pneumothorax resolved after two weeks without intervention
what to advise him to not occur again?
A. Not to go on an airplane for one years
B. Not to play football for one year
Answer is: Avoid travel 1 week
1042. COPD drowsy and high CO2, how you will ventilate?
A. Non-invasive MV (BIBAB)
Answer is: A
159
1043. Pt with multiple stone passing in the urine and fatigue Us shows multiple stones in the
kidney Labs PTH HIGH CA IS normal phosphate is normal how to confirm the Dx?
Answer is: Sestamibi scan.
1044. Pt is with non-acth dependent cushing for adrenalectomy, what is the postoperative
management?
A. post-operative fludrocortisone
B. postoperative metatone
C. preoperative hydrocortisone
D. pre-operative alpha blocker
Answer is: C
1045. typical cushing scenario, but the patient has low ACTH. and they mentioned she has taking
topical steroid for skin options?
A. iatrogenic cushing
B. abdominal CT
Answer is: A
1046. Asymptotic pt. with hemochromatosis with high ferritin level what to do?
A. check ferritin after 6 months
B. gastric endoscopy
Answer is: Repeated phlebotomy
1047. Pt. K/c of DM and take steroid for asthma after surgery admitted to ICU, Lab: glu (2.3) ref
(3) normal, Na = normal other lab all normal, reason for admission?
A. DKA
B. Adrenal insufficiency
C. Conn'ssyndrome
D. No Cushing choice
Answer is: B
1048. pt did graham patch for gastric perforation, then he had esophageal bleeding, and ask
about the next appropriate step in management?
A. Endoscopy
B. Laparotomy
C. CT scan
Answer is: A
160
1049. Patient after prolonged difficult ERCP, developed neck, chest and abdomen surgical
emphysema. What was injured?
A. Esophagus
B. trachea
C. Duodenum
D. bile duct
Answer is: C
1051. Patient underwent esophageal dilation. After 12h patient had symptoms and they did
gastrografin which showed leak. Temperature is 39. BP 100/50?
A. Stent
B. Esophagectomy
C. Surgical drainage
Answer is: C
1054. Early satiety, with 10% weight loss in 8 months, Haemoglobin was 8.5, with low MCV, what
to do next in management?
A. Empirical H. pylori eradication treatment
B. Gastroscopy
Answer is: B
161
1056. She is on warfarin she fell down and had an intercranial bleeding? Anticoagulant Rx
management?
A. FFP
B. VIT K
C. FFP AND VIT.K
Answer is: C
1057. Pt known case of lung cancer and had history of epilepsy and on medication, which
medication is CI to this pt?
Answer is: Bupropion
1059. pt w/ hypotension, cold extremities, radial pulse more than femoral. Case of coarctation of
aorta, what is the next step?
A. CT
B. X-Ray initial
Answer is: B
1060. Clear case of coarctation of the Aorta (absence and delay of femoral pulse) asking about
the confirmatory test?
A. ECho
B. X ray
C. CT angio
Answer is: A
1062. 65 years old with DVT, which of the following factor indicate thrombophilia assessment?
A. age
B. Hx of ocp use
C. Connective tissue disease
D. Negative family hx
Answer is: C
162
1063. A lady presents to your clinic after cardiac event. Which of the is true about her condition?
A. Antiplatelet use for short duration prevents ischemic disease
B. CCB is indicated indefinitely in patients with ischemic heart disease
C. Oestrogen replacement therapy in post-menopausal women prevents ischemic
events
D. ACE inhibitors are indicated indefinitely for heart failure
Answer is: D
1064. Pregnant women eat at nights and had heartburn symptoms during bed rest what is the
management?
A. Lifestyle modification
Answer is: A
1065. Elderly with GERD not respond to PPI what is the next step?
A. manometry
B. 24h monitoring PH if normal endoscopy
Answer is: B
1067. pt with thyroid mass about 2*2 in left side just below the angle of mandible did FNA
revealed follicular thyroid what is the dx?
A. Ectopic thyroid
B. Apparent thyroid
C. Metastatic thyroid
D. Thyroglossal cyst
Answer is: C
1068. lady with 2-year history of frontal sinus area headache. The headache usually lasts
between 12-24 hrs. This time the headache is associated with nausea and is continuo days.
She takes over the counter combined decongestant and analgesic but it does not relieve the
pain. There is no papilledema. What are you gonna do?
A. CT scan of the sinus
B. brain MRI
C. more history and physical
Answer is: C
163
1069. ung female with thyroid nodule. TSH is low and T4 is high. FNA was inadequate. Next?
A. FNA
B. Thyroid isotope scintigraphy
Answer is: B
1070. Female with family hx of crohn's ... but symptoms of UC rectal pain and bloody, what is
next to establish diagnosis? NEW20
A. Upper endo
B. Fecal calprotectin
C. Anorectal manometry
Answer is: B
1071. 30 years old female, not pregnant, presents with fever and flank pain, urine culture shows
50,000 E coli. What is the appropriate empirical antibiotics?
A. piperacillin/tazobactam
B. Nitrofurantoin
C. ceftriaxone
Answer is: C
1072. female with calcium high, PO4 low vitamin D low. Diagnosis?
A. 1ry hyper PTH
B. 2ry hyper PTH
Answer is: A
1073. Lady obese with DM what tell her about the diet beside the excursive?
A. Low fat diet decreases 5-10% of the weight
B. low glucose index lowers the weight 5-10%
C. low carbohydrate more effectively than low fat
Answer is: B
1074. 44-year-old male married for three years complaining of decreased libido painful tender
breasts by lottery bilaterally breastmilk can be expressed manually he came in for an
evaluation of his complaints physical examination is otherwise normal, prolactin level is very
high what investigation would you like to do?
A. brain MRI
B. Ct scan
C. Adrenal levels
D. Abdominal us
Answer is: A
164
1075. female pt present with prolactinemia milky discharge from both breasts' prolactin level
was 1400 and there was what to do?
A. Surgery
B. Carbogline
Answer is: B
1076. Patient with cushing features, labs showed low ACTH and 24 urine cortisone was
high...what to do?
A. Head CT
B. Abdominal CT
Answer is: B
1078. recurrent fractures, kidney stone, hyper parathyroid, other labs normal?
A. 24 h urine ca
Answer is: sestamibi scan
1079. 25 y/o female with unilateral leg swelling without any history recent surgery or trauma.
What also you will ask in the history?
A. history of OCP use
Answer is: A
1080. Ultrasound finding (which was provided) renal with multiple nodules?
A. Polycystic kidney disease
Answer is: A
1081. Female pt with leg pain and swelling, what you will ask in history?
A. history of OCP
Answer is: A
165
1083. Female came with left leg pain and swelling with red streak, what is the causative
organisms?
A. Strept pyogen
B. bacteroides
C. klebsiella
Answer is A
1085. 27 yo lady c/o urine come out from the virginal, during micturition?
A. Urethroviginal fistula
B. Ureterovaginal fistula
C. Vesicovaginal fistula
Answer is: A
1086. Postpartum female with a 2-month history of morning stiffness (> 2 hours) of the proximal
interphalangeal joints of the ring and index fingers and both wrists. Labs: Low Hb, High CRP?
A. Viral arthritis
B. Rheumatoid arthritis
C. Reactive arthritis
Answer is: B
1087. fluid within thick wall in lesser sac post treatment of pancreatitis?
Answer is: pancreatic pseudocyst, conservative ttt, unless infected > percutaneous
drainage.
1089. case of obese pt his wife said he is snoring loudly what is the next step?
Answer is: Polysomnography
1090. Case i think the dx was sleep apnea what is the investigation?
Answer is: Polysomnography
1091. Case i think the dx was sleep apnea what is the treatment?
A. home o2
B. nasal cpap
Answer is: B
166
1092. Copd elderly admitted exuberation at night became aggressive disoriented what the nurse
should do first?
A. Call the duty dr for lorazepam iv
B. Restrain her
C. Call family sit beside her
D. Elevate the head of bed put nasal oxygen try to orient her about place and time
Answer is: D
1093. 70years old admitted to day care in the past 2 weeks he had decreased interest,
decreased weight and appetite he is away from his family and he has short term memory
impairment diagnosis?
A. Alzheimer
B. depression
C. vascular dementia
D. Delirium
Answer is: B
1094. patient DM and has symptoms of UTI in lap high creatinine what’s contraindicated?
A. nitrofurantoin
B. cipro
Answer is: A
1096. Patient with basal ganglia disorder and absent gag reflex. How to provide nutrition?
A. Gastrostomy
B. Jejunostomy.
C. NGT
Answer is: B
1097. Female had femoral fracture then after I think a week developed respiratory symptoms?
A. Fat embolism syndrome
Answer is: A
167
1099. after surgery what is best DVT prophylaxis?
A. enoxaparin and mechanical
Answer is: A
1100. Patient diagnosed with GERD taking omeprazole 20mg but not improving, what to do?
A. Add ranitidine at bedtime
B. Same treatment twice daily
Answer is: B
1101. Patient 23 old came with murmur “forgot the description sadly”, she had history of TOF
repair at age 3 years?
A. VSD
B. MS
C. AR
D. PR
Answer is: D
1102. Patient after pancreatitis episode develops upper GI bleeding picture, scope was done,
gastric fundus bleeding was found, sclerotherapy done Duplex ultrasound showed: splenic
vein thrombosis with patent portal vein what is your management?
A. Splenectomy
B. Portal-systemic shunt
Answer is: A
1103. Septic arthritis started on cloxacillin: report came after 3 days with staph aureus resistant
to cefotaxime (cefoxitin)?
A. start Vanco
B. add gents
C. continue cloxacillin
Answer is: A
1104. A 42 YO female P6036 complaining of vaginal fullness and heaviness increased through the
day. She uses manual pressure to empty her bladder completely. She has stress
incontinence. Diagnosis?
A. cystocele
B. rectocele
C. enterocele
D. periodoncia
Answer is: A
168
1105. pancreatitis case managed conservatively at the day 3 the pt improving clinically and
laboratory. Everything normal except Amylase 250. US showed>> Gallstone at the gall
bladder with dilated extra hepatic ducts (nothing was mentioned about CBD). What is the
appropriate management?
A. lap chole before discharge
B. elective lap chole
C. ERCP
Answer is: A
1106. Patient known case of asthma on inhaled corticosteroids and short acting beta agonist as
needed what to add?
A. Add LABA
Answer is: A
1108. give vesicoureteral barium and there’s dilatation in one of ureter & bladder & pelvic?
A. cystourethral reflux
Answer is: A
1111. 70 y/o female bed bounded due to basal ganglia (bleeding or damage can’t remember
clearly), weak mastication muscles, absent gag reflex, losing wt, (no mention of coma) how
are going to feed her?
A. NGT
B. Can’t remember the others
Answer is: jejunostomy tube
169
1113. Strong factor in Graves opth?
A. Male
B. Smoking
C. increased free t3 & t4.
Answer is: B
1115. 60 y/o smoker with pleural effusion. What is the cause of it?
A. Esophageal rupture.
B. Esophageal cancer.
C. Gastric cancer
Answer is: B
1118. Pt complaining of fever and headache with picture of blood smear (schistocytes) hige ptt ,
pt , low fibrinogen, what is the diagnosis?
A. DIC
B. ITP
C. TTP normal coagulation profile
Answer is: A
170
1120. Old patient known to have hepatitis C, present with weight loss, fatigue... US report show
mass (forget description), what's is the diagnosis?
A. hepatocellular carcinoma
B. Hepatocellular adenoma
Answer is: A
1121. Female patient with complain of change behaviour and other thing before menstrual cycle
and resolved in the second day of menstrual cycle affect her work “absence” and increase
visit to ER, what is related to her condition?
A. mood swing
B. Irritability
C. Two more option related to mood
Answer is: A
1123. Child ingest multi vitamins, present with black vomiting, what is the diagnosis?
A. iron poisoning
Answer is: A
1124. Child ingest 20 tablet acetaminophens one day ago, present with right upper quadrant
pain what is the antidote U use in her condition?
A. N-acytyle
B. Deferox
C. No need to treat in her stage
Answer is: A
1125. Femur fracture the developed dyspnoea + x ray shows diffuse finding + rash what is the
diagnosis?
A. PE
B. fat embolism
C. 2 other option
Answer is: B
171
1126. Patient known to have hashimito disease present with a enlargement of thyroid with
heard bruit and US show extension of thyroid what is the cancer?
A. follicular
B. Medullary
C. Lymphoma
Answer is: C
1127. Midline mass compressed symptoms cause dysphagia and dyspnoea what it the
diagnosis?
A. lymphoma
B. Thymoma
C. Goiter
Answer is: B
1128. Patient has allergy after taking contrast CT what initial to do?
A. adrenaline
B. Intubation
C. Steroid
Answer is: A
1129. Pt chronic alcoholic, complains of progressive dysphagia, he’s cachectic on Digital Rectal
Exam there’s clots of blood with stool. Dx?
A. Esophageal ca
B. Acidic peptic disease
C. Pancreatitis
Answer is: A
172
1133. Patient K/C of HTN on medication, with prostatic enlargement, no sign of malignancy,
Vitaly stable?
A. Alpha blocker
B. Surgical
Answer is: A
1134. Patient with cannon A waves and raised JVP asking about the mx?
A. ICD
B. Permanent pacemaker
Answer is: B
1135. Patient with chest pain during exercise, normal resting ECG, what to order?
A. Exercise Stress
B. ECG
Answer is: A
1136. pt in 20s c/o sharp pain centrally after activity for 3 weeks, what next?
A. Reassure and follow up 1 week
B. Nitrate
C. Ibuprofen
Answer is: C
1137. Pt Elderly with hypertension + on routine cheak up and on cardiac exam we found there is
(sever aortic stenosis) and (left ventrical failler) with normal EF = PT Asymptomatic?
A. Aortic valve replacement
Answer is: bad recall, but mostly is A.
1138. Pt with pleuritic chest pain and ECG pic of diffuse elevated ST. mx? (ECG pic.)
A. NSAID
Answer is: A
1139. Patient with previous viral infection then pleuritic chest pain ecg shows diffuse ST
elevation and PR depression what to expect on chest exam? (ECG pic.)
A. Pericardial rub
Answer is: A
1140. ECG with tall T wave and potassium level 6.5 immediate action? (ECG pic.)
A. calcium gluconate
B. insulin
C. dialysis
D. sodium bicarbonate
Answer is: A
173
1141. asthmatic patient, last OPD peak flow was 600, when to say it is acute severe asthma?
A. peak flow of less 250
B. O2 saturation 94%
Answer is: A
1142. Smoker, SOB, cough yellowish sputum, fever, examination wheezing, febrile, Dx?
A. community-acquired pneumonia
B. Acute bronchitis
C. COPD.
D. TB
Answer is: A
1143. Alcoholic patient presented to the ER cyanosed and loss of conscious how you will
manage?
A. CPAP.
B. Mechanical ventilation
C. Nasal oxygen.
D. Aminophylline infusion
Answer is: B
1144. Sickle cell disease patient. Present with crisis. Respiratory symptoms. Splenomegaly.
Hemoglobin 3.4 g/dl (I’m sure of value). What’s the best initial step in management?
A. Splenectomy
B. IV fluid and analgesia
C. RBC transfusion
Answer is: B, if the case is ACS > C
1145. Women with sickle disease had children with sickle cell or lost children because of it not
sure she married a new husband and wants to know if she gets pregnant what are the
chances of her children to have it, what’s ur next step?
A. genetic testing of both
B. electrophoresis of new husband
C. electrophoresis of the woman
Answer is: B
1146. Patient has fever and headache petechiae. what’s the diagnosis?
A. Hus
B. TTP
C. ITP
Answer is: B
174
1147. Pt complains of SOB and difficult swallowing U/S Midline mass compress trachea?
A. Goiter
B. Thymoma.
C. Lymphoma
Answer is: bad recall (US?!), if neck mass extends to chest > A, otherwise > B.
1151. 26 yrs male patient with crampy abdominal pain not related to specific food with diarrhea
alternating with constipation?
A. Irritable bowel.
B. chrons
C. lactose intolerance
Answer is: A
1153. SLE nephritis and impaired renal function + proteinuria what lifestyle modifications to do?
A. smoke cessation
Answer is: A
1154. 5 days post orthopedic surgery had sudden dyspnea and confusion on examination shows
rash on neck and on cxr bilateral lower lobe infiltrates. cause?
A. Fat embolism
B. PE
C. pneumonia
Answer is: A
175
1155. How many quality years does smoking take from a smoker?
A. 10
Answer is: A
1156. Old pt with hx of recent travel, came with difficulty getting aroused/awaken, he reports
multiple falls, examination shows no head wounds and skull intact, most likely diagnosis?
A. post-concussion syndrome
B. chronic subdural hematoma
Answer is: B
1158. Patient has GRED and takes ppi and improved after developing H. Pylori and eradication
what tell patient after that?
A. no thing
B. need to increase dose of ppi after that_decrease devolpe of Barret esophagous
Answer is: Decrease dose of PPI
1159. Patient female asthma and mitral regurge 3/6-degree, systolic murmur and sever short of
breathing on SABA and take it on examination lung clear which of the following is most
appropriate diagnosis??
A. Spirometry
B. TTE
C. TEE
Answer is: B
1161. Female 80 old age pneumonia and she rest less and disoriented in ward and noon room o2
saturation 94 temperature 38.8 what nurse do first?
A. Ask from family side bed
B. elevated bed o2 mask and oranted her to surround environment
Answer is: A
176
1162. Male patient COPD confuse disoriented excessive secretion ph 7.23, oxygen saturation 74
pco2 is 6.8, pco2 8.6 mol/l what make to this patient
A. nasal CPAP
B. mechanical ventlation
C. O2 mask
D. Thiopheline
Answer is: B
1164. Female BMI 34 and has polyuria polydipsia weight loss, she has +ve family history for Dm2
she unbooked DM and her fasting sugar 7.5 mmol HA1c is 7.8 what best next management?
NEW20
A. diet and exercise and metformin
B. diet and exercise
C. sulfonylurea
A1c more than 7.5 start metf
D. biguanide
Answer is: B
1165. Patient with spastic movement, clasp knife, what is the diagnostic exam?
A. ECG
B. EEG
C. MRI brain and spine
D. Nerve study
Answer is: C
1166. Patient is having very high blood pressure 220/180, not decreasing with BB or diuretics,
what is the Mx?
A. Increase BB
B. Add CCB
C. Alpha blocker
Answer is: C, if there ACE is more accurate
A case about patient with increase JVP, hypotension with equal air entery to the lung:
A. Cor pulmonal
B. Lung contusion
C. Cardiac tamponed
Answer is: C
177
1167. What to ask asthmatic patient to assess the severity of the condition?
A. How many times you use the medication
B. Are you waking up from sleep due to attacks
C. How many times you come to ER
Answer is: B
1169. pt with history of peptic ulcer and +ve urea breath test. You started and finished the
treatment course. When you can re-examine the urea breath test?
A. at least 4 weeks
B. at least 2 weeks
C. at least 1 week
D. examine now
Answer is: A
1170. patient had knee pain, no trauma no fever and also have epigastric pain?
A. ibuprofen
B. paracetamol
C. aspirin.
Answer is: B
1171. Pt heavy smoker with symptoms of GERD, upper endoscopic examination show’s
squamous cell with high-grade dysplasia, next step in management:
A. Council smoking cessation
B. Endoscopic mucosal resection
Answer is: B
1172. Pt with 2 weeks history of watery diarrhea, vitals were stable What is the expected acid-
base abnormality?
A. Metabolic acidosis
B. Metabolic alkalosis
C. Compensated metabolic acidosis
D. Compensated metabolic alkalosis
Answer is: A or C, accurate answer according ABG
178
1173. Burning chest pain for 6 months increased at night and unpleasant taste when lifting
heavy objects?
A. Esophagitis
B. Acute gastritis
C. Boerhaave syndrome (its do blood vomiting)
D. Perforated peptic ulcer
Answer is: A
1175. What symptom makes you suspect eosinophilic esophagitis the most?
A. prolonged chewing of food
Answer is: A
1177. Pt on renal disease and decreased GFR need to dialysis in next year What risk of death in
this pt?
A. Renal failure
B. Coagulopathy.
C. Cardiovascular disease
Answer is: C
1178. Patient HTN has EF 55% and LVH. Cause of HF in this case?
A. Systolic dysfunction.
B. Diastolic dysfunctions.
C. HCM
Answer is: B
1180. Old man falls down and has hx so they put for him FC due to immobility Given
paracetamol and Codeine for pain Then he develops fever and nurse noticed cloudy urine
Mx?
A. Review the need for Foly cath
Answer is: A
179
1181. Patient with cannon A waves and raised JVP asking about the mx?
A. ICD
B. Permanent pacemaker
Answer is: B
1182. Pt w/t mitral valve prolapse and she will undergo for thyroidectomy
A. Give amoxicillin to prevent infective endocarditis
B. No need for prophylaxis
Answer is: B
1183. patient with CHF and DM and HTN not tolerating ACEi what to switch it with?
A. ARBS (lorstan)
Answer is: A
1185. x of DM, heart failure and admitted for hysterectomy and received Normal Salin cuz poor
oral intake, 4 d nurse noticed decreases in Sat and SOB, o/e: crackles How to prevent this
complication?
A. cardiopulmonary consult
B. daily fluid assessment
Answer is: B
1186. Patient with raised JVP, Hypotension, unclear heart sound, clear lung sounds. How to
confirm the Dx?
A. Echo
B. CXR
C. ECG
Answer is: A
1187. male presented with macro-orchidism, long face, wide ears, joint laxity, mid diastolic
murmur best heard at apex area. What is the Dx?
A. Fragile X syndrome
Answer is: A
1188. case with flat T wave what you will find in the pt the?
A. Hypokalemia
Answer is: A
180
1189. Female with 3 wks hs of productive cough with decrease both FEV and ratio What other
finding?
A. Increase lung compliance
B. Decrease lung compliance
Answer is: A
1192. scenario about a man with epilepsy poor controlled, have productive cough and fever
Asked about what the source of infection?
A. Aspiration
B. TB
C. Infectious mononucleosis
D. Pneumonia
Answer is: A
1193. COPD elderly admitted exerberation at night became aggressive disoriented what the
nurse should do first?
A. Call the duty dr for lorazepam iv.
B. Restrain her
C. Call family sit beside her
D. elevate head of bed put nasal oxygen try to oriented her about place and time
Answer is: D, if he not hypoxic > C.
1194. Patient k/c of COPD alert, has moderate respiratory distress.SO2 93%, ABG showed
hypoxia, hypercapnia and acidosis. What is next step?
A. noninvasive ventilation
B. Decrease oxygen
C. Increase oxygen
D. Mechanical ventilation
Answer is: A
181
1196. pt medically free go to tooth extraction and pt develop petechial rash andHb high+
erythropoietin low + Platelet high
A. myelofibrosis
B. essential thrombocytopenia
C. poly sethemia vera
Answer is: C
1198. 18 Y has ALL. 17 days after chemo he develops fever around 38.7 C. No focus of infection
found. Labs shows: low WBC 0.6 (normal was 4-10x..), Neutrophil level 60% (normal 40-
60%), i’m not sure if he has low platelets as well, what you’ll do:
A. Blood urine... culture and paracetamol
B. Blood, urine,... culture and PO Abx
C. Blood, urine,... culture and IV Abx
Answer is: C
1199. Known case of polymyalgia rheumatica presents with malignancy symptoms (weight loss
and other non-specific constitutional sx), what is the diagnosis? Labs showed anemia, low
platelets and increased WBC (lymphocytes)?
A. CLL
Answer is: A
1200. Male pt asymptomatic came for checkup, drinks alcohol occasionally every weekend. LFT
showed mildly elevated enzymes, ferritin 490 high, TIBC high, dx?
A. alcoholic hepatitis
B. hemochromatosis
Answer is: B
1201. Long scenario about pt had dialysis 3 day ago central line and now came again for dialysis
peripheral line nurse mentioned he had fever and infection in site of previous dialysis site
what to do?
A. culture and iv abx and stop dialysis
B. culture iv abx and remove the center line
Answer is: B
1202. Patient was found to have cavity on x ray, what is the type of precaution?
A. Airborne
Answer is: A
182
1203. Facial plethora and JVP distended, which of the following is associated with this condition?
A. SLCC
Answer is: A
1204. Pt with bilateral hydronephrosis with dilated bladder and proximal urethra?
A. Posterior urethral valve
B. Uretropelvic occlusion
Answer is: A
1205. Clear case with headache that is thropining and unilateral aggravating with light and
movement?
A. Migraines
Answer is: A
1208. SLE patients with active arthritis and malar rash treatment?
A. Hydroxychloroquine
Answer is: A, if there steroid is more accurate.
1211. Patient with a BMI of 40, she doesn’t c/o visual disturbance, no hypothyroid symptoms,
she came with lab works of: TSH high, prolactin normal, FSH and LH are abnormal. What is
the possible cause?
A. non-functioning prolactinoma
B. Hypothyroidism
C. Morbid obesity
Answer is: if Q cmae like this > B, in 2019 Q bank simmilar Q with CT adenoma > A.
183
1212. Type visual defect come with pituitary adenoma?
A. Bitemporal hemianopia
Answer is: A
1213. The first marker that indicate acute hepatitis B infection is?
A. HBsAg.
B. HBeAg. IgM is better answer
C. Anti HBc IgG
Answer is: A
1214. Patient is Caucasian and symptoms started when he was introduced to regular food
Patient has no history of drug use or alcohol drinking, presented with vomiting, diarrhea and
jaundice. Which of the following indicates her infection?
A. HAV IgM
B. HAV IgG
Answer is: A
1218. Hemochromatosis asymptotic, liver enzymes within normal only increased in ferritin level
750 Whats appropriate next step?
A. regular phlebotomy every month
B. follow up after 6 months.
Answer is: A
1219. Patient saying that he has a disease, and all the doctors he visited told him he didn’t have
anything and the tests are normal, but still, he is saying that he has, diagnosis?
A. Somatization
B. Something pain disorder
C. Hypochondriasis
Answer is: C
184
1220. What is the favorable place for AV fistula?
A. brachial artery basalic vein
B. brachial artery cephalic vein
C. radial artery basalic vein
D. radial artery cephalic vein
Answer is: D
1222. Biopsy from lymph node show normal follicular cell, next step?
A. Excisional biopsy
B. US
C. CT
Answer is: A
1224. Elderly with meningitis, what prophylactic TTT to give contact people?
A. ceftriaxone
B. Cipro
C. Rifampin
Answer is: C
1225. Patient with (xray show apical lung TB) what type of precautions to do?
A. contact
B. Airborn
C. Droblet
Answer is: B
1226. Pt DM and hypertension. And k/c of chronic HF. Past history of MI. Has decreased systolic
ejection fraction with no symptoms. His K is elevated (~6). Blood pressure controlled
110/70s. He’s on ACEI, frusemide, Atorvastatin. What should you do regarding his med?
A. Add BB(?)
B. D/c ACEI
C. D/C statin
D. D/c furosemide
Answer is: B
185
1227. Pt had some abnormality in her period. When press the breast the milk is expressed
manually and some abnormality? TSH HIGH, T4 (can’t remember), PROLACTINE HIGH, what is
the cause?
A. prolactinoma
B. Hypothyroidism
Answer is: B, according to T4 level.
1228. Pt (forgot what he has) but in labs he has low K (~2). What the meds that seemed to cause
this?
A. Antiacid
B. NSAID
Answer is: pseudonephrine
1229. HYPERTENSION ABD HF. AND LV hypertrophy no valvular lesion. What the cause of HF?
A. Dysfunction in CO
B. DIASTOLIC DYSFUNCTION
C. STOLIC DYSFUNCTION
Answer is: B
1231. admitted pt after pelvis surgery was on enoxaparin bc dx with DVT. Sudden tachypnoea
and tachycardia. ECG of PE and pt is stable?
A. CONTINUE Same medication
B. Thrombolysis
C. IVC filter
D. Switch the drug
Answer is: A
1233. Pt with CKD has hyperkalaemia what should the pt eat with same amount every day?
A. Green leaves
B. Grapes
C. Tomatoes
Answer is: C
186
1234. Old patient has hypercalcemia, what is the important initial step?
A. Normal saline
B. IV pamidronate
Answer is: A
1235. Female pt C/O generalized muscle and bone ache associated with abdominal cramp or
epigastric this symptom resolves when she drink a cup of cold milk what is the diagnosis? Lab
result: ca+2 = normal, phosphate = low, vit D3 = low, PTH = with upper normal level
A. milk alkali syndrome.
B. primary hyperparathyroidism
C. 2ndry hyperparathyroidism
Answer is: C
1236. Case of thyroid cancer (medullary thyroid cancer) best appropriate management?
A. total thyroidectomy
B. hemithyroidectomy
C. Follow up
Answer is: A
1237. Pt with bronchogenic carcinoma, presented with progressive SOB, there’s elevated jvp,
clear lung and quiet heart sounds. What will confirm your dx?
A. CXR
B. ECHO
C. ECG
Answer is: B
1239. A 25-year-old man was referred to the Endocrine Clinic with a recent history of fragility
fracture. He had no significant past medical history and was on no medication. On
examination, he had sparse facial and xillary hair growth. Which of the following investigation
useful in establishing the aetiology of his fragility fractures?
A. Calcitonin levels
B. DEXA
C. Insulin like growth factor (IGF1)
D. Testosterone and gonadotrophin levels
Answer is: D
187
1240. 59 Y.O. female Patient diagnosed with MI and was treated at the hospital, upon her
discharge she asked you what is the best way to prevent reinfarction?
A. Use of antiplatelet for short duration.
B. Use of Calcium channel blocker indefinitely.
C. Use of ACEI if she develops Heart failure to prevent cardiac changes,
D. Use of spironolactone for long duration.
Answer is: C
1241. Old pt after rectal surgery he is not doing well after in the recovery he starts to have leg
pain he developed DVT from the popliteal to the femoral?
A. Enoxaparin
B. Heparin
C. Warfarin
D. IVC
Answer is: A
1242. Young female presented with dyspnoea, low grade fever and arthralgia. On examination
there was tender erythematous nodules on her shin. And on auscultation basal crackles.
Whaic is best nexts step in management?
A. Chest X Ray
B. Steroid
C. Blood culture
D. skin biopsy
Answer is: A
1243. Female pt C/O generalized muscle and bone ache associated with abdominal cramp or
epigastric this symptoms resolve when she drink a cup of cold milk what is the diagnosis? Lab
result: ca+2 = high, phosphate = low, vit D3 = low, PTH = with upper normal level?
A. milk alkali syndrome.
B. primary hyperparathyroidism
C. 2ndry hyperparathyroidism
Answer is: B
1244. Diastolic murmur with pistol shot sound in the femoral artery?
A. aortic stenosis
B. Mitral stenosis
C. aortic regerg
D. mitral regerg
Answer is: C
188
1245. Female coming back from east asia, typical presentation of TB with chest X ray showing
cavitation, what will you do next?
A. Sputum for acid fast bacilli
B. Start anti tb
Answer is: A
1247. Pediatric patient with coryza, conjunctivitis, and white spots in the mouth, what is the
diagnosis?
A. Measles
B. Rubella
C. Mumps
Answer is: A
1248. Child with cough, nasal congestion, fever, fatigue, rash start on face then to the body?
A. coxacki
B. Rubella
Answer is: B
1250. Patient with swelling of left lobe thyroid + move with swallowing no LN involvement >> the
diagnosis was left lobe medullary thyroid cancer + no LN involvement. What is the
management?
A. total thyroidectomy
B. subtotal thyroidectomy
C. lobectomy
Answer is: A
1251. 75 y/o male k/c of HTN, DM, Hx of TIA presented with palpitation. ECG demonstrated 75
bpm, irregular. What is the best next step?
A. give anticoagulation
Answer is: A
189
1252. Case of a cancerous pt did multiple pleural tap due to pleural effusion with no
improvement, next?
A. Chemical pleurodesis
Answer is: A
1253. Pt with TB. CXR revealed pleural effusion. How to knowledge the effusion is due TB?
A. Pleural protein over serum <0.3
B. Pleural LDH > 100
C. Pleural WBC >1000
D. Pleural LDH over serum 1/3
Answer is: C
1254. symptoms of meningitis, how many hours after beginning antibiotics before discontinuing
isolation?
A. 24h
B. 48h
Answer is: A
1255. Case describe typical symptoms of bronchiolitis / asking about the diagnosis (Chronic
productive cough, SOB)?
A. Bronchiolitis
Answer is: A
1256. pt came with low libido and low activity BMI 40, refer to endocrinologist FSH LH TSH T3
low, MRI brain show 2.5 cm pituitary adenoma what's most likely dx?
A. morbid obese
B. non-functioning pituitary adenoma
Answer is: B
1257. Women with pelvic pain did Ct abdomen and found adrenal mass most common adrenal
mass?
A. Non-functioning adenoma
Answer is: A
190
1259. 35 Y.O. Pt was diagnosed with GERD and was started on omeprazole 20mg. not
responding despite mild improvement he still complains of nausea. no dysphasia His P/E is
insignificant. what is your next step?
A. Add ranitidine at bedtime.
B. Refer him for upper Gl endoscopy.
C. Increase omeprazole to 40mg.
D. Follow up after 2 weeks.
Answer is: C
1261. Dm patient on metformin 1 g BID, HA1C 6,9. What is most appropriate next step in
management?
A. Increase metformin dose
B. Give insulin
C. Add gliptin
D. No change
Answer is: D
1262. Pt liver cirrhosis with tender ascites. Which of the following drug is best to prevent
recurrence?
A. Aldosterone antagonist
B. Loop diuretic
C. Osmotic diuretic
Answer is: A
1263. Pt will do CT with contrast after give contrast develop whizzing what you want to do?
A. Steroid
B. Epinephrine
Answer is: B
191
1264. A 36-year-old woman presents to the Emergency Room with a 1-week history of fever,
dysuria, and flank pain. She is admitted to the hospital after fluid resuscitation (a total of 6
litres of isotonic saline) and started on IV antibiotics. She also requires a low dose of
norepinephrine to maintain her blood pressure. The day after admission, she complains of
shortness of breath and requires 5 liters of supplemental oxygen; oxygen saturation is 91%
after this management. Chest X-ray confirms new, bilateral alveolar infiltrates. Bedside
echocardiogram does not show evidence of left ventricular dysfunction, Blood pressure
78/34 mmHg, Heart rate 125 /min Respiratory rate 25/min Temperature 38.6, Oxygen
saturation 95%, Which of the following is the most likely diagnosis?
A. Diffuse alveolar haemorrhage
B. Cardiogenic pulmonary edema
C. Healthcare-associated pneumonia
D. Acute respiratory distress syndrome
Answer is: D
1265. Known case of GERD on PPI, symptoms only mild improvement, endoscopy done:
esophagitis, what's the most important next step?
A. repeat Endoscopy in 6m.
B. Nissen fundoplication.
C. esophageal manometry.
D. ambulatory PH monitoring
Answer is: C
1267. Diabetes patient with severe unilateral knee joint pain, on exam it's erythematous,
swollen, warm. patient has fever, joint aspiration shows I think 55.000 WBC, Culture was
negative, crystals still pending. Diagnosis?
A. Septic arthritis
B. Gout
C. Pseudogout
D. Charcot knee
Answer is: A
192
1269. Post chemotherapy with dry mouth what's the electrolytes abnormalities?
A. Hypercalcemia
B. hypocalcemia
C. Hypernatremia
D. Hyponatremia
Answer is: B
1270. 30-year-old patient with recurrent UTI and electrolytes disturbance, US show multiple thin
wall cyst throughout the parenchyma What is Dx?
A. Adult polycystic kidney
B. Medullary sponge kidney
Answer is: A
1271. old female was on OCPs in the last 12 years the was small hepatic adenoma What is the
appropriate next step?
A. Surgical resection
B. Stop OCPs
Answer is: B
1272. Women took OCP for 12 years came with RUQ pain imaging revealed (mass 5 cm) on the
right?
Answer is: stop OCPs
1273. Patient had bypass surgery and has heart rate of 120 and bp 90/60 and decrease
Increased peripheral resistance, what type of shock is that?
A. Cardiogenic
B. Hypovolemic
C. Septic
Answer is: increase peripheral resistance > A, decrease peripheral resistance > C.
1274. MVA case, tachycardia, hypotension, low pulmonary wedge pressure, what type of shock?
A. Hypovolemic
Answer is: A
1275. A man who is a known case of diabetes presented with hemiparesis 15 hours after some
procedure? NEW20
A. tpA
B. Warfarin
C. Ct angio
Answer is: incomplete Q, if there CT is more accurate, otherwise medical ttt > aspirin
and heparin.
193
1276. Female K/C of HTN and diabetic nephropathy on ACEI, Thiazide diuretics. Presented to the
clinic complaining of lower limb swelling, upon examination he was found to have +3 pitting
edema with blood pressure almost controlled 143/84 Lab wise: Renal profile showed Cr 200
elevated, however this is his baseline the same reading 3 months ago. What is your
action? NEW20
a. switch thiazide to furosemide
b. continue the same
c. stop ACEI
Answer is: A
1278. Lady have DM HTN IHD on ttt now she has HIGH cholesterol how to manage?
A. Statin
B. Diet
Answer is A
1279. Patient with advanced breast ca, had kussmaul respiration? NEW20
a. Dilated cardiomyopathy
b. Obstructive cardiomyopathy
c. Restrictive cardiomyopathy
Answer is: B
1280. Repeated this is the full scenario Elderly man with confusion and gait changes, slow gait
for two months, dx?
a. Frontotemporal
b. Alzheimer
c. CJD
d. NPH
Answer is: D
194
1282. 70-year male HTN with progressive decline in cognitive state MRI shows Periventricular
white matter Hyperintensities what is th Dx?
a. vascular dementia
b. Alzheimer dis
c. Normal pressure hydrocephalus
Answer is: A
1285. Female 3 months post SVD, after one month she complains about having stool inc. and
passing flatus from the vagina?
a. recto vaginal fistula.
b. perineum tear.
Answer is: A
1286. Old woman complaining of inability to control urination. when feels she the need to
urinate can’t stop it till, she reaches the toilet. Type?
A. Urge
B. Stress
C. Overflow
D. mixed
Answer: A
1287. post-partum 2-month breastfeeding and she got UTI (E. coli that is sensitive to nitro,
trimetho/sulfa, ciprofloxacin) she is asymptomatic What you will give?
Answer is: no need treatment
195
1290. male, highest risk factor?
A. BMI more than 31
B. Waist circumference 100
Answer is: B
1291. Pt with compressive fracture with Dexa scan result what’s the diagnosis?
A. Osteomalacia
B. Osteoporosis
C. Osteopenia
D. paget
Answer is: B
1292. year old female on oral contraceptive pill for 12 years, developed gradual RUQ pain
(not something acute), CT was ordered, Hepatic adenoma 5 cm diagnosed, how to manage?
A. stop OCP
B. Excision
C. left hepatectomy
D. liver transplant (not sure, but something wrong
Answer is: A
1294. Case of women have dysuria and dyspareunia with normal urinalysis and no fever?
Answer is: Urethral diverticulum, N.B: triad of dysuria, dyspareunia, urinary incontinence
1295. Female, multipara, complaining of 3 years history of urinary leak 4 times/day, and urgency
7- 8/day, affecting her life she avoided travel and she use pad for it, on examination marked
pelvic muscle weakness. What is the best initial management?
A. Anticholinergic drugs for 6 weeks
B. Pelvic m exercise and behavioural bladder therapy for 6 w.
C. Cystoscopy.
D. Urine culture. and ABx indicated
Answer is: B
1296. Ultrasound finding (which was provided) renal with multiple nodules?
Answer is: Polycystic kidney disease
1297. Pregnant women eat at nights and had heartburn symptoms during bed rest what is the
management?
Answer is: Lifestyle modification
196
1298. 27 yo lady c/o urine come out from the virginal, during micturition?
A. Urethrovaginal fistula
B. Ureterovaginal fistula
C. Vesicovaginal fistula
Answer is: A
1300. Patient RTA with head trauma, increased urine output, decrease in Urine osmolarity
increased blood osmolarity?
Answer is: Central diabetes insipidus
1302. Pt with bronchogenic carcinoma, presented with progressive SOB, there’s elevated jvp,
clear lung and quiet heart sounds. What will confirm your dx?
A. CXR
B. ECHO
C. ECG
Answer is: B
1303. 21 boy go to dentist clinic for tooth extraction he have bleeding after extraction and
bruises and he know case of factor VIII dif , lab show slight increase in PT?
A. Von wellbrand
B. haemophilia A
Answer is: A
1304. 65 y/o women come for annual check-up found to have harsh ejection systolic murmur
which propagated to the neck. What’s the following factors that decide the time of surgery?
A. low pulse pressure
B. Intensity of murmur
C. patient symptoms
D. left ventricular hypertrophy
Answer is: C
197
1305. Patient was found to have cavity on x ray, what is the type of precaution?
A. Airborne
Answer is: A
1306. known case of polymyalgia rheumatica presents with malignancy symptoms (weight
loss and other non-specific constitutional sx), what is the diagnosis? Labs showed anemia,
low platelets and increased WBC (lymphocytes)?
A. CLL
Answer is: A
1307. 50 yo pt with 3wk H of fatigue and tiredness and tonge and lower limb she also
has unilateral tongue fasciculation. diagnosis is?
A. myasthenia gravis
B. Myasthenic syndrome.
C. Mononeuropathy.
D. Motor neuron disease
Answer is: D
1310. Post chemotherapy with dry mouth what's the electrolytes abnormalities?
A. Hypercalcemia
B. hypocalcaemia
C. Hypernatremia
D. hyponatremia
Answer is: B
198
1311. 18 Y has ALL. 17 days after chemo he develops fever around 38.7 C. No focus of infection
found. Labs shows: low WBC 0.6 (normal was 4-10x..), Neutrophil level 60% (normal 40-
60%), i’m not sure if he has low platelets as well, what you’ll do?
A. Blood urine... culture and paracetamol
B. Blood, urine,... culture and PO Abx
C. Blood, urine,... culture and IV Abx
Answer is: C
1312. Female had gastroenteritis and she took metoclopramide that leads to involuntary
movement facial grimace and tongue protruding what to give how to manage?
A. Diphenhydramine
B. Epinephrine
C. Cyproheptadine
D. Tizanidine
Answer is: A
1315. Young female smoker patient recently diagnosed with SLE, life style modification?
Answer is: Smoking cessation
1316. Female pt with RA, using methotrexate. Labs were done 4 months ago and
it showed high LFT. US showed 5cm hopdense lesion on the upper border of
liver. Whats the most appropriate next step in management?
A. Stop methotrexate.
B. Resection of liver.
Answer is: A
199
1317. Grey Turner sign?
Answer is: Necrotizing pancreatitis
1319. Asthmatic pt with signs and symptoms (wheezing, difficulty breathing.) which of the
following acid base balance may be expected to this patient?
A. Metabolic acidosis
B. Metabolic alkalosis
Answer is: Respiratory acidosis
1321. DKA baby with nausea and vomiting and noticed weight loss, which acid base abnormality
you may found?
A. Respiratory alkalosis
B. Metabolic acidosis
C. Metabolic alkalosis
Answer is: B
1322. A tall patient complaining of dyspnoea and chest pain. CXR: Opacity on the upper border
of the right lung that is less than 3 cm. What's the management?
A. Tube thoracostomy.
B. US guided needle biopsy.
C. Observation.
Answer is: A
1323. Pt with hx of IHD with no fever, low CO, low wedge pressure and normal Rt ventricular
pressure. What’s the type of shock?
A. Septic
B. Cardiogenic
C. Anaphylactic
D. Hypovolemic
Answer is: D
200
1324. Pt with bleeding. INR 2.1 what to give?
A. Vitamin K
B. FFP
C. Blood transfusion
Answer is: A, if sever bleeding > B.
1325. Pt with CHF and A fib. What will you use to control his cardiac rhythm?
A. Bisoprolol
B. Adenosine
Answer is: Amiodarone, for rate > A.
1326. Asthma pt with 6-8 times night symptoms. She is on SABA. What to add?
A. Inhaled corticosteroids/formeterol.
B. Inhaled corticosteroids and anticholinergic.
C. Inhaled corticosteroids and albuterol as needed.
Answer is: A
1327. Elderly with constipation and left lower quadrant fullness. What's the diagnosis?
A. Constipation
B. Ureteric stone
C. Urinary tract infection
Answer is: A
1328. Adenoma was found incidentally in a pt complaining of recurrent headache. What's the
management?
A. Neurosurgery referral.
B. Order anterior pituitary hormones.
Answer is: B
1329. Elderly with progressive memory loss and personality changes. Living normally, he can
cook and do his daily activity. What's the management?
A. Refer to geriatric
B. Start risperidone
C. Transfer to a care facility
Answer is: A
201
1331. 70 years old admitted to day care in the past 2 weeks he had decreased interest,
decreased weight and appetite he is away from his family and he has short term memory
impairment diagnosis?
A. Alzheimer
B. depression
C. vascular dementia
Answer is: B
1332. patient dm and has symptoms of uti in lap high creatinine what’s contraindicated?
A. nitrofurantion
B. cipro
Answer is: A
1333. 60 yo alcoholic presents with chronic epigastric pain and diarrhea for 6 months, he
reported 5 bulky stool per day. Stool analysis show 42 gram of fat per day?
A. Lipas 30000 with each meal
B. Low fat diet
Answer is: A
1334. patient K/C of COPD came with dyspnoea and increase oral secretion and
breathlessness (don’t remember exactly the symptoms but sure about the results) O2: 83%,
RR: 7, PH: 7.24, Most appropriate management?
A. CPAP
B. MV
C. Facial mask
Answer is: B
1335. Patient with schizophrenia and he was on amisulpride and iloperidone for years (not sure
about the name of the second drug) came with orofacial abnormal movement for one
month .. What is the cause of his symptoms?
A. Travide dyskinasia
B. Symptoms of schizophrenia
Answer is: A
1336. Patients present with central obesity, facial changes, decrease lipido, abdominal
pigmentation, muscle weakness, back pain, Dx?
A. Cushing disease
B. Adrenal adenoma
C. Adrenal carcinoma
Answer is: A
202
1337. Patient with mid ejection systolic murmur radiate to carotid, echo show normal ventricle
(something like that and didn’t mention anything about EF) and aortic valve
is severely stenosed?
A. AVR
B. Follow up
Answer is: B
1338. 80 y/o patient admitted for pneumonia, at night she was found disoriented and
restless BP: 150/90, RR: 22, O2: 94. Best initial step the nurse should do?
A. Orient the patient and nasal O2
B. Restrict the patient
C. Lorazepam
D. Call her family to come and stay with the patient
Answer is: D
1339. Patient with hypothyroidism, you started thyroxine, she came after 2 weeks still
complaining of symptoms, TSH was still high .. What you should do?
A. Increase the dose and check after 3 weeks
B. Increase the dose and check after 6 weeks
C. Decrease the dose and check after 3 weeks
D. Decrease the dose and check after 6 weeks
Answer is: B
1340. 58 y/o patient with DVT, which of the following indicates thrombophilia?
A. Age
B. Hx of OCP
C. connective tissue disease
Answer is: C
1342. Question about patient diagnosed with bronchiectasis, right way informs the patient?
A. Tell him the diagnosis
B. Tell him you have inflammation in the lung...
Answer is: B
1343. Pregnant 24 weeks complain of fever, back pain and fatigue BP: normal, Temp: 38 Protein
++ Leukocyte 9 Management?
A. Oral abx
B. Admit and IV abx Can’t remember the rest
Answer is: B
203
1344. Patient with recurrent renal stones (they didn’t mention any other manifestation) Labs all
normal even calcium was normal, PTH very high Next step?
A. 24h calcium in urine
Answer is: A sestamibi
1345. Patient medically free, not on any medication, her BP 180/140 (not sure 140 or 120)
Labs all normal even sodium was normal (138) except potassium 2.4. What is your
diagnosis?
A. Primary hyperaldosteronism
B. Chronic hypertension
C. I forgot the rest
Answer is: A
1346. 45 years old male smoker complain of dyspnoea, haemoptysis, clubbing and other
features suggest bronchiectasis What is the highest diagnostic test of value?
A. CXR
B. Bronchoscope
C. High resolution CT
Answer is: C
1347. Patient on TPN then after few days INR increases, what is the cause?
A. Vit A
B. Vit K
C. Vit D
Answer is: B
1348. 25 years old female with nephrotic syndrome due to minimal change disease Which of the
following is the most effective in decreasing proteinuria?
A. Prednisolone
B. ACEI
Answer is: A
1349. Female post cholecystectomy complains of chills and rigors, all labs normal (BP, Temp, HR)
What is the cause?
A. Septic shock
B. Sepsis
C. Bacteremia
D. SIRS
Answer is: D
204
1350. Case of a patient with multiple vomiting and ECG showed flat T wave what you will find?
A. Alkali urine
B. Hyperkalaemia
C. Aciduria
Answer is: A
1351. A 25-year-old man was referred to the Endocrine Clinic with a recent history of fragility
fracture. He had no significant past medical history and was on no medication. On
examination, he had sparse facial and axillary hair growth. Which of the following
investigation useful in establishing the aetiology of his fragility fractures?
A. Calcitonin level
B. DEXA
C. Insulin like growth factor (IGF1)
D. testosterone and gonadotrophin levels
Answer is: D
1352. 60 yo alcoholic presents with chronic epigastric pain and diarrhea for 6 months, he
reported 5 bulky stool per day Stool analysis show 42 gram of fat per day?
A. Lipas 30000 with each meal
B. Low fat diet
Answer is: A
1353. patient K/C of COPD came with dyspnoea and increase oral secretion and breathlessness
(don’t remember exactly the symptoms but sure about the results)
O2: 83%
RR: 7
PH: 7.24
Most appropriate management?
A. CPAP
B. MV
C. Facial mask
Answer is: B
1354. Patients present with central obesity, facial changes, decrease lipido, abdominal
pigmentation, muscle weakness, back pain Dx?
A. Cushing disease
B. Adrenal adenoma
C. Adrenal carcinoma
Answer is: A
205
1355. Patient will do thyroid surgery with history of mitral valve prolapse and mitral
regurgitation?
A. Give prophylaxis abx for endocarditis
B. Give prophylaxis abx for wound infection
C. No indication for abx prophylaxis
Answer is: C
1356. Patient with mid ejection systolic murmur radiate to carotid, echo show normal ventricle
(something like that and didn’t mention anything about EF) and aortic valve is severely
stenosed?
A. AVR
B. Follow up
Answer is: B
1357. 80 y/o patient admitted for pneumonia, at night she was found disoriented and restless
BP: 150/90
RR: 22
O2: 94
Best initial step the nurse should do?
A. Orient the patient and nasal O2
B. Restrict the patient
C. Lorazepam
D. Call her family to come and stay with the patient
Answer is: D
1358. Patient with hypothyroidism, you started thyroxine, she came after 2 weeks still
complaining of symptoms, TSH was still high What you should do?
A. Increase the dose and check after 3 weeks
B. Increase the dose and check after 6 weeks
C. Decrease the dose and check after 3 weeks
D. Decrease the dose and check after 6 weeks
Answer is: B
1359. 58 y/o patient with DVT, which of the following indicates thrombophilia?
A. Age
B. Hx of OCP
C. connective tissue disease
Answer is: C
206
1361. Patient with recurrent renal stones (they didn’t mention any other manifestation)
Labs all normal even calcium was normal PTH very high Next step?
A. 24h calcium in urine
Answer is: A sestamibi
1362. Patient medically free, not on any medication, her BP 180/140 (not sure 140 or 120)
Labs all normal even sodium was normal (138) except potassium 2.4 What is your
diagnosis?
A. Primary hyperaldosteronism
B. Chronic hypertension
Answer is: A
1363. Child came from Africa with headache, fever, sore throat, absent reflexes, unable to raise
his head and knees while he is lying on his back, CSF shows normal glucose, normal protein
and lymphocytosis What is the organism?
A. Polio
B. CMV
C. EBV
D. Corona virus
Answer is: A
1364. 45 years old male smoker complain of dyspnea, hemoptysis, clubbing and other features
suggest bronchiectasis What is the highest diagnostic test of value?
A. CXR
B. Bronchoscope
C. High resolution CT
Answer is: C
1365. Patient on TPN then after few days INR increases, what is the cause?
A. Vit A
B. Vit K
C. Vit D
Answer is: B
1366. Female diagnosed with DM type 2, which of the following infections she is at risk of?
A. Candida
B. Bacterial vaginosis
C. Trichomoniasis
Answer is: A
207
1367. 25 years old female with nephrotic syndrome due to minimal change disease Which of the
following is the most effective in decreasing proteinuria?
A. Prednisolone
B. ACEI
Answer is: A
1368. Female complain of unilateral lower limb edema, bluish pigmentation, intact femoral and
distal pulses What is the next best step?
A. CT venography
B. US duplex
Answer is: B
1369. Case of a patient with multiple vomiting and ECG showed flat T wave what you will find?
A. Alkali urine
B. Hyperkalaemia
C. Aciduria
Answer is: C
1370. 30 years old female known case of epilepsy and poorly controlled, now came with fever
and productive cough (copious amount) with foul smelling and clubbing for 14 days, what is
the mechanism of this condition?
A. Aspiration
Answer is: A
1371. 29 years old male came for screening, his sister has polycystic kidney disease, what is the
best test to screen him?
A. US
B. CT
C. MRI
Answer is: A
1373. pt with cirrhosis and ascites, what is the best to prevent the recurrence?
A. TIPS
B. frousmaide and spironolactone
Answer is: B
1374. case of meningitis, (gram +ve cocci in chain) What is the Mx?
A. amoxicillin
B. ceftriaxone and vancomycin
Answer is: B
208
1375. typical case of brucellosis, what is the Mx?
A. doxycycline and streptomycin
B. rifampicin and Trimethoprim sulfamethoxazole
Answer is: A
1376. female with cough and cervical lymphadenopathy, lung auscultation bilateral crackles,
shine nodules, CXR bilateral lung infiltration and hilar lymphadenopathy, biopsy showed non
casating granulmoa , what is the Mx? NEW20
A. steroid
B. Anti TB
Answer is: A
1380. 55 years old male came with headache and tiredness, lab show high Hb (20) and slightly
elevation of platelet (420), wbc ( i forget), what is the dx?
A. polycythaemia rubra vera
B. myelofibrosis
C. essential thrombocytosis
Answer is: A
1381. male pt , knee septic arthritis, start empirical Abx (Cloxacillin), culture showed staph
aureus resistance, what to do?
A. continue same drug
B. start vancomycin
Answer is: B
1382. Pt came with abdominal cramps, vomiting and bloody diarrhea. What is the most common
organism?
A. Amebiasis
B. Tinae something
C. Ascariasis
Answer is: A
209
1383. 42 years old female had a multiple fractures and renal colic Lab were given all are normal
except for PTH was very high. On x ray renal stone were noticed What is the next appropriate
test?
A. Sestamibi scanning
B. Intravenous pylogram
C. urine ca 24h
Answer is: A
1386. DM with UTI symptom and inv show (high) creatinine, which drug contradicted?
A. Nitrofurantoin
B. Ciprofloxacin
Answer is: A
1387. RA pt on methotrexate and NSAID but still not control, what to add?
Answer is: Adenolimab
1388. long case and inv ask about Dx (high fritten- low lymphocyte)?
Answer is: Adult still disease
1389. pr on TB drug and has elevated uric acid level (they say it like that and) what is the cause?
A. Pyrazinamide
Answer is: A
1390. pt with sign of cushion (thin skin, puflohump ...etc) Not on any medication and lab show:
low ACTH what is the next test?
A. CT adrenal
B. MRI pituitary
Answer is: A
1392. DM and HTN on many drugs and have grade 3 limb edema , lab show low creatinine
clearnas what to do?
A. stop ACE
B. Thang frousimed to thiazide
Answer is: A
210
1393. pt with femoral #, they mentioned in the lab Low creatinine clearance, ask about DVT
prophylaxis?
A. enoxaparin
B. Heparin sulphate
Answer is: B
1395. Q about pt asymptomatic pt and no goiter His lab show: TSH (high) T4 (N) I think its sub
clinical hypothyroidism and ask what is the appropriate next step?
A. Reassurance and discharge
B. Repeat the test after 6w
C. give thyroxine
Answer is: according to TSH level, more than 10 > C, less than 10 > B.
1396. 30y pt with recurrent UTI and electrolytes disturbance, US show multiple thin wall cyst
throughout the parenchyma What is Dx?
A. Adult polycystic kidney
B. Medullary sponge kidney
Answer is: A
1397. Septic arthritis started on cloxacillin: report came after 3 days with staph aureus resistant
to cefotaxime (cefoxitin)?
A. start Vanco
B. add gents
C. continue cloxacillin
Answer is: A
1399. Patient known case of asthma on inhaled corticosteroids and short acting beta agonist as
needed what to add what?
Answer is: Add LABA
211
1402. Most common valvular malignancy?
Answer is: SCC
1405. 70 y/o female bed bounded due to basal ganglia (bleeding or damage can’t remember
clearly), weak mastication muscles, absent gag reflex, losing wt, (no mention of coma) how
are going to feed her?
Answer is: Jejunostomy tube
1406. Pt known case of asthma on nebulizer Ventolin and now she is pregnant what is your
treatment plan for her asthma (I wrote the meaning of the Q)?
Answer is: Continue the same
1407. Patient “forgot age” known history of diabetes mellitus, complaining of ptosis, inability to
adduct, elevate and depress right eye. Upon examination pupils are reactive bilaterally what
is the most common cause?
A. posterior communicating artery aneurysm
B. cavernous sinus thrombosis
C. 3rd nerve neuropathy
Answer is: C
1408. Pt with HTN, DM with well demarcated non tender swelling? NEW20
A. erysipelas
B. Necrobiosis Lipoidica Diabeticorum
Answer is: bad recall, A > is tender, B > is atrophy of skin.
1409. Retrosternal & epigastric pain & regurgitation & heart burn, did barium X ray show
dilatation of esophagus and lower esophagus stenosis and there’s multiple contraction in it
(no esophageal movement) what’s the diagnosis?
A. GERD
B. achalasia cardia
C. diffuse spasmatic esophagus
Answer is: B
212
1410. Heart burn & regurgitation NOT respond to PPI, endoscope see esophagitis, what you will
do?
A. PH 24 hrs
B. Manometry
Answer is: B
1411. Female pt C/O generalized muscle and bone ache associated with abdominal cramp or
epigastric this symptom resolves when she drink a cup of cold milk, what is the diagnosis?
Lab result: ca+2 = normal, phosphate = low, vit D3 = low, PTH = with upper normal level
A. milk alkali syndrome.
B. primary hyperparathyroidism
C. 2ndry hyperparathyroidism
Answer is: C
1415. Obese diabetic hypertensive female patient tried lifestyle but it was not effective. BMI is
28. What is the best drug with tolerable side effects?
A. Orlistat
B. Lorcaserin
C. Phentermine-topiramate
Answer is: A
1416. Pt known case of asthma on nebulizer vintolin and now she is pregnant what is your
treatment plan for her asthma (i wrote the meaning of the Q)?
Answer is: Continue same medications
213
1417. 66 years old patient come with progressive difficulty breath. In history he is being treated
for bronchogenic carcinoma. In P/E: JVP elevated, lung clear and heart sound very quiet.
What’s the confirmatory investigation?
A. CXR
B. ECHO
C. ECG
D. ABG
Answer is: B
1419. Patient was found to have cavity on x ray, what is the type of precaution?
Answer is: Airborne
1421. BB toxicity?
Answer is: give glucagon
1422. Patient with schizophrenia and he was on amisulpride and iloperidone for years (not sure
about the name of the second drug) came with orofacial abnormal movement for one month
What is the cause of his symptoms?
A. Travide dyskinasia
B. Symptoms of schizophrenia
Answer is: A
1423. Give vesicoureteral barium and there’s dilatation in one of ureter & bladder & pelvic?
A. cysto-ureteral reflux
Answer is: A
214
215